LSN Adult Health II Test 2

Pataasin ang iyong marka sa homework at exams ngayon gamit ang Quizwiz!

The nurse should teach the patient having a vasectomy that what occurs after the procedure? A. The amount of ejaculate will be noticeably decreased B. He may have difficulty maintaining an erection for several months C. An alternative form of contraception must be used for 6 to 8 weeks D. The testes will gradually decrease production of sperm and testosterone

C. An alternative form of contraception must be used for 6 to 8 weeks Rationale: Until sperm distal to the anastomotic site is ejaculated or absorbed by the body, the semen will contain sperm and alternative contraceptive methods must be used. When a postoperative semen examination reveals no sperm, the patient is considered sterile. Following vasectomy, there is rarely noticeable difference in the amount of ejaculate because ejaculate is primary seminal and prostatic fluid. Vasectomy does not cause erectile dysfunction (ED), nor does it affect testicular production of sperm or hormones

A 19 year old patient calls the school clinic and tells the nurse, "My menstrual period is very heavy this time. I have to change my tampon every 4 hours." Which action should the nurse take next? A. Tell the patient that her flow is not unusually heavy B. Schedule the patient for an appointment later that day C. Ask the patient how heavy her menstrual flow usually is D. Have the patient call again if the heavy flow continues

C. Ask the patient how heavy her menstrual flow usually is Rationale: Because a heavy menstrual flow is usually indicated by saturating a pad or tampon in 1 to 2 hours, the nurse should first assess how heavy the patient's usual flow is. There is no need to schedule the patient for an appointment that day. The patient may need to call again, but this is not the first action the nurse should take. Telling the patient that she does not have a heavy flow implies that the patient's concern is not important.

The patient arrives at the urgent care facility worried about the fish smell of her vaginal discharge. What does the nurse suspect will be diagnosed? A. Cervicitis B. Trichomoniasis C. Bacterial vaginosis D. Vulvovaginal candidiasis

C. Bacterial vaginosis Rationale: Bacterial vaginosis is characterized by watery vaginal discharge with a fishy odor. Cervicitis displays mucopurulent discharge and postcoital spotting. Trichomoniasis has frothy greenish or gray discharge, Vulvovaginal candidiasis has thick, white, curdy discharge

During neurologic assessment of the older adult, what should the nurse know is an effect of aging on the neurologic system? A. Absent deep tendon reflexes B. Below average intelligence score C. Decreased sensation of touch and temperature D. Forms the outer layer of protective membranes around the brain and spinal cord

C. Decreased sensation of touch and temperature Rationale: A decrease in sensory receptors caused by degenerative changes leads to a diminished sense of touch, temperature, and pain in the older adult. Reflexes are decreased but not normally absent and intelligence does not decrease, although there may be some memory loss. Hypothalamic modifications lead to increased frequency of spontaneous awakening with interrupted sleep and insomnia

The nurse is caring for a patient with pelvic inflammatory disease (PID) requiring hospitalization. Which nursing intervention will be included in the plan of care? A. Monitor liver function tests B. Use cold packs PRN for pelvic pain C. Elevate the head of the bed at least 30 degrees D. Teach the patient how to perform Kegel exercises

C. Elevate the head of the bed at least 30 degrees Rationale: The head of the bed should be elevated to at least 30 degrees to promote drainage of the pelvic cavity and prevent abscess formation higher in the abdomen. Although a possible complication of PID is acute perihepatitis, liver function test results will remain normal. There is no indication for increased fluid intake. Application of heat is used to reduce pain. Kegel exercises are not helpful in PID

The laboratory result of a specimen from a 20 year old female patient shows human papillomavirus (HPV). What would the nurse suspect the patient's diagnosis to be? A. Syphilis B. Gonorrhea C. Genital warts D. Genital herpes

C. Genital warts Rationale: Genital warts are caused by human pappilomavirus (HPV). Syphilis is caused by T. pallidum. Gonorrhea is caused by N. gonorrhoeae. Genital herpes are caused by HSV

A patient tells the nurse that she would like a prescription for oral contraceptives to control her premenstrual dysphoric disorder (PMD-D) symptoms. Which patient information that contraindicates oral contraceptives should be communicated to the health care provider? A. Bilateral breast tenderness B. Frequent abdominal bloating C. History of migraine headaches D. Previous spontaneous abortion

C. History of migraine headaches Rationale: Oral contraceptives are contraindicated in patients with a history of migraine headaches. The other patient information would not prevent the patient from receiving oral contraceptives

A 45 year old patient with chronic arterial disease has a brachial systolic blood pressure (SBP) of 132mmHg and an ankle SBP of 102mmHg. The ankle brachial index is______________ and indicates __________ (mild/moderate/severe) arterial disease

102/132= 0.77; mild

Following teaching about medications for PAD, the nurse determines that additional instruction is necessary when the patient says, A. I should take one aspirin a day to prevent clotting in my legs B. The lisinopril I use for my blood pressure may help me walk further without pain C. I will need to have frequent blood tests to evaluate the effect of the Trental I will be taking. D. Pletal should help me increase my walking distance and speed and help prevent pain in my legs

C. I will need to have frequent blood tests to evaluate the effect of the Trental I will be taking. Rationale: Trental allows blood cells to pass through small vessels, but there are no blood tests related to it. Warfarin (Coumadin), which needs INR blood tests, is not recommended for prevention of CVD events in patients with PAD. All of the other statements are correct in relation to treatment of PAD

What should be included in the management of a patient with delirium? A. The use of restraints to protect the patient from injury B. The use of short acting benzodiazepines to sedate the patient C. Identification and treatment of underlying causes when possible D. Administration of high doses of an antipsychotic drug such as haloperidol (Haldol)

C. Identification and treatment of underlying causes when possible Rationale: Care of the patient with delirium is focused on identifying and eliminating precipitating factors if possible. Treatment of underlying conditions, and discontinuing medications that induce delirium are important. Drug therapy is reserved for those patients with severe agitation because the drugs themselves may worsen delirium

What causes initial incomplete SCI to result in complete cord damage? A. Edematous compression of the cord above the level of the injury B. Continued trauma to the cord resulting from damage to stabilizing ligaments C. Infarction and necrosis of the cord caused by edema, hemorrhage, and metabolites D. Mechanical transection of the cord by sharp vertebral bone fragments after the injury

C. Infarction and necrosis of the cord caused by edema, hemorrhage, and metabolites Rationale: The primary SCI rarely affects the entire cord, but the pathophysiology of secondary injury may result in damage that is the same as mechanical severance of the cord. Complete cord dissolution occurs through autodestrution of the cord by hemorrhage, ischemia, edema, and the presence of metabolites, which lead to cell death and permanent neurologic deficit

Which information will the nurse teach a patient who has chronic prostatitis? A. Ibuprofen (Motrin) should provide good pain control B. Prescribed antibiotics should be taken for 7 to 10 days C. Intercourse or masturbation will help relieve symptoms D. Cold packs used every 4 hours will decrease inflammation

C. Intercourse or masturbation will help relieve symptoms Rationale: Ejaculation helps drain the prostate and relieve pain. Warm baths are recommended to reduce pain. Nonsteroidal antiinflammatory drugs (NSAIDs) are frequently prescribed but usually do not offer adequate pain relief. Antibiotics for chronic prostatitis are taken 4 to 12 weeks

The nurse has administered prescribed IV mannitol (Osmitrol) to an unconscious patient. Which parameter should the nurse monitor to determine the medication's effectiveness? A. Blood pressure B. oxygen saturation C. Intracranial pressure D. Hemoglobin and hematocrit

C. Intracranial pressure Rationale: Mannitol is an osmotic diuretic and will reduce cerebral edema and intracranial pressure. It may initially reduce hematocrit and increase blood pressure., but these are not the best parameters for evaluation of the effectiveness of the drug. O2 saturation will not directly improve as a result of mannitol administration.

For what purpose would the nurse use the Mini-Mental State examination to evaluate a patient with cognitive impairment? A. It is a good tool to determine the etiology of dementia B. It is a good tool to evaluate mood and thought processes C. It can help to document the degree of cognitive impairment in delirium and dementia D. It is useful for initial evaluation of mental status, but additional tools are needed to evaluate changes in cognition over time

C. It can help to document the degree of cognitive impairment in delirium and dementia Rationale: The Mini-Mental State examination is a tool to document the degree of cognitive impairment, and it can be used to determine a baseline from which changes over time can be evaluated. It does not evaluate mood or thought processes but can detect dementia and delirium and differentiate these from psychiatric mental illness. It cannot help to determine the etiology

The patient is receiving donepezil (Aricept), lorazepam (Ativan), risperidone (Risperdal), and sertraline (Zoloft) for the management of AD. What benzodiazepine medication is being used to help manage this patient's behavior? A. Sertraline (Zoloft) B. Donepezil (Aricept) C. Lorazepam (Ativan) D. Risperidone (Risperdal)

C. Lorazepam (Ativan) Rationale: Lorazepam (Ativan) is a benzodiazepine use to manage behavior with AD. Sertraline (Zoloft) is a selective serotonin reuptake inhibitor use to treat depression. Donepezil (Aricept) is a cholinesterase inhibitor used for decreased memory and cognition. Risperidone (Risperdal) is an antipsychotic used for behavior management

What should the nurse do when providing care for a patient with an acute attack of trigeminal neuralgia? A. Carry out all hygiene and oral care for the patient B. Use conversation to distract the patient from pain C. Maintain a quiet, comfortable, draft free environment D. Have thee patient examine the mouth after each meal for residual food

C. Maintain a quiet, comfortable, draft free environment Rationale: Because attacks of trigeminal neuralgia may be precipitated by hot or cold air mvovement on the face, jarring movements, or talking, the environment should be of moderate temperature and free of drafts, and patients should not be expected to converse during the acute period. Patients often prefer to carry out their own oral care because they are afraid someone else may inadvertently injure them or precipitate an attack. The nurse should stress that oral hygiene be performed because patients often avoid it, but residual food in the mouth after eating occurs more frequently with Bell's palsy

When caring for a patient on the first postoperative day after an abdominal aortic aneurysm repair, which assessment finding is most important for the nurse to communicate to the health care provider? A. Presence of flatus B. Hypoactive bowel sounds C. Maroon colored liquid stool D. Abdominal pain with palpation

C. Maroon colored liquid stool Rationale: Loose, bloody, maroon colored stools at this time may indicate intestinal ischemia or infarction and should be reported immediately because the patient may need an emergency bowel resection. The other findings are normal on the first postoperative day after abdominal surgery

In providing care for patients with chronic, progressive neurologic disease, what is the major goal of treatment that the nurse works toward? A. Meet the patient's personal care needs B. Return the patient to normal neurologic function C. Maximize the neurologic functioning for as long as possible D. Prevent the future development of additional chronic disease

C. Maximize the neurologic functioning for as long as possible Rationale: The nurse can attempt to assist patient's in maximizing neurologic function and self care abilities, alleviate physical symptoms, and prevent complications for as along as possible. Many chronic neurologic diseases involve progressive deterioration in physical or mental capabilities and have no cure, with devastating results for patients and families

A 19 year old patient visits the health care clinic for a routine check up. Which questions should the nurse ask to determine whether a Pap test is needed? A. "Have you had sexual intercourse?" B. "Do you use any illegal substances?" C. "Do you have cramping with your periods?" D. "At what age did your menstrual periods start?"

A. "Have you had sexual intercourse?" Rationale: The current American Cancer Society recommendation is that a Pap test be done every 3 years, starting 3 years after the first sexual intercourse and no later than age 21 years. The information about menstrual periods and substance abuse will not help determine whether the patient requires a Pap test

When admitting an acutely confused patient with a head injury, which action should the nurse take? A. Ask family members about the patient's health history B. Ask leading questions to assist in obtaining health data C. Wait until the patient is better oriented to ask questions D. Obtain only the physiologic neurologic assessment date

A. Ask family members about the patient's health history Rationale: When admitting a patient who is likely to be a poor historian, the nurse should obtain health history information from others who have knowledge about the patient's health. Waiting until the patient is oriented or obtaining only physiologic data will result in incomplete assessment data, which could adversely affect decision making about treatment. Asking leading questions may result in inaccurate or incomplete information

A 46 year old patient who has had blood drawn for an insurance screening has a positive Venereal Disease Research Laboratory (VDRL) test. Which action should the nurse take next? A. Ask the patient about past treatment for syphilis B. Explain the need for blood and spinal fluid cultures C. Schedule fluorescent treponemal antibody absorption (FAT-Abs) testing. D. Assess for the presence of chancres, flulike symptoms, or a rash on the trunk

A. Ask the patient about past treatment for syphilis Rationale: When antibody testing is positive for syphilis, the antibodies remain present for an indefinite period of time even after successful treatment, so the nurse should inquire about previous treatment before doing other assessments or testing. Culture, FAT-Abs testing, and assessment for symptoms may be appropriate based on whether the patient has been previously treated for syphilis

When assessing an aging adult man, what does the nurse note as a normal finding? A. Decreased penis size B. Decreased pubic hair C. A decrease in scrotal color D. Unilateral breast enlargement

A. Decreased penis size Rationale: A decrease in the size of the penis is a normal finding in the older man. Loss of pubic hair is not normal, not is the enlargement of one breast. The normally darker color of the scrotum does not change with aging

A 19 year old patient has genital warts around her external genitalia and perianal area. She tells the nurse that she not sought treatment until now because "the warts are so disgusting". Which nursing diagnosis is consistent with these data? A. Disturbed body image related to feelings about the genital warts B. Ineffective coping related to denial of increased risk for infection C. Risk for infection related to lack of knowledge about transmission D. Anxiety related to impact of condition on interpersonal relationships

A. Disturbed body image related to feelings about the genital warts Rationale: The patient's statement that her lesions are disgusting suggests that disturbed body image is the major concern. There is no evidence to indicate ineffective coping or lack of knowledge about mode of transmission. The patient may be experiencing anxiety, but there is nothing in the data indicating that the genital warts are impacting interpersonal relationships

Which structure of the female breast carries milk from the alveoli to the lactiferous sinuses? A. Ducts B. Areola C. Nipple D. Adipose

A. Ducts Ducts carry milk from the alveoli to the lactiferous sinuses. The areola is the pigmented center of the breast, the nipple is the erectile tissue that contains pores, and adipose tissue makes up the majority of the non lactating breast tissue

A patient reports feeling numbness and tingling of the left arm before experiencing a tonic-clonic seizure. The nurse determines that this history is consistent with what type of seizure? A. Focal B. Atonic C.Absence D.Myoclonic

A. Focal Rationale: The initial symptoms of a focal seizure involve clinical manifestations that are localized to a particular part of the body or brain. Symptoms of an absence seizure are staring and a brief loss of consciousness. In an atonic seizure, the patient loses muscle tone and (typically) falls to the ground. Myoclonic seizures are characterized by a sudden jerk of the body or extremities

Which stage of syphilis is identified by the absence of clinical manifestations and a positive fluorescent treponemal antibody absorption (FTA-Abs) test? A. Latent B. Primary C. Secondary D. Late (tertiary)

A. Latent Rationale: Lack of clinical manifestations but a positive treponemal antibody test with normal cerbrospinal fluid (CSF) occurs in the latent stage. The primary stage is characterized by a chancre, regional lymphadenopathy, and genital ulcers. The secondary stage has flu-like symptoms and cutaneous lesions. The late or tertiary stage is characterized by gummas, cardiovascular changes, and neurosyphilis

A patient who is 2 days post femoral popliteal bypass graft to the right leg is being cared for on the vascular unit. Which action by a licensed practical/vocational nurse (LPN/LVN) caring for the patient requires the registered nurse to intervene? A. The LPN/LVN has the patient to sit in a chair for 2 hours B. The LPN/LVN gives the prescribed aspirin after breakfast C. The LPN/LVN assists the patient to walk 40 feet in the hallway D. The LPN/LVN places the patient in Fowler's position for meals

A. The LPN/LVN has the patient to sit in a chair for 2 hours Rationale: The patient should avoid sitting for long periods because of the increased stress on the suture line caused by leg edema and because of the risk for venous thromboembolism (VTE). The other actions by the LPN/LVN are appropriate

The son of a patient with early-onset AD asks if he will get AD. What should the nurse tell this man about the genetics of AD? A. The risk for it is higher for the children of parents of early onset AD B. Women get AD more often than men do, so his chances of getting AD are slim C. The blood test for the ApoE gene to identify his type of AD can predict who will develop it D. This type of AD is not as complex as regular AD, so he does not need to worry about getting AD

A. The risk for it is higher for the children of parents of early onset AD Rationale: The risk is higher for the children of parents with early onset AD. Women do get AD more often than men, but that is more likely related to women living longer than men to the type of AD. Gene testing for ApoE-4 allele of the nineteenth chromosome is used for research with the late onset AD but does not predict who will develop the disease. Late-onset AD is more genetically complex than early onset AD and is more common in those over age 60.

The extent of urinary obstruction caused by BPH can be determined by which diagnostic study? A. Uroflowmetry B. A cystometrogram C. transrectal ultrasound D. Postvoiding catheterization

A. Uroflowmetry Rationale: Uroflowmeetry is used to measure the volume of urine expelled from the bladder to determine the extent of urethral blockage. Cystourethroscopy may also evaluate the degree of obstruction, but a cystometrogram measures bladder tone. A transrectal ultrasound may determine the size and configuration of the prostate gland. Postvoiding catheterization measure residual urine

A 47 year old patient who is experiencing hypogonadism has decided to try the testosterone gel Testim. What should the nurse teach the patient and his wife about the gel? A. Wash the hands with soap and water after applying it B. His wife should apply to help him feel better about using it C. Do not wear clothing over the area until it has been absorbed D. The gel may be taken buccally if it is not effective on the abdomen

A. Wash the hands with soap and water after applying it Rationale: The gel may spread the testosterone to others if it is not washed of his hands after application. If his wife applies the gel, she should wear gloves to prevent absorption of the testosterone and its effects on her body. Clothing over the area until it has dried is recommended. The gel is only topical; a buccal testosterone tablet is called Striant

A 56 year old patient is concerned about having a moderate amount of vaginal bleeding after 5 years of menopause. The nurse will anticipate teaching the patient about A. endometrial biopsy B. endometrial ablation C. uterine balloon therapy D. dilation and curettage (D&C)

A. endometrial biopsy Rationale: A postmenopausal woman with vaginal bleeding should be evaluated for endometrial cancer, and endometrial biopsy is the primary test for endometrial cancer. D&C will be needed only if the biopsy does not provide sufficient information to make a diagnosis. Endometrial ablation and balloon therapy are used to treat menorrhagia, which is unlikely in this patient

A patient scheduled for a prostatectomy for prostate cancer expresses the fear that he will have erectile dysfunction. In responding to this patient, the nurse should keep in mind that A. erectile dysfunction can occur even with a nerve sparing procedure B. the most common complication of this surgery is postoperative bowel continence C. retrograde ejaculation affects sexual function more frequently than erectile dysfunction D. preoperative sexual function is the most important factor in determining postoperative erectile dysfunction

A. erectile dysfunction can occur even with a nerve sparing procedure Rationale: A major complication after prostatectomy (even with nerve sparing procedures) is erectile dysfunction

A patient's eyes jerk while the patient looks to the left. The nurse will record this finding as A. nystagmus. B. CN VI palsy. C. ophthalmic dyskinesia. D. oculocephalic response.

A. nystagmus. Rationale: Nystagmus is defined as fine, rapid jerking movements of the eyes

Symptoms of BPH are primarily caused by A. obstruction of the urethra. B. untreated chronic prostatitis. C. decreased bladder compliance. D. excessive secretion of testosterone.

A. obstruction of the urethra Rationale: Benign prostatic hyperplasia (BPH) is a benign enlargement of the prostate gland. Enlargement of the prostate gradually compresses the urethra, eventually leading to partial or complete urethral obstruction. Compression of the urethra ultimately leads to development of clinical symptoms

A nurse will plan to teach a 67 year old patient who has been diagnosed with orchitis about A. pain management B. emergency surgery C. application of heat to the scrotum D. aspiration of fluid from the scrotal sac

A. pain management Rationale: Orchitis is very painful, and effective pain management will be needed. Heat, aspiration, and surgery are not used to treat orchitis

A patient has a tumor in the cerebellum. The nurse will plan interventions to A. prevent falls B. stabilize mood C. avoid aspiriation D. improve memory

A. prevent falls Rationale: Because functions of the cerebellum include coordination and balance, the patient with dysfunction is at risk for falls. The cerebellum does not affect memory, mood, or swallowing ability

A 20 year old female patient who is being seen in the family medicine clinic for an annual physical examination reports being sexually active. The nurse will plan to teach thee patient about A. testing for chlamydia infection B. immunization for herpes simplex C. infertility associated with the human papillomavirus (HPV) D. the relationship between the herpes virus and cervical cancer

A. testing for chlamydia infection Rationale: Testing for chlamydia is recommended by the Centers for Disease Control and Prevention for all sexually active women younger than age 25 years. HPV infection does not cause infertility. There is no vaccine available for herpes simplex, and herpes simplex infection does not cause cervical cancer

The nurse finds a patient in bed having a generalized tonic-clonic seizure. During the seizure activity, what actions should the nurse take first (select all that apply)? A. Loosen restrictive clothing B. Turn the patient to the side C. Protect the patient's head from injury D. Place a padded tongue blade between the patient's teeth E. Restrain the patient's extremities to prevent soft tissue and bone injury

A.B.C. Loosen restrictive clothing; Turn the patient to the side; Protect the patient's head from injury Rationale: The focus is on maintaining a patent airway and preventing patient injury. The nurse should not place objects in the patient's mouth or restrain the patient

Cultures used in the diagnosis of STIs can be obtained from (Select all that apply) A. urine B. vagina C. urethra D. rectum E. endocervix

A.B.C.D.E. urine, vagina, urethra, rectum, endocervix Rationale: Specimens for STI testing should be based on the type of sexual practices and risk for exposure. Samples for diagnostic STI testing can be obtained from urine, vagina, endocervix, urethra, and rectum

Which manifestation of menopause are related to estrogen deficiency? (select all that apply) A. Cessation of menses B. Breast engorgement C. Vasomotor instability D. Reduction of bone fractures E. Decreased cardiovascular risk

A.C. Cessation of menses; vasomotor instability Rationale: The lack of estrogen in menopause contributes to many of the signs of aging, including cessation of menses, vasomotor instability (hot flashes), atrophic changes of vaginal and external genitalia and breast tissue, increased risks for osteoporosis and coronary artery disease, redistribution of fat, muscle and joint pain, loss of skin elasticity, and atrophic lower urinary tract changes

During an assessment of the motor system, the nurse finds that the patient has a staggering gain and an abnormal arm swing. What should the nurse use this information to do? A. Assist the patient to cope with the disability B. Plan a rehabilitation program for the patient C. Protect the patient from injury caused by falls D. Help to establish a diagnosis of cerebellar dysfunction

C. Protect the patient from injury caused by falls Rationale: The nursing neurologic assessment of the motor system shows a potential for falls, which the nurse should help protect the patient from. Cerebellar dysfunction is present but a diagnosis cannot be established only with this information. Assisting the patient to cope or planning a rehabilitation program will also not occur with only this part of the assessment

A male patient who has a profuse, purulent urethral discharge with painful urination is seen at the clinic. Which information will be most important for the nurse to obtain? A. Sexual orientation B. Immunization history C. Recent sexual contacts D. Contraceptive preference

C. Recent sexual contacts Rationale: Information about sexual contacts is needed to help establish whether the patient has been exposed to a sexually transmitted infection and because sexual contacts also will need treatment. The other information also may be gathered but is not as important in determining the plan of care for the patient's current symptoms

Which type of seizure occurs in children, is also known as a petit mal seizure, and consists of a staring spell that lasts for a few seconds? A. Atonic B. Simple focal C. Typical absence D. Atypical absence

C. Typical focus Rationale. The typical or simple absence seizure is also known as petit mal and the child has staring spells that last for a few seconds. Atonic seizures occur when the patient falls from loss of muscle tone accompanied by brief unconsciousness. Simple focal seizures have focal motor, sensory, or autonomic symptoms related to the area of the brain involved without loss of consciousness. Staring spells in atypical absence seizures last longer than those in typical absence seizures and are accompanied by peculiar behavior during the seizure or confusion after the seizure

Which question will the nurse ask a patient who has been admitted with a benign occipital lobe tumor to assess for functional deficits? A. Do you have difficulty in hearing? B. Are you experiencing visual problems? C. Are you having any trouble with your balance? D. Have you developed any weakness on one side?

B. Are you experiencing visual problems? Rationale: Because the occipital lobe is responsible for visual reception, the patient with a tumor in this area is likely to have problems with vision. The other questions will be better for assessing the function of the temporal lobe, cerebellum, and frontal lobe.

The patient is diagnosed with a superficial vein thrombosis (SVT). Which characteristic should the nurse know about SVT? A. Embolization to lungs may result in death B. Clot may extend to deeper veins if untreated C. Vein is tender to pressure and there is edema D. Typically found in the iliac, inferior, or superior vena cava

B. Clot may extend to deeper veins if untreated Rationale: If left untreated, a superficial vein thrombosis (SVT) may extend to deeper veins and VTE may occur. VTE may embolize to the lungs and have tenderness to pressure and edema. SVTs usually occur in superficial leg veins and have a tenderness, itchiness, redness, warmth, pain, inflammation, and induration along the course of the superficial vein

A patient tells the nurse that he decided to seek treatment for erectile dysfunction (ED) because his wife "is losing patience with the situation." The nurse's follow up questions should focus on the man's identified concern with A. low self esteem B. role performance C. increased anxiety D. infrequent intercourse

B. role performance Rationale: The patient's statement indicates that the relationship with his wife is his primary concern. Although anxiety, low self-esteem, and ineffective sexuality patterns may also be concerns, the patient information suggests that addressing the role performance problem will lead to the best outcome for this patient

Postoperative nursing care for the woman with a gynecologic fistula includes (select all that apply) A. bed rest B. bladder training C. warm sitz bath D. perineal hygiene E. use of stool softeners

C.D. warm sitz bath; perineal hygiene Rationale: Postoperatively, perineal hygiene is important to prevent infection. Warm sitz baths should be taken three times daily if possible

A 49 year old patient tells the nurse that she is postmenopausal but has recently had occasional spotting. Which initial response by the nurse is appropriate? A. "A frequent cause of spotting is endometrial cancer." B. "How long has it been since your last menstrual period?" C. "Breakthrough bleeding is not unusual in women your age." D. "Are you using prescription hormone replacement therapy?"

D. "Are you using prescription hormone replacement therapy?" Rationale: In postmenopausal women, a common cause of spotting is hormone replacement therapy. Becuase breakthrough bleeding may be a sign of problems such as cancer or infection, the nurse would not imply that this is normal. The length of time since the last menstrual period is not relevant to the patient's symptoms. Although endometrial cancer may cause spotting, this information is not appropriate as an initial response

Which patient is most at risk for developing delirium? A. A 50 year old woman with cholecystitis B. A 19 year old man with a fractured femur C. A 42 year old woman having an elective hysterectomy D. A 78 year old man admitted to the medical unit with complications related to heart failure

D. A 78 year old man admitted to the medical unit with complications related to heart failure Rationale: Risk factors that can precipitate delirium include age of 65 years or older, male gender, and severe acute illness (e.g. heart failure). The 78 year old man has the most risk factors for delirium

What would best indicate successful achievement of outcomes for the patient with cranial surgery? A. Ability to return home in 6 days B. Ability to meet all self care needs C. Acceptance of residual neurologic deficits D. Absence of signs and symptoms of increased ICP

D. Absence of signs and symptoms of increased ICP Rationale: The primary goal after cranial surgery is prevention of increased ICP, and interventions to prevent ICP and infection postoperatively are nursing priorities. The residual deficits, rehabilitation potential, and ultimate function of the patient depend on the reason for surgery, the postoperative course, and the patient's general state of health

Which topic should the nurse include in patient teaching for a patient with a venous stasis ulcer on the left lower leg? A. Need to increase carbohydrate intake B. Methods of keeping the wound area dry C. Purpose of prophylactic antibiotic therapy D. Application of elastic compression stockings

D. Application of elastic compression stockings Rationale: Compression of the leg is essential to healing of venous stasis ulcers. High dietary intake of protein rather than carbohydrates is needed. Prophylactic antibiotics are not routinely used for venous ulcers. Moist dressings are used to hasten wound healing

What is the protective fluid of the central nervous system (CNS)? A. Synaptic cleft B. Limbic system C. Myelin sheath D. Cerebrospinal fluid

D. Cerebrospinal fluid Rationale: The protective fluid of the central nervous system (CNS) is cerebrospinal fluid (CSF). The synaptic cleft is the space where neurotransmitters cross from neuron to neuron. The limbic system is the area of the brain concerned with emotion, aggression, feeding behavior, and sexual response. Myelin sheath is the insulator for the conduction of impulses in the CNS and peripheral nervous system (PNS)

The patient has been diagnosed with a cerebral concussion. What should the nurse expect to see in this patient? A. Deafness, loss of taste, and CSF otorrhea B. CSF otorrhea, vertigo, and Battle's sign with a dural tear C. Boggy temporal muscles because of extravasation of blood D. Headache, retrograde amnesia, and transient reduction in LOC

D. Headache, retrograde amnesia, and transient reduction in LOC Rationale: A cerebral concussion may include a brief disruption in LOC, retrograde amnesia, and a headache, all of short duration. A parietal fracture may have deafness, loss of taste, and CSF otorrhea. A basilar skull fracture may have a dural tear with CSF or brain otorrhea, rhinorrhea, hearing difficulty, vertigo, and Battle's sign, or CSF otorrhea

Following a TURP, a patient has continuous bladder irrigation. Four hours after surgery, the catheter is draining thick, bright red clots and tissue. What should the nurse do? A. Release the traction on the catheter. B. Clamp the drainage tube and notify the patient's health care provider. C. Manually irrigate the catheter until the drainage is clear D. Increase the rate of the irrigation and take the patient's vital signs

D. Increase the rate of the irrigation and take the patient's vital signs Rationale: Bleeding and blood clots from the bladder are expected after a TURP, and continuous irrigation is used to keep clots from obstructing the urinary tract. The rate of the irrigation may be titrated to keep the clots from forming, if ordered, but the nurse should also check the vital signs because hemorrhage is the most common complication of prostatectomy. The traction on the catheter applies pressure to the operative site to control bleeding and should be relieved on schedule. The catheter will be manually irrigated only to release a blockage. Clamping the drainage tube is contraindicated because it would distend the bladder.

What differentiates chronic bacterial prostatitis from acute prostatitis? A. Postejaculatory pain B. Frequency, urgency, and dysuria C. Symptoms of urinary tract infection (UTI) D. Most common reason for recurrent UTIs in adult men

D. Most common reason for recurrent UTIs in adult men Rationale: Chronic bacterial prostatitis commonly causes UTIs in adult men and recurs frequently. The other options are true of both chronic and acute prostatitis, although not as severe with chronic

The classic manifestations associated with Parkinson's disease is tremor, rigidity, akinesia, and postural instability. What is a consequence related to rigidity? A. Shuffling gait B. Impaired handwritting C. Lack of postural stability D. Muscle soreness and pain

D. Muscle soreness and pain Rationale: The degeneration of dopamine-producing neurons n the substantia nigra of midbrain and basal ganglia lead to these signs. Muscle soreness, pain, and slowness of movement are patient function consequences related to rigidity. Shuffling gait, absent arm swing while walking, absent blinking, masked facial expression, saliva drooling, and difficulty initiating movement are all related to akinesia. Impaired handwritting and hand activities are related to the tremor of Parkinson's disease (PD). Being unable to stop themselves from going forward or backward is from postural instability

The health care provider has prescribed bed rest with the feet elevated for a patient admitted to the hospital with venous thromboembolism. Which action by the nurse to elevate the patient's feet is best? A. The patient is placed in the Trendelenburg position B. Two pillows are positioned under the affected leg C. The bed is elevated at the knee and pillows are placed under the feet D. One pillow is placed under the thighs and two pillows are placed under the lower legs

D. One pillow is placed under the thighs and two pillows are placed under the lower legs Rationale: The purpose of elevating the feet is to enhance venous flow from the feet to the right atrium, which is best accomplished by placing two pillows under the feet and one under the thighs. Placing the patient in the Trendelenburg position will lower the head below heart level, which is not indicated for this patient. Placing pillows under the calf or elevating the bed at the knee may cause blood stasis at the calf level

The patient is suspected of having a new brain tumor. Which test will the nurse expect to be ordered to detect a small tumor? A. CT scan B. Angiography C. Electroencephalography (EEG) D. Positron emission tomography (PET) scan

D. Positron emission tomography (PET) scan Rationale: Th positron emission tomography (PET) scan or MRI is used to reliably detect very small tumors. The CT and brain scans are used to identify the location of a lesion. Angiography could be used to determine blood flow to the tumor and further localize it. Electroencephalography (EEG) would be used to rule out seizures

During examination of the female reproductive system, the nurse would note which finding is abnormal? A. clear vaginal discharge B. Perineal episiotomy scars C. Nonpalpable Skene's glands D. Reddened base of the vulva

D. Reddened base of the vulva Rationale: The vulva should be the color of the skin or slightly pink. Redness indicates inflammation and possible genital herpes. A small amount of clear vaginal discharge is normal in females, as are episiotomy scars in women who have had children. Skene's glands should not be palpable

The cremasteric reflex is absent in which problem of the scrotum and testes? A. Hydrocele B. Varicocele C. Spermatocele D. Testicular torsion

D. Testicular torsion Rationale: The cremasteric reflex is elicited by light stroking of the inner aspect of the thigh in a downward direction with a tongue blade. In testicular torsion, or a twisted spermatic cord that supplies blood to the testes and epididymis, this reflex is absent on the swollen side. Hydrocele is scrotal lymphedema from interference with lymphatic drainage of the scrotum Varicocele is dilation of the veins that drain the testes. Spermatocele is a sperm containing cyst of the epididymis

A 62 year old patient who has Parkinson's disease is taking bromocriptine (Parlodel). Which information obtained by the nurse may indicate a need for a decrease in the dosage? A. The patient has a chronic dry cough B. The patient has four loose stools a day C. The patient develops a deep vein thrombosis D. The patient's blood pressure is 92/52 mmHg

D. The patient's blood pressure is 92/52 mmHg Rationale: Hypotension is an adverse effect of bromocriptine, and the nurse should check with the health care provider before giving the medication. Diarrhea, cough, and deep vein thrombosis are not associated with bromocriptine use

In planning care and patient teaching for the patient with venous leg ulcers, the nurse recognizes that the most important intervention in healing and control of this condition is A. discussing activity guidelines B. using moist environment dressings C. taking horse chestnut seed extract daily D. applying graduated compression stockings

D. applying graduated compression stockings Rationale: Compression is essential for treating chronic venous insufficiency (CVI), healing venous ulcers, and preventing ulcer recurrence. Use of custom ftted graduated compression stockings is one option for compression therapy

A patient in the sexually transmitted infection clinic has a positive Venereal Disease Research Laboratory (VDRL) test, but no chancre is visible on assessment. The nurse will plan to send specimens for A. Gram stain B. cytologic studies C. rapid plasma reagin (RPR) agglutination D. fluorescent treponemal antibody absorption (FTA-Abs)

D. fluorescent treponemal antibody absorption (FTA-Abs) Rationale: Because false positives are common with VDRL and RPR testing, FTA-Abs testing is recommended to confirm a diagnosis of syphilis. Gram staining is used for other sexually transmitted infections (STIs) such as gonorrhea and Chlamydia and cytologic studies are used to detect abnormal cells (e.g. neoplastic cells)

Which of these nursing actions for a patient with Guillain-Barré syndrome is appropriate for the nurse to delegate to an experienced unlicensed assistive personnel (UAP)? A. Nasogastric tube feeding q4hr B. artificial tear administration q2hr C. Assessment for bladder distention q2hr D. passive range of motion to extremities q4hr

D. passive range of motion to extremities q4hr Rationale: Assisting a patient with movement is included in UAP education and scope of practice. Administration of tube feedings, administration of ordered medications, and assessment are skills requiring more education and expanded scope of practice, and the RN should perform these skills

Dementia is defined as a A. syndrome that results only in memory loss B. disease associated with abrupt changes in behavior C. disease that is always due to reduced blood flow to the brain D. syndrome characterized by cognitive dysfunction and loss of memory

D. syndrome characterized by cognitive dysfunction and loss of memory Rationale: Dementia is a syndrome characterized by dysfunction in loss or memory, orientation, attention, language, judgement, and reasoning. Personality changes and behavioral problems such as agitation, delusions, and hallucinations may result

When teaching a patient with problems of pelvic support to perform Kegel exercises, what should the nurse tell the patient to do? A. Contract the muscles used to stop rectal gas expulsion B. Tighten the lower abdominal muscles over the bladder area C. Squeeze all of the perineal muscles as if trying to close the vagina D. Lie on the floor and do leg lifts to strengthen the abdominal muscles

A. Contract the muscles used to stop rectal gas expulsion Rationale: The muscles that should be exercised are those affected by trying to stop an expulsion of gas from the rectum or stop urine midflow. Kegel exercises help to strengthen muscular support of the perineum, pelvic floor, and bladder and are also beneficial for problems with pelvic support and stress incontinence

During rehabilitation, a patient with a spinal cord injury begins to ambulate with long leg braces. Which level of injury does the nurse associate with this degree of recovery? A. L1-2 B. T6-7 C. T1-2 D. C7-8

A. L1-2 Rationale: During rehabilitation, the patient with SCI at L1-2 is able to maintain good sitting balance and full use of a wheelchair. The patient ambulates with long leg braces. Patients with higher level injury are unable to attain this degree of ambulation

A patient is admitted to the ED with a possible cervical SCI following an automobile crash. During admission of the patient, what is the highest priority for the nurse? A. Maintaining a patent airway B. Assessing the patient for head and other injuries C. Maintaining immobilization of the cervical spine D. Assessing the patient's motor and sensory function

A. Maintaining a patent airway Rationale: The need for a patent airway is the first priority for any injured patient, and a high cervical injury may decrease the gag reflex and the ability to maintain an airway as well as the ability to breathe. Maintaining cervical stability is then a consideration, along with assessing for other injuries and the patient's neurologic status

During admission of a patient with a severe head injury to the emergency department the nurse places the highest priority on assessment for A. Patency of airway B. Presence of neck injury C. Neurologic status with Glasgow coma scale D. Cerebrospinal fluid leakage from the ears or nose

A. Patency of airway Rationale: The nurse's initial priority in the emergency management of a patient with a severe head injury is to ensure that the patient has a patent airway

What should the nurse include when teaching the patient with acute PID how to care for herself? A. Promote rest in Semi Fowler's position B. Perform vaginal irrigations every 4 hours C. Instruct the patient to use tampons to control vaginal discharge D. The patient should ambulate frequently to promote drainage of exudate

A. Promote rest in Semi Fowler's position Rationale: Physical rest in semi-Fowler's position promotes drainage of the pelvic cavity by gravity and may prevent the development of abscesses high in the abdomen. Coitus, douching, and tampon use should be avoided to prevent spreading infection upward from the vagina, although frequent perineal care should be performed to remove infectious drainage

When planning care for the patient with trigeminal neuralgia, which patient outcome should the nurse set as the highest priority? A. Relief of pain B. Protection of the cornea C. Maintenance of nutrition D. Maintenance of positive body image

A. Relief of pain Rationale: The pain of trigeminal neuralgia is excruciating, and it may occur in clusters that continue for hours. The condition is considered benign with not major effects except the pain. Corneal exposure is a problem in Bell's palsy or it may occur following surgery for the treatment of trigeminal neuralgia. Maintenance of nutrition is important but not urgent, because chewing may trigger trigeminal neuralgia and patients then avoid eating. Except during an attack, there is no change in facial appearance in a patient with trigeminal neuralgia, and body image is more disturbed in response to the paralysis typical of Bell's palsy

A college athlete is seen in the clinic 6 weeks after a concussion. Which assessment information will the nurse collect to determine whether the patient is developing postconcussion syndrome? A. Short term memory B. Muscle coordination C. Glasgow Coma Scale D. Pupil reaction to light

A. Short term memory Rationale: Decreased short term memory is one indication of postconcussion syndrome. The other data may be assessed but are not indications of postconcussion syndrome

During assessment of the patient with vulvar, what should the nurse expect to find? A. Soreness and itching of the vulva B. Labial lesions with purulent exudate C. Severe excoriation of the labia and perineum D. Painless, firm nodules embedded in the labia

A. Soreness and itching of the vulva Rationale: Early signs of cancer of the vulva include pruritus, or burning, soreness of the vulva, and discharge or bleeding of the vulva, with edema of the vulva and lymphadenopathy occurring as the disease progresses. Labial lesions and excoriation more commonly occur with infections, and nodules are more often cysts or lipomas

While admitting a 42 year old patient with a possible brain injury after a car accident to the emergency department (ED), the nurse obtains the following information. Which finding is most important to report to the health care provider?> A. The patient takes warfarin (Coumadin) daily. B. The patient's blood pressure is 162/94 mmHg C. The patient is unable to remember the accident D. The patient complains of a severe dull headache

A. The patient takes warfarin (Coumadin) daily. Rationale: The use of anticoagulants increases the risk for intracranial hemorrhage and should be immediately reported. The other information would not be unusual in a patient with a head injury who had just arrived in the ED

A 50 year old patient is diagnosed with uterine bleeding caused by leiomyoma. Which information will the nurse include in the patient teaching plan? A. The symptoms may decrease after the patient undergoes menopause B. The tumor size is likely to increase throughout the patient's lifetime C. Aspirin or acetaminophen may be used to control mild to moderate pain D. The patient will need frequent monitoring to detect any malignant changes

A. The symptoms may decrease after the patient undergoes menopause Rationale: Leiomyomas appear to depend on ovarian hormones and will atrophy after menopause, leading to a decrease in symptoms. Aspirin use is discouraged because the antiplatelet effects may lead to heavier uterine bleeding. The size of the tumor will shrink after menopause. Leiomyomas are benign tumors that do not undergo malignant changes

A patient is on the surgical unit after a radical abdominal hysterectomy. Which finding requires a report to the health care provider? A. Urine output of 125 mL in the first 8 hours after surgery B. Decreased bowel sounds in all four abdominal quadrants C. One-inch area of bloody drainage on the abdominal dressing D. Complaints of abdominal pain at the incision site with coughing

A. Urine output of 125 mL in the first 8 hours after surgery Rationale: The decreased urine output indicates possible low blood volume and further assessment is needed to assess for possible internal bleeding. Decreased bowel sounds, minor drainage on the dressing, and abdominal pain with coughing are expected after this surgery

Which cancer is associated with intrauterine exposure to diethylstilbestrol (DES) or metastasis from another gynecologic cancer? A. Vaginal B. Ovarian C. Cervical D. Endometrial

A. Vaginal Rationale: Vaginal cancer is usually related to metastases of other cancers or intrauterine exposure to diethylstilbestrol (DES). Cervical, endometrial, and ovarian cancer have other or unknown causes

For a 65 year old woman who has lived with a T1 spinal cord injury for 20 years, which of the following health teaching instructions should the nurse emphasize? A. a mammogram needed every year B. bladder function tends to improve with age C. heart disease is not common in persons with spinal cord injury D. as a person ages the need to change body position is less important

A. a mammogram needed every year Rationale: Health promotion and screening are important for an older patient with a spinal cord injury. Older adult women with spinal cord injuries should perform monthly breast examination and undergo yearly mammography

The nurse observes a patient ambulating in the hospital hall when the patient's arms and legs suddenly jerk and the patient falls to the floor. The nurse will first A. assess the patient for injury B. give the scheduled divalproex (Depakote) C. document the timing and description of the seizure D. notify the patient's health care provider about the seizure

A. assess the patient for injury Rationale: The patient who had had a myoclonic seizure and fall is at risk for head injury and should first be evaluated and treated for this possible complication., Documentation of the seizure, notification of the health care provider, and administration of antiseizure medications are also appropriate actions, but the initial action should be assessment for injury

Which syndrome of incomplete SCI is described as cord damage common in the cervical region resulting in greater weakness in upper extremities than lower? A. central cord syndrome B. anterior cord syndrome C. posterior cord syndrome D. cauda equina and conus medullaris syndromes

A. central cord syndrome Rationale: In central cord syndrome, the motor weakness and sensory loss are present in upper extremities; lower extremities are not usually affected

A priority goal of treatment for the patient with AD is to A. maintain patient safety. B. maintain or increase body weight. C. return to a higher level of self-care. D. enhance functional ability over time.

A. maintain patient safety. Rationale: The overall management goals that the patient with AD will (1) maintain functional ability for as long as possible, (2) be maintained in a safe environment with a minimum of injuries, (3) have personal care needs met, and (4) have dignity maintained. The nurse should place emphasis on patient safety and planning and providing nursing care

When performing discharge teaching for a patient after a vasectomy, the nurse instructs the patient that he A. should continue to use other methods of birth control for 6 weeks B. should not have sexual intercourse until his 6 week follow up C. may have temporary erectile dysfunction (ED) because of swelling D. will notice a decrease in the appearance and volume of his ejaculate

A. should continue to use other methods of birth control for 6 weeks Rationale: Because it takes 6 weeks to evacuate sperm that are distal to the vasectomy site, the patient should use contraception for 6 weeks. ED that occurs after vasectomy is psychologic in origin and not related to postoperative swelling. The patient does not need to abstain from intercourse. The appearance and volume of the ejaculate are not changed because sperm are a minor component of the ejaculate

Why is the Glasgow Coma Scale used? A. to quickly assess the LOC B. to assess the patient's ability to communicate C. to assess the patient's ability to respond to commands D. to assess the patient's coordination with motor responses

A. to quickly assess the LOC Rationale: The Glasgow Coma Scale (GCS) is used to quickly assess the LOC with a standardized system. The three areas assessed are the patient's ability to open eyes, speak, and obey commands to verbal or painful stimulus. Although best motor response is an indicator, it is not used to assess coordination

Social effects of a chronic neurologic disease include (select all that apply) A. divorce. B. job loss. C. depression. D. role changes. E. loss of self-esteem.

A.B.C.D.E. Divorce, job loss, depression, role changes, loss of self esteem Rationale: Social problems related to chronic neurologic disease may include changes in roles and relationships (e.g. divorce, job loss, roll changes); other psychologic problems (e.g. depression, loss of self esteem) also may have social effects

Which events cause increased ICP (select all that apply)? A. vasodilation B. necrotic tissue edema C. blood vessel compression D. edema from initial brain insult E. brainstem compression and herniation

A.B.D. Vasodilation; necrotic tissue edema; edema from initial brain insult Rationale: Increased ICP is caused by vasodilation and edema from an initial brain insult or necrotic tissue. Blood vessel compression and brainstem compression and herniation occur as a result of increased ICP

When caring for a patient who experienced a T2 spinal cord transection 24 hours ago, which collaborative and nursing actions will the nurse include in the plan of care (select all that apply)? A. urinary catheter care B. nasogastric (NG) tube feeding C. continuous cardiac monitoring D. Administration of H2 receptor blockers E. maintenance of a warm room temperature

A.C.D.E. urinary catheter care; continuous cardiac monitoring; administration of H2 receptor blockers; maintenance of a warm room temperature Rationale: The patient is at risk for bradycardia and poikilothermia caused by sympathetic nervous system dysfunction and should have continuous cardiac monitoring and maintenance of a relatively warm room temperature. To avoid bladder distention a urinary retention catheter is used during this acute phase. Stress ulcers are a common complication, but can be avoided through the use of H2 receptor blocker such as famotidine. Gastrointestinal motility is decreased initially, and NG suctioning is indicated

Which statement by the patient indicates that the nurse's teaching about treating vaginal candidiasis was effective? A. "I will tell my partner that we cannot have intercourse for a month." B. "I should clean carefully after each urination and bowel movement." C. "I can douche with warm water if the itching continues to bother me." D. "I will insert the antifungal cream right before I get up in the morning."

B. "I should clean carefully after each urination and bowel movement." Rationale: Cleaning of the perineal area will decrease itching caused by contact of the irritated tissues by bacteria in the stool. Sexual intercourse should be avoided for 1 week. Douching will disrupt normal protective mechanisms in the vagina. The cream should be used at night so that it will remain in the vagina for longer periods of time

The patient is diagnosed with chronic inflammatory demyelinating polyneuropathy (CIDP) after nerve conduction velocity test. How will this patient with CIDP be treated differently than a patient with Guillain Barre syndrome? A. rehabilitation B. Corticosteroids C. Plasmapheresis D. IV immunoglobulin

B. Corticosteroids Rationale: The patient with chronic inflammatory demyelinating polyneuropathy (CIDP) will respond to corticosteroids and the patient with Guillain Barre syndrome will not. Both patients will benefit from rehabilitation, plasmapheresis, and IV immunoglobulin

Which finding would the nurse expect when assessing the legs of a patient who has a lower motor neuron lesion? A. Spasticity B. Flaccidity C. No sensation D. Hyperactive reflexes

B. Flaccidity Rationale: Because the cell bodies of lower motor neurons are located in the spinal cord, damage to the neuron will decrease motor activity of the affected muscles. Spasticity and hyperactive reflexes are caused by upper motor neuron damage. Sensation is not impacted by motor neuron lesions

Mitoxantrone is being considered as treatment for a patient with progressive relapsing MS. The nurse explains that a disadvantage of this drug compared to other drugs used for MS is what? A. It must be give subcutaneously every day B. It has a lifetime dose limit because of cardiac toxicity C. It is a muscle relaxant that increases the risk of drowsiness D. It is an anticholinergic agent that causes urinary incontinence

B. It has a lifetime dose limit because of cardiac toxicity Rationale: Mitoxantrone cannot be used for more that 2 to 3 years because it is an antineoplastic drug that causes cardiac toxicity, leukemia, and infertility. It is an immunosuppressant given IV monthly when patients inadequate responses to other drugs

A 72 year old man asks the nurse whether it is normal for him to become impotent at his age. The best response by the nurse includes what information? A. Most decreased sexual function in older adults is due to psychologic stress B. Physiologic changes of aging may require increased stimulation for an erection to occur C. Although the penis decreases in size in older men, there should be no change in sexual function D. Benign changes in the prostate gland that occur with aging can cause a decreased ability to attain an erection

B. Physiologic changes of aging may require increased stimulation for an erection to occur Rationale: Age related changes in sexual function in men include a need for increased stimulation for an erection and a possible decreased response to sexual stimulation. There may be a decreased ability to attain an erection, but it is not related to prostatic changes. A negative social attitude toward sexuality in older adults may also affect the sexual activity of people in this age group

Which cerebrospinal fluid analysis result will be most important for the nurse to communicate to the health care provider? A. Specific gravity 1.007 B. Protein 65 mg/dL (0.65 g/L) C. Glucose 45 mg/dL (1.7 mmol/L) D. White blood cell (WBC) count 4 cells/L

B. Protein 65 mg/dL (0.65 g/L) Rationale: The protein level is high. The specific gravity, WBCs, and glucose values are normal

When providing care for a patient with ALS, the nurse recognizes what as one of the most distressing problems experienced by the patient? A. Painful spasticity of the face and extremities B. Retention of cognitive function with total degeneration of motor function C. Uncontrollable writhing and twisting movements of the face, limbs, and body D. Knowledge that there is 50% chance the disease has been passed to any offspring

B. Retention of cognitive function with total degeneration of motor function Rationale: In ALS there is gradual degeneration of motor neurons with extreme muscle wasting from lack of stimulation and use. However, cognitive function is not impaired and patients feel trapped in a dying body. Chorea manifested by writhing, involuntary movements is characteristic of HD. As an autosomal dominant genetic disease, HD also has a 50% chance of being passed to each offspring

A hospitalized patient with a history of cluster headache awakens during the night with a severe stabbing headache. Which action should the nurse take first? A. Put a moist hot pack on the patient's neck B. Start the prescribed PRN O2 at 6L/min C. Give the ordered PRN acetaminophen (Tylenol) D. Notify the patient's health care provider immediately

B. Start the prescribed PRN O2 at 6L/min Rationale: Acute treatment for cluster headache is administration of 100% O2 at 6 to 8L/min. If the patient obtains relief with the O2, there is no immediate need to notify the health care provider. Cluster headaches last only 60 to 90 minutes, so oral pain medications have minimal effect. Hot packs are helpful for tension headaches but are not as likely to reduce pain associated with a cluster headache

During preoperative preparation of the patient scheduled for an AAA, why should the nurse establish baseline data for the patient? A. All physiologic processes will be altered postoperatively B. The cause of the aneurysm is a systemic vascular disease C. Surgery will be canceled if any physiologic function is not normal D. BP and HR will be maintained well below baseline levels during the postoperative period

B. The cause of the aneurysm is a systemic vascular disease Rationale: Because atherosclerosis is a systemic disease, the patient with an AAA is likely to have cardiac, pulmonary, cerebral, or lower extremity vascular problems that should be noted and monitored throughout the perioperative. Postoperatively, the BP is balanced: high enough to keep adequate flow through thee artery to prevent thrombosis but low enough to prevent bleeding at the surgical site

Which assessment data for a patient with Guillain Barre syndrom will require the nurse's most immediate action? A. The patient's sacral area skin is reddened B. The patient is continuously drooling saliva C. the patient complains of severe pain in the feet D. the patient's blood pressure (BP) is 150/82mmHg

B. The patient is continuously drooling saliva Rationale: Drooling indicates decreased ability to swallow, which places the patient at risk for aspiration and requires rapid nursing and collaborative actions such as suctioning and possible endotracheal intubation. The foot pain should be treated with appropriate analgesics, the BP requires ongoing monitoring, and skin integrity requires intervention, but these actions are not as urgently needed as maintenance of respiratory function

The nurse expects the assessment of a patient who is experiencing a cluster headache to include A. nuchal rigidity B. Unilateral ptosis C. projectile vomiting D. throbbing, bilateral facial pain

B. Unilateral ptosis Rationale: Unilateral eye edema, tearing, ptosis are characteristic of cluster headaches. Nuchal rigidity suggests meningeal irritation, such as occurs with meningitis, Although nausea and vomiting may occur with migraine headaches, projectile vomiting is more consistent with increased intracranial pressure. Unilateral sharp, stabbing pain, rather than throbbing pain, is characteristic of cluster headaches

What is the best explanation of stereotactic radiosurgery? A. Radioactive seeds are implanted in the brain B. Very precisely focused radiation destroys tumor cells C. Tubes are placed to redirect CSF from one are to another D. The cranium is opened with removal of a bone flap to open the dura

B. Very precisely focused radiation destroys tumor cells Rationale: A stereotactic radiosurgery technique uses precisely focused radiation to destroy tumor cells. The radiation is computer and imagery guided. Brachytherapy uses radioactive seeds to deliver radiation. Ventricular shunts are used to redirect CSF from one area to another. A craniotomy is done by first making burr holes and then opening the cranium by connecting the holes to remove a flap of bone to expose the dura mater.

In telling a patient with infertility what she and her partner can expect, the nurse explains that A. ovulatory studies can help determine tube patency B. a hysterosalpingogram is a common diagnostic study C. the cause will remain unexplained for 40% of couples D. if postcoital studies are normal, infection tests will be done

B. a hysterosalpingogram is a common diagnostic study Rationale: Tubal factors (i.e. occlusion or deformity) are assessed most commonly by means of a hysterosalpingogram

The nurse will plan to teach a 34 year old patient diagnosed with stage 0 cervical cancer about A. radiation B. conization C. chemotherapy D. radical hysterectomy

B. conization Rationale: Because the carcinoma is in situ, conization can be used for treatment. Radical hysterectomy, chemotherapy, or radiation will not be needed

A couple is scheduled to have a Huhner test for infertility. In preparation for the test, the nurse will instruct the couple about A. being sedated during the procedure B. determining the estimated time of ovulation C. experiencing shoulder pain after the procedure D. refraining from intercourse before the appointment

B. determining the estimated time of ovulation Rationale: For the Huhner test, the couple should have intercourse at the estimated time of ovulation and then arrive for the test 2 to 8 hours after intercourse. The other instructions would be used for other types of fertility testing

The early stage of AD is characterized by A. no noticeable change in behavior B. memory problems and mild confusion C. increased tie spent sleeping or in bed D. Incontinence, agitation, and wandering behavior

B. memory problems and mild confusion Rationale: An initial sign of AD is a subtle deterioration in memory

The priority nursing assessment for a patient being admitted with a brainstem infarction is A. pupil reaction B. respiratory rate C. reflex reaction time D. level of consciousness

B. respiratory rate Rationale: Vital centers that control respiration are located in the medulla and part of the brainstem, and will require priority assessments because respiratory function may be life threatening. The other information will also be obtained by the nurse but is not as urgent

An age related finding during the assessment of the older woman's reproductive system is A. dysparenuia B. vaginal atrophy C. nipple enlargement D. increased vulvar skin turgor

B. vaginal atrophy Rationale: With advancing age, changes occur in the female reproductive system. Many of these changes are related to the alteration in estrogen production that is associated with menopause. A reduction in circulating estrogen and an increase in androgens in postmenopausal women are associated with breast and genital atrophy

Which assessments will the nurse make to monitor a patient's cerebellar function (Select all that apply)? A. Test for graphesthesia B. Observe arm swing with gait C. Perform the finger to nose test D. Assess heat and cold sensation E. Measure strength against resistance

B.C. Observe arm swing with gait; Perform the finger to nose test Rationale: The cerebellum is responsible for coordination and is assessed by looking at the patient's gait and the finger to nose test. The other assessments will be used for other parts of the neurologic assessment

Which laboratory tests are used to diagnose Chlamydia (select all that apply)? A. Pap test B. Gram stain C. Rapid plasma reagin (RPR) D. Nucleic acid amplified test (NAAT) E. Venereal Disease Research Laboratory (VDRL) F. Fluorescent treponemal anitbody absorption (FTA-Abs)

B.D. Gram stain; Nucleic acid amplified test (NAAT) Rationale: Gram stain smears, nucleic acid amplification test (NAAT), and cultures can screen for Chlamydia from vaginal, endocervical, urinary, and urethral samples. The NAAT can also be used to detect gonorrhea. A Pap test detects potentially cancerous cells. The rapid plasma reagin (RPR), the Venereal Disease Research Laboratory (VDRL), and fluorescent treponemal antibody absorption (FTA-Abs) tests all screen for syphilis

A patient has an induced abortion with suction cutterage at an ambulatory surgical center. Which instructions will the nurse include when discharging the patient? A. "Avoid contraceptives until your reexamination B. "Heavy vaginal bleeding is expected for 2 weeks." C. "Abstain from sexual intercourse for the next 2 weeks" D. "Irregular menstrual periods are expected for a few months"

C. "Abstain from sexual intercourse for the next 2 weeks" Rationale: Because infection is a possible complication of this procedure, the patient is advised to avoid intercourse until the reexamination in 2 weeks. Patients may be started on contraceptives on the day of the procedure. The patient should call the doctor if heavy vaginal bleeding occurs. No change in the regularity of the menstrual periods is expected

The nurse has just received change of shift report about the following four patients. Which patient should be assessed first? A. A patient with a cervical radium implant in place who is crying in her room B. A patient who is complaining of 5/10 pain after an abdominal hysterectomy C. A patient with a possible ectopic pregnancy who is complaining of shoulder pain D. A patient in the fifteenth week of gestation who has uterine cramping and spotting

C. A patient with a possible ectopic pregnancy who is complaining of shoulder pain Rationale: The patient with the ectopic pregnancy has symptoms consistent with rupture and needs immediate assessment for signs of hemorrhage and possible transfer to surgery. The other patients should also be assessed as quickly as possible but do not have symptoms of life threatening complications

An unconscious patient is admitted to the emergency department (ED) with a head injury. The patient's spouse and teenage children stay at the stay at the patient's side and ask many questions about the treatment being given. What action is best for the nurse to take? A. Call the family's pastor or spiritual advisor to take them to the chapel B. Ask the family to stay in the waiting room until the assessment is completed C. Allow the family to stay with the patient and briefly explain all procedures to them D. Refer the family members to the hospital counseling service to deal with their anxiety

C. Allow the family to stay with the patient and briefly explain all procedures to them Rationale: The need for information about the diagnosis and care is very high in family members of acutely ill patients. The nurse should allow the family to observe care and explain the procedures unless they interfere with emergent care needs. A pastor or counseling service can offer some support, but research supports information as being more effective. Asking the family to stay in the waiting room will increase their anxiety

In establishing screening programs for populations at high risk for STIs, the nurse recognizes that which microorganism causes nongonococcal urethritis in men and cervicitis in women? A. Treponema pallidum B. Neisseria gonorrhoeae C. Chlamydia trachomatis D. Herpes simplex virus (HSV)

C. Chlamydia trachomatis Rationale: Chlamydia trachomatis can cause nongonococcal urethritis in men and cervicitis in women. Herpes simplex virus (HSV) causes genital herpes. Treponema pallidum causes syphilis. Neisseria gonorrhoeae causes gonorrheae.

The nurse is obtaining the pertinent health history for a man being evaluated for infertility. Which question focuses on a possible cause of infertility? A. Are you circumcised? B. Have you had surgery for phimosis? C. Do you use medications to improve muscle mass? D. Is there a history of prostate cancer in your family?

C. Do you use medications to improve muscle mass? Rationale: Testosterone or testosterone-like medications may adversely affect sperm count. The other information will be obtained in the health history but does not affect the patient's fertility

What is demonstrated when the patient stands with the feet close together and eyes closed and the patient sways or falls? A. Pronator drift B. Absent patellar reflex C. Positive Romberg test D. Absence of two point discrimination

C. Positive Romberg test Rationale: A positive Romberg test is demonstrated when the patient is unable to maintain balance with the feet together and then closing the eyes. Pronator drift is observed when the patient holds both arms fully extended at shoulder level in front of him with the palms upward and eyes closed; the downward drift and palm pronation indicates a problem in the opposite motor cortex. Absent patellar reflex is when there is no response to striking the patellar tendon just below the patella. Absence of two point discrimination is seen when the two points of a calibrated compass are on the tips of the fingers and are not recognized as two distinct points

A 49 year old man who has type 2 diabetes , high blood pressure, hyperlipidemia, and gastroesophageal reflux tells the nurse that he has had recent difficulty in achieving an erection. Which of the following drugs from his current medications list may cause erectile dysfunction (ED)? A. Ranitidine (Zantac) B. Atorvastatin (Lipitor) C. Propranolol (Inderal) D. Metformin (Glucophage)

C. Propranolol (Inderal) Rationale: Some antihypertensives may cause ED, and the nurse should anticipate a change in antihypertensive therapy. The other medications will not affect erectile function

Which action by a new nurse who is giving fondaparinux (Arixtra) to a patient with a lower leg venous thromboembolism (VTE) indicates that more education about the drug is needed? A. The nurse avoids rubbing the injection site after giving the drug B. The nurse injects the drug into the abdominal subcutaneous tissue C. The nurse ejects the air bubble from the syringe before giving the drug D. The nurse does not check partial thromboplastin time (PTT) before giving the drug

C. The nurse ejects the air bubble from the syringe before giving the drug Rationale: The air bubble is not ejected before giving fondiparinux to avoid loos of the drug. The other actions by the nurse are appropriate for subcutaneous administration of a low molecular weight heparin (LMWH). LMWHs typically do not require ongoing PTT monitoring and dose adjustment

What happens at the synapse? A. The synapse physically joins two neurons B. The nerve impulse is transmitted only from one neuron to another neuron C. The presynaptic terminal submits a nerve impulse through the synaptic cleft to the receptor site on the postsynaptic cell D. When a presynaptic cell releases excitatory neurotransmitters, the postsynaptic cell depolarizes enough to generate an action potential

C. The presynaptic terminal submits a nerve impulse through the synaptic cleft to the receptor site on the postsynaptic cell Rationale: The presynaptic terminal submits neurotransmitter impulses through they synaptic cleft to the receptor site on the postsynaptic cell, either another neuron or a gland or muscles. If there are enough presynaptic cells releasing excitatory neurotransmitters on a single neuron, the sum of their input is enough to generate an action potential. The synapse is not a physical connection between neurons, as there is a space between them where the neurotransmitter goes from one neuron to another

During care of the patient following femoral bypass graft surgery, the nurse immediately notifies the health care provider (HCP) if the patient experiences A. fever and redness at the incision site B. 2+ edema of the extremity and pain at the incision site. C. a loss of palpable pulses and numbness and tingling of the feet D. increasing ankle brachial indices and serous drainage from the incision

C. a loss of palpable pulses and numbness and tingling of the feet Rationale: Loss of palpable pulses, numbness, and tingling of the extremity, pallor, cyanosis, or cold are indications of occlusion of the bypass graft and need immediate medical attention. Pain, redness, and serous drainage at the incision site are expected postoperatively. Ankle brachial index measurements are not recommended due to increased risk for graft thrombosis, but this would decrease with occlusion

The nurse performing a focused assessment of left posterior temporal lobe functions will assess the patient for A. sensation of the left side of the body B. reasoning and problem solving ability C. ability to understand written and oral language D. voluntary movements on the right side of the body

C. ability to understand written and oral language Rationale: The posterior temporal lobe integrates the visual and auditory input for language comprehension. Reasoning and problem solving are functions of the anterior frontal lobe. Sensation on the left side of the body is located in the right postcentral gyrus. Voluntary movement on the right side is controlled in the left precentral gyrus

When evaluating outcomes of a glycerol rhizotomy for a patient with trigeminal neuralgia, the nurse will A. assess if the patient is doing daily facial exercises B. question if the patient is using an eye shield at night C. ask the patient about social activities with family and friends D. remind the patient to chew on the unaffected side of the mouth

C. ask the patient about social activities with family and friends Rationale: Because withdrawal from social activities is a common manifestation of trigeminal neuralgia, asking about social activities will help in evaluating if the patient's symptoms have improved. Glycerol rhizotomy does not damage the corneal reflex or motor functions of the trigeminal nerve, so there is no need to use an eye shield, do facial exercises, or take precautions with chewing

During assessment of a patient with SCI, the nurse determines that the patient has a poor cough with diaphragmatic breathing. Based on this finding, what should be the nurse's first action? A. Institute frequent turning and repositioning B. Use tracheal suctioning to remove secretions C. assess lung sounds and respiratory rate and depth D. Prepare the patient for endotracheal intubation and mechanical ventilation

C. assess lung sounds and respiratory rate and depth Rationale: Because pneumonia and atelectasis are potential problems related to ineffective coughing and the loss of intercostal and abdominal muscle function, the nurse should assess the patient's breath sounds and respiratory function to determine whether secretions are being retained or whether there is progression of respiratory impairment. If the patient cannot count to 10 aloud without taking a breath, immediate attention is needed. Suctioning is not indicated unless lung sounds indicate retained secretions. Position changes will help to mobilize secretions. Intubation and mechanical ventilation are used if the patient becomes exhausted from labored breathing or if arterial blood gases (ABGs) deteriorate

When administering a mental status examination to a patient with delirium, the nurse should A. wait until the patient is well rested B. administer an anxiolytic medication C. choose a place without distracting stimuli D. reorient the patient during the examination

C. choose a place without distracting stimuli Rationale: Because overstimulation by environmental factors can distract the patient from the task of answering the nurse's questions, these stimuli should be avoided. The nurse will not wait to give the examination because action to correct the delirium should occur as soon as possible. Reorienting the patient is not appropriate during the examination. Antianxiety medications may increase the patient's delirium

The plan of care for a patient immediately after a perineal radical prostatectomy will include decreasing the risk for infection related to A. urinary incontinence B. prolonged urinary stasis C. fecal wound contamination D. suprapubic catheter placement

C. fecal wound contamination Rationale: The perineal approach increases the risk for infection because of the incision is located to close to the anus, and contamination with the feces is possible. Urinary stasis and incontinence do not occur because the patient has a retention catheter in place for 1 to 2 weeks. A urethral catheter is used after the surgery

On physical examination of a patient with headache and fever, the nurse should suspect a brain abscess when the patient has A. Seizures B. nuchal rigidity C. focal symptoms D. signs of increased ICP

C. focal symptoms Rationale: The symptoms of brain abscess closely resembles those of meningitis and encephalitis, including fever, headache, nausea, vomiting, and increased ICP, except that the patient also may have some focal symptoms that reflect the local area of the abscess

During the neurologic assessment, the patient is unable to respond verbally to the nurse but cooperates with the nurse's directions to move his hands and feet. The nurse will suspect A. cerebellar injury. B. a brainstem lesion. C. frontal lobe damage. D. a temporal lobe lesion.

C. frontal lobe damage. Rationale: Expressive speech (ability to express the self in language) is controlled by Broca's area in the frontal lobe. The temporal lobe contains Wernicke's area, which is responsible for receptive speech (ability to understand language input). The cerebellum and brainstem do not affect higher cognitive functions such as speech

What is the most important measure in the treatment of venous leg ulcers? A. Elevation of the affected leg B. Application of topical antibiotics C. graduated compression stockings D. application of moist to dry dressings

C. graduated compression stockings Rationale: Although leg elevation, moist dressings, and systemic antibiotics are useful in treatment of venous stasis ulcers, the most important factor is compression, which minimizes venous stasis, venous hypertension, and edema and prevents recurrence. Compression may be applied with various methods including stockings, elastic bandages or wraps, or a Velcro wrap, among others

A patient is diagnosed with moderate dementia after multiple strokes. During assessment of the patient, the nurse would expect to find, A. excessive nighttime sleepiness B. difficulty eating and swallowing C. loss of recent and long term memory D. Fluctuating ability to perform simple tasks

C. loss of recent and long term memory Rationale: Loss of both recent and long term memory is characteristic of moderate dementia. Patients with dementia have frequent nighttime awakening. Dementia is progressive, and the patient's ability to perform tasks would not have periods of improvement. Difficulty eating and swallowing is characteristic of severe dementia

A patient is hospitalized with new onset Guillain Barre syndrome. The most essential assessment for the nurse to complete is A. determining level of consciousness B. checking strength of the extremities C. observing respiratory rate and effort D. monitoring the cardiac rate and rhythm

C. observing respiratory rate and effort Rationale: The most serious complication of Guillain Barre syndrome is respiratory failure, and the nurse should monitor respiratory function continuously. The other assessments will also be included in nursing care, but they are not as important as respiratory assessment

The nurse performing an assessment with a patient who has chronic peripheral artery disease (PAD) of the legs and an ulcer on the right second toe would expect to find A. dilated superficial veins. B. swollen, dry, scaly ankles. C. prolonged capillary refill in all the toes. D. a serosanguineous drainage from the ulcer.

C. prolonged capillary refill in all the toes. Rationale: Capillary refill is prolonged in PAD because of the slower and decreased blood flow to the periphery. The other listed clinical manifestations are consistent with chronic venous disease.

A 65 year old woman was just diagnosed with Parkinson's disease. The priority nursing intervention is A. searching the Internet for educational videos B. evaluating the home for environmental safety C. promoting physical exercise and a well balanced diet D. designing an exercise program to strengthen and stretch specific muscles

C. promoting physical exercise and a well balanced diet Rationale: Promotion of physical exercise and a well balanced diet are major concerns of nursing care for patients with Parkinson's disease

A female patient tells the nurse that she has been having nightmares and acute anxiety around men being sexually assaulted 3 months ago. The most appropriate nursing diagnosis for the patient it A. anxiety related to effects of being raped B. sleep deprivation related to frightening dreams C. rape-trauma syndrome related to rape experience D. ineffective coping related to inability to resolve incident

C. rape-trauma syndrome related to rape experience Rationale: The patient's symptoms are most consistent with the nursing diagnoses of sleep deprivation, ineffective coping, and anxiety address some aspects of the patient's symptoms but do not address the problem as completely as the rape-trauma syndrome diagnosis

An abnormal finding noted during physical assessment of the male reproductive system is A. descended testes. B. symmetric scrotum. C. slight clear urethral discharge. D. the glans covered with prepuce.

C. slight clear urethral discharge Rationale: In physical assessment of the male external genitalia, urethral discharge is an abnormal finding. Normal findings include diamond shaped hair distribution, and a circumcised or uncircumcised penis. (In circumcision, the prepuce is removed.) If the male patient is uncircumcised, the glans is covered by the prepuce; no penile lesions or penile discharge is observed; the scrotum is symmetric, no masses are identified, testes are descended; and no inguinal hernia is detected.

During neurologic testing, the patient is able to perceive pain elicited by pinprick. Based on this finding, the nurse may omit testing for A. position sense B. patellar reflexes C. temperature perception D. heel-to-shin movements

C. temperature perception Rationale: If pain sensation is intact, assessment of temperature sensation may be omitted because both sensations are transmitted by the same ascending pathways

A 50 year old woman weighs 95kg and has a history of tobacco use, high blood pressure, high sodium intake, and sedentary lifestyle. When developing an individualized care plan for her, the nurse determines that the most important risk factors for peripheral artery disease (PAD) that must be modified are A. weight and diet B. activity level and salt intake C. tobacco use and high blood pressure D. sedentary lifestyle and exercise training

C. tobacco use and high blood pressure Rationale: Significant risk factors for peripheral artery disease include tobacco use, hyperlipidemia, elevated levels of high sensitivity C-reactive protein, diabetes mellitus, and uncontrolled hypertension; the most important is tobacco use. Other risk factors include family history, hypertriglyceridemia, hyperuricemia, increasing age, obesity, sedentary lifestyle, and stress

When taking a nursing history from a patient with BPH, the nurse would expect the patient to report A. nocturia, dysuria, and bladder spasms. B. urinary frequency, hematuria, and perineal pain. C. urinary hesitancy, postvoid dribbling, and weak urinary stream. D. urinary urgency with a forceful urinary stream and cloudy urine.

C. urinary hesitancy, postvoid dribbling, and weak urinary stream. Rationale: Classic symptoms of uncomplicated BPH are those associated with irritative symptoms, including nocturia, frequency, urgency, dysuria, bladder pain, and incontinence associated with inflammation of infection. Obstructive symptoms caused by prostate enlargement include diminished caliber and force of the urinary stream, hesitancy or difficulty initiating voiding, intermittent urination, dribbling at the end of urination, and a feeling of incomplete bladder emptying because of urinary retention. Bladder spasms, hematuria, perineal pain, and cloudy urine do not occur with BPH

What is the most common screening intervention for detecting BPH in men over 50? A. PSA level B. Urinalysis C. Cystoscopy D. Digital rectal exam

D. Digital rectal exam Rationale: The prostate gland can be easily palpated by rectal examination, and enlargement of the gland is detected early if yearly examinations are performed. IF symptoms of prostatic hyperplasia are present, further diagnostic testing, including prostate-specific antigen (PSA), a urinalysis, and a cystoscopy may be indicated

Which aneurysm is uniform in shape and a circumferential dilation of the artery? A. False aneurysm B. Pseudoaneurysm C. Saccular aneurysm D. Fusiform aneurysm

D. Fusiform aneurysm Rationale: The fusiform aneurysm is circumferential and relatively uniform in shape. The false aneurysm or pseudoaneurysm is not an aneurysm but a disruption of all of the arterial wall layers with bleeding that is contained by surrounding anatomic structures. Saccular aneurysms are the pouchlike bulge of an artery

When assessing the body functions of a patient with increased ICP, what should the nurse assess first? A. Corneal reflex testing B. pupillary reaction to light C. Extremity strength testing D. circulatory and respiratory status

D. circulatory and respiratory status Rationale: Of the body functions that should be assessed in an unconscious patient, cardiopulmonary status is the most vital function and gives priorities to the ABCs (airway, breathing, circulation).

A patient is suspected of having a brain tumor. The signs and symptoms include memory deficits, visual disturbances, weakness of right upper and lower extremities and personality changes. The nurse recognizes that the tumor is most likely located in the a. Frontal lobe b. Parietal lobe c. Occipital lobe d. Temporal lobe

a. Frontal lobe Rationale: A unilateral frontal lobe tumor may result in the following signs and symptoms: unilateral hemiplegia, seizures, memory deficit, personality and judgement changes, and visual disturbances. A bilateral frontal lobe tumor may cause symptoms associated with a unilateral frontal lobe tumor and an ataxic gait

What is one indication for early surgical therapy of the patient with a spinal cord injury? a. There is incomplete cord lesion involvement. b. The ligaments that support the spine are torn. c. A high cervical injury causes loss of respiratory function. d. Evidence of continued compression of the cord is apparent.

d. Evidence of continued compression of the cord is apparent. Rationale: Although surgical treatment of SCIs often depends on the preference of the HCP, surgery is usually indicated when there is continued compression of the cord by extrinsic forces or when there is evidence of cord compression. Other indications may include progressive neurologic deficit, compound fracture of the vertebra, bony fragments, and penetrating wounds of the cord

To prevent pregnancy in a patient who has been sexually assaulted, the nurse in the emergency department will plan to teach the patient about the use of A. mifepristone b. dilation and evacuation. c. methotrexate with misoprostol. d. levonorgestrel (Plan-B One-Step).

d. levonorgestrel (Plan-B One-Step). Rationale: Plan B One step reduces the risk of pregnancy when taken within 72 hours of intercourse. The other methods are used for therapeutic abortion but not for pregnancy prevention after unprotected intercourse

Which factors are associated with endometrial cancer? (Select all that apply) A. Obesity B. Smoking C. Family history D. Early sexual activity E. Early menarche and late menopause F. Unopposed estrogen only replacement therapy

A.B.F. Obesity, Smoking, Unopposed estrogen only replacement therapy Rationale: Endometrial cancer is at higher risk in obese patients because adipose cells store estrogen, which is the major risk factor, especially unopposed estrogen. Smoking is a risk factor for endometrial and cervical cancer. Early sexual activity is a risk factor for cervical cancer. Family history and early menarche and late menopause causing increased menstrual cycles are risk factors for ovarian cancer

Which condition is transmitted through a wound contamination, causes painful tonic spasms and seizures, and can be prevented by immunization? A. Tetanus B. Botulism C. Neurosyphilis D. Systemic inflammatory response system

A. Tetanus Rationale: Tetanus is transmitted through wound contamination, causes painful tonic spasms or seizures, and can be prevented with immunization

Which action should the nurse take when a 35 year old patient has a Pap test result of minor cellular changes? A. Teach the patient about a colposcopy B. Teach the patient about punch biopsy C. Schedule another Pap test in 4 months D. Administer human papillomavirus (HPV) vaccine

C. Schedule another Pap test in 4 months Rationale: Patients with minor changes on the Pap test can be followed with Pap tests every 4 to 6 months because these changes may revert to normal. Punch biopsy or colposcopy may be used if the Pap test shows more prominent changes. The HPV vaccine may reduce the risk for cervical cancer, but it is only recommended for ages 9 through 26 years

What describes Montgomery's tubercles in the female breast? A. Store milk during lactation B. Secrete milk during lactation C. Sebaceous-like glands on areola D. Erectile tissues containing pores

C. Sebaceous-like glands on areola Rationale: Montgomery's tubercles are similar to sebaceous glands and lubricate the nipple. Milk is stored in the lactiferous sinuses and secreted from the alveoli during lactation. The nipple has erectile tissue and contains pores for milk delivery

A 33 year old patient with multiple sclerosis (MS) is to begin treatment with glatiramer acetate (Copaxone). Which information will the nurse include in patient teaching? A. Recommendation to drink at least 4L of fluid daily B. Need to avoid driving or operating heavy machinery C. How to draw up and administer injections of the medication D. use of contraceptive methods other than oral contraceptive

C. How to draw up and administer injections of the medication Rationale: Copaxone is administered by self injection. Oral contraceptives are an appropriate choice for birth control. There is no need to avoid driving or drink large fluid volumes when taking glatiramer

Which area of the brain regulates functions of the endocrine system and autonomic nervous system? A. Basal ganglia B. Temporal lobe C. Hypothalamus D. Reticular activating system

C. Hypothalamus Rationale: The hypothalamus regulates functions of the endocrine system and autonomic nervous system (ANS). The basal ganglia function includes initiation, execution, and completion of voluntary movements, learning, emotional response, and automatic movements associated with skeletal muscle activity. The temporal lobe integrates somatic, visual, and auditory data and contains Wernicke's area. The reticular activating system regulates arousal and sleep wake transitions with communication among the brainstem, and the cerebral cortex

The nurse should advise the woman recovering from surgical treatment of an ectopic pregnancy that A. she has an increased risk for salpingitis B. bed rest must be maintained for 12 hours to assist in healing C. having one ectopic pregnancy increases her risk for another one D. intrauterine devices and infertility treatments should be avoided

C. having one ectopic pregnancy increases her risk for another one Rationale: Risk factors for ectopic pregnancy include a history of pelvic inflammatory disease, prior ectopic pregnancy, current use of a progestin releasing intrauterine device (IUD), failure of progestin only birth control, prior pelvic or tubal surgery, and procedures used in infertility treatment

When assessing a patient with possible peripheral artery disease (PAD), the nurse obtains a brachial blood pressure of 147/82 mm Hg and an ankle pressure of 112/74 mmHg. The nurse calculates the patient's ankle-brachial index (ABI) as

0.76 Rationale: The ABI is calculated by dividing the ankle systolic BP by the brachial systolic BP

A patient with an intracranial problem does not open his eyes to any stimulus, has no verbal response except moaning and muttering when stimulated, and flexes his arm in response to painful stimuli. What should the nurse record as the patient's GCS score? A. 6 B. 7 C. 9 D. 11

B. 7 Rationale: No opening of eyes=1; incomprehensible words=2; flexion withdrawal=4. Total=7

Calculate the CPP for the patient with an ICP of 34mmHg and a systemic BP of 108/64mmHg.

45mmHg Rationale: MAP=DBP= 1/3(SBP-DBP)= 64+15=79 CPP= MAP-ICP= 79-34=45

The cerebral perfusion pressure (CPP) is the pressure needed to ensure blood flow to the brain. Normal CPP is 60 to 100 mmHg. Calculate the CPP of a patient whose blood pressure (BP) is 106/52 mmHg and ICP is 14mmHg.

56mmHG Rationale: Mean arterial pressure (MAP)= diastolic blood pressure (DBP) + 1/3 (systolic blood pressure [SBP]-DBP = 52+18=70 Cerebral perfusion pressure (CPP)= MAP-ICP= 70-14=56

A patient has atrial fibrillation and develops an acute arterial occlusion at the iliac artery bifurcation. What are the six Ps of acute arterial occlusion the nurse may assess in this patient that require immediate notification of the HCP?

Pain, Pallor, pulselessness, parasthesia, paralysis, poikilothermia. The HCP requires immediate notification to begin immediate intervention to prevent tissue necrosis and gangrene

A patient in the sexually transmitted infection (STI) clinic tells the nurse that she is concerned she may have been exposed to gonorrhea. To determine whether the patient has gonorrhea, the nurse will plan to A. interview the patient about symptoms of gonorrhea B. take a sample of cervical discharge for Gram staining C. draw a blood specimen or rapid plasma reagin (RPR) testing D. obtain secretions for a nucleic acid amplification test (NAAT)

D. obtain secretions for a nucleic acid amplification test (NAAT) Rationale: NAAT has a high sensitivity (similar to a culture) for gonorrhea. Because women have few symptoms of gonorrhea, asking the patient about symptoms may not be helpful in making a diagnosis. Smears and Gram staining are not useful because the female genitourinary tract has many normal flora that resemble Neisseria gonorrhoeae. RPR testing is used to detect syphilis

In a patient with a disease that affects the myelin sheath of nerves, such as multiple sclerosis, the glial cells affected are the A. microglia B. astrocytes C. ependymal cells D. oligodendrocytes

D. oligodendrocytes Rationale: Glial cell types include oligodendrocytes, astrocytes, ependymal cells, and microglia, and each has specific functions. Oligodendrocytes are specialized cells that produce the myelin sheath of nerve fibers in the central nervous system (CNS), and they are located primarily in the white matter of the CNS

Which nursing diagnosis is of highest priority for a patient with Parkinson's disease who is unable to move the facial muscles? A. Activity intolerance B. Self-care deficit: toileting C. Ineffective self-health management D. Imbalanced nutrition: less than body requirements

D. Imbalanced nutrition: less than body requirements Rationale: The data about the patient indicate that poor nutrition will be a concern because of decreased swallowing. The other diagnoses may also be appropriate for a patient with Parkinson's disease, but the data do not indicate that they are current problems for this patient.

To prevent or decrease age-related changes that occur after menopause in a patient who chooses not to take hormone therapy, the most important self-care measure to teach is a. maintaining usual sexual activity. b. increasing the intake of dairy products. c. performing regular aerobic, weight-bearing exercise. d. taking vitamin E and B-complex vitamin supplements.

c. performing regular aerobic, weight-bearing exercise. Rationale: A regular program (three to four times per week) of moderate aerobic and weight bearing exercises can slow the process of bone loss and a tendency toward weight gain. Exercise is important for menopausal women in modifying risk factors for coronary artery disease, including stress, obesity, physical inactivity, and hypertension

Rest pain is a manifestation of PAD that occurs due to a chronic a. vasospasm of small cutaneous arteries in the feet. b. increase in retrograde venous blood flow in the legs. c. decrease in arterial blood flow to the nerves of the feet. d. decrease in arterial blood flow to the leg muscles during exercise.

c. decrease in arterial blood flow to the nerves of the feet. Rationale: Rest pain most often occurs in the forefoot or toes and is aggravated by limb elevation. Rest pain occurs when blood flow is insufficient to meet basic metabolic requirements of the distal tissues. Rest pain occurs more often at night because cardiac output tends to drop during sleep an the limbs are at the level of the heart. Patients often try to achieve partial pain relief by dangling the leg over the side of the bed or sleeping in a chair to allow gravity to maximize blood flow

The following male patients recently arrived in the emergency department. Which one should the nurse assess first? A. A 19-year-old who is complaining of severe scrotal pain B. A 60-year-old with a nontender ulceration of the glans penis C. A 64-year-old who has dysuria after brachytherapy for prostate cancer D. A 22-year-old who has purulent urethral drainage and severe back pain

A. A 19-year-old who is complaining of severe scrotal pain Rationale: The patient's age and symptoms suggest possible testicular torsion, which will require rapid treatment to prevent testicular necrosis. The other patients also require assessment by the nurse, but their history and symptoms indicate nonemergent problems (acute prostatitis, cancer of the penis, and radiation associated urinary tract irritation)

A woman who has been postmenopausal for 10 years calls the clinic because of vaginal bleeding. The nurse schedules a visit for the patient and informs her to expect to have which diagnostic procedure? A. An endometrial biopsy B. Abdominal radiography C. A laser treatment to the cervix D. Only a routine pelvic examination and Pap test

A. An endometrial biopsy Rationale: Postmenopausal vaginal bleeding is an early sign of endometrial cancer. When it occurs, a sample of endometrial tissue must be taken to exclude cancer. An endometrial biopsy can be done as an office procedure and is indicated in this case. Abdominal x-rays and Pap tests are not reliable tests for endometrial cancer. Laser treatment of the cervix is indicated only for cervical dysplasia

The nurse will plan to teach the female patient with genital warts about the A. importance of regular Pap tests B. increased risk for endometrial cancer C. appropriate use of oral contraceptives D. symptoms of pelvic inflammatory disease (PID)

A. importance of regular Pap tests Rationale: Genital warts are caused by human papillomavirus (HPV) and increase the risk for cervical cancer. There is no indication that the patient needs teaching about PID, oral contraceptives, or endometrial cancer

The nurse admitting a patient who has a right frontal lobe tumor would expect the patient may have A. expressive aphasia. B. impaired judgment. C. right-sided weakness. D. difficulty swallowing.

B. impaired judgment. Rationale: The frontal lobe controls intellectual activities such as judgement. Speech is controlled in the parietal lobe. Weakness and hemiplegia occur on the contralateral side from the tumor. Swallowing is controlled by the brainstem

Which topics will the nurse include when preparing to teach a patient with recurrent genital herpes simplex (select all that apply)? A. Infected areas should be kept moist to speed healing. B. Sitz baths may be used to relieve discomfort caused by the lesions. C. Genital herpes can be cured by consistent use of antiviral medications. D. Recurrent genital herpes episodes usually are shorter than the first episode. E. The virus can infect sexual partners even when you do not have symptoms of infection.

B.D.E. Sitz baths may be used to relieve discomfort caused by the lesions; Recurrent genital herpes episodes usually are shorter than the first episode; The virus can infect sexual partners even when you do not have symptoms of infection. Rationale: Patients are taught that shedding of the virus and infection of sexual partners can occur even in asymptomatic periods, that recurrent episodes resolve more quickly, and that sitz baths can be used to relieve pain caused by the lesions. Antiviral medications decrease the number of outbreaks but do not cure herpes simplex infections. Infected areas may be kept dry if this decreases pain and itching

The newly admitted patient has moderate AD. What does the nurse know this patient will need help with? A. Eating B. Walking C. Dressing D. Self care activities

C. Dressing Rationale: In the moderate stage of AD, the patient may need help with getting dressed. In the severe stage, patients will be unable to dress or feed themselves and are usually incontinent

The nurse's initial action for a patient with moderate dementia who develops increased restlessness and agitation should be to A. reorient the patient to time, place, and person B. administer a PRN dose of lorazepam (Ativan) C. assess for factors that might be causing discomfort D. assign unlicensed assistive personnel (UAP) to stay in the patient's room

C. assess for factors that might be causing discomfort Rationale: Increased motor activity in the patient with dementia is frequently the patient's only way of responding to factors such as pain, so the nurse's initial action should be to assess the patient for any precipitating factors. Administration of sedative drugs may be indicated, but this should not be done until assessment for precipitating factors has been completed and any of these factors have been addressed. Reorientation is unlikely to be helpful for the patient with moderate dementia. Assigning UAP to stay with the patient may also be necessary, but any physical changes that may be causing the agitation should be addressed first

A 53 year old patient is scheduled for an annual physical examination. The nurse will plan to teach the patient about the purpose of A. urinalysis collection B. uroflowmetry studies C. prostate specific antigen (PSA) testing D. transrectal ultrasound scanning (TRUS)

C. prostate specific antigen (PSA) testing Rationale: An annual digital rectal exam (DRE) and PSA are usually recommended starting at age 50 years for men who have an average risk for prostate cancer. Urinalysis and uroflowmetry studies are done if patients have symptoms of urinary tract infection of changes in urinary stream. TRUS may be ordered if the DRE or PSA results are abnormal

A patient with suspected meningitis is scheduled for a lumbar puncture. Before the procedure, the nurse will plan to A. enforce NPO status for 4 hours B. Transfer the patient to radiology C. administer a sedative medication D. help the patient to a lateral position

D. help the patient to a lateral position Rationale: For a lumbar puncture, the patient lies in the lateral recumbent position. The procedure does not usually require a sedative, is done in the patient room, and has no risk for aspiration

A patient at the clinic says "I always walk after dinner, but lately my leg cramps and hurts after just a few minutes of starting. The pain goes away after I stop walking, though." The nurse should A. look for the presence of tortuous veins bilaterally on the legs B. ask about any skin color changes that occur in response to cold C. assess for unilateral swelling, redness, and tenderness of either leg D. palpate for the presence of dorsalis pedis and posterior tibial pulses

D. palpate for the presence of dorsalis pedis and posterior tibial pulses Rationale: The nurse should assess for other clinical manifestations of peripheral artery disease in a patient who describes intermittent claudication. Changes in skin color that occur in response to cold are consistent with Raynaud's phenomenon. Tortuous veins on the legs suggest venous insufficiency. Unilateral leg swelling, redness, and tenderness indicate venous thromboembolism

The nurse is caring for a patient with critical limb ischemia who has just arrived on the nursing unit after having percutaneous transluminal balloon angioplasty. Which action should the nurse perform first? A. Obtain vital signs B. Teach wound care C. Assess pedal pulses D. Check the wound site

A. Obtain vital signs Rationale: Bleeding is a possible complication after catheterization of the femoral artery, so the nurse's first action should be to assess for changes in vital signs that might indicate hemorrhage. The other actions are also appropriate but can be done after determining that bleeding is not occurring.

The first nursing intervention for the patient who has been sexually assaulted is to A. treat urgent medical problems B. contact support person for the patient C. provide supplies for the patient to cleanse self D. document bruises and lacerations of the perineum and the cervix

A. treat urgent medical problems Rationale: In the care of a victim of sexual assault, shock and other urgent medical problems (e.g. head injury, hemorrhage, wounds, fractures) are treated first

Which components are able to change to adapt to small increases in intracranial pressure (ICP) (select all that apply)? A. Blood B. Skull bone C. Brain tissue D. Scalp tissue E. Cerebrospinal fluid (CSF)

A.C.E. Blood; brain tissue; cerebrospinal fluid (CSF) Rationale: Blood adapts with increased venous outflow, decreased cerebral blood flow (CBF), and collapse of veins and dural sinuses. Brain tissue adapts with distention of the dura, slight compression of tissue, or herniation. Cerebrospinal fluid (CSF) adapts with increased absorption, decreased production, and displacement into the spinal canal. Skull bone and scalp tissue do not adapt to changes in intracranial pressure (ICP)

The patient calls the office and says that she thinks she has a "yeast infection". What signs or symptoms should the nurse expect in this patient? (select all that apply) A. Intense itching and dysuria B. Hemorrhagic cervix and vagina C. Pruritic, frothy greenish or gray discharge D. Thick, white, cottage cheese like discharge E. Mucopurulent discharge and postcoital spotting

A.D. Intense itching and dysuria; thick, white, cottage cheese like discharge Rationale: "Yeast infection" or vulvovaginal candidiasis has intense itching and dysuria from urine coming in contact with fissures or irritated areas in the vulva. The discharge is thick, white, and curdlike. Hemorrhagic cervix and vagina occur with trichomoniasis and produce a pruritic, frothy greenish or gray discharge. Mucopurulent discharge and postcoital spotting from cervical inflammation are seen with cervicitis

A patient with a small AAA is not a good surgical candidate. What should the nurse teach the patient that is one of the best ways to prevent expansion of the lesion? A. Avoid strenuous physical exertion B. Control hypertension with prescribed therapy C. Comply with prescribed anticoagulant therapy D. Maintain a low calcium diet to prevent calcification of the vessel

B. Control hypertension with prescribed therapy Rationale: Increased systolic blood pressure (SBP) continually puts pressure on the diseased area of the artery, promoting its expansion. Small aneurysms can be treated by decreasing blood pressure (BP), modifying atherosclerosis risk factors, and monitoring the size of the aneurysm. Anticoagulants are used during surgical treatment of aneurysms, but physical activity is not known to increase their size. Calcium intake is not related to calcification in arteries

Vasogenic cerebral edema increases intracranial pressure by A. shifting fluid in the gray matter B. altering the endothelial lining of cerebral capillaries C. leaking molecules from the intracellular fluid to the capillaries D. altering the osmotic gradient flow into the intravascular component

B. altering the endothelial lining of cerebral capillaries Rationale: Vasogenic cerebral edema occurs mainly in white matter. It is caused by changes in the endothelial lining of cerebral capillaries

Which finding from the nurse's physical assessment of a 42 year old male patient should be reported to the health care provider? A. One testis hangs lower then the other B. Genital hair distribution is diamond shaped C. Clear discharge is present at the penile meatus D. Inguinal lymph nodes are nonpalpable bilaterally

C. Clear discharge is present at the penile meatus Rationale: Clear penile discharge may be indicative of a sexually transmitted infection (STI). The other findings are normal and do not need to be reported

The first priority of interprofessional care of a patient with a suspected acute aortic dissection is to A. reduce anxiety B. monitor for chest pain C. control blood pressure D. increase myocardial contractility

C. control blood pressure Rationale: The initial goals of therapy for acute aortic dissection without complications are blood pressure (BP) control and pain management. BP control reduces stress on the aortic wall by reducing systolic BP amd myocardial contractility

A patient is diagnosed with primary syphilis during her eighth week of pregnancy. The nurse will plan to teach the patient about the A. likelihood of a stillbirth B. plans for cesarean section C. intramuscular injection of penicillin D. antibiotic eye drops for the newborn

C. intramuscular injection of penicillin Rationale: A single injection of penicillin is recommended to treat primary syphilis. This will treat the mother and prevent transmission of the disease to the fetus. Instillation of erythromycin into the eyes of the newborn is used to prevent gonorrheal eye infections. C-section is used to prevent the transmission of herpes to the newborn. Although stillbirth can occur if the fetus is infected with syphilis, treatment before the 10th week of gestation will eliminate in utero transmission to the fetus.

A 68 year old male patient tells the nurse that he is worried because he does not respond to sexual stimulation the same way he did when he was younger. Which is the nurse's best response to the patient's concern? A. "Interest in sex frequently decreases as men get older." B. "Many men need additional sexual stimulation with aging." C. "Erectile dysfunction is a common problem with older men." D. "Tell me more about how your sexual response has changed."

D. "Tell me more about how your sexual response has changed." Rationale: The initial response by the nurse should be further assessment of the problem. The other statements by the nurse are accurate but may not respond to the patient's concerns.

Which medication taken by a patient with restless legs syndrome should the nurse discuss with the patient? A. Ibuprofen B. Multivitamin C. Acetaminophen D. Diphenhydramine

D. Diphenhydramine Rationale: Antihistamines can aggravate restless legs syndrome. The other medications will not contribute to restless leg syndromes

During evaluation and treatment of gonorrhea in a young man at the health clinic, about what is it most important for the nurse to question the patient? A. A prior to history of STIs B. When the symptoms begin C. The date of his last sexual activity D. The names of his recent sexual partners

D. The names of his recent sexual partners Rationale: All sexual contacts of patients with gonorrhea must be notified, evaluated, and treated for STIs to prevent reinfection and further transmission. The other information may be helpful in diagnosis and treatment, but the nurse must try to identify the patient's sexual partners

A patient who has a large cystocele was admitted 10 hours ago but has not yet voided. If the patient reports no urge to void, which action should the nurse take first? A. Insert a straight catheter per the PRN order B. Encourage the patient to increase oral fluids C. Notify the health care provider of the inability to void D. Use an ultrasound scanner to check for urinary retention

D. Use an ultrasound scanner to check for urinary retention Rationale: Because urinary retention is common with a large cystocele, the nurse's first action should be to use an ultrasound bladder scanner to check for the presence of urine in the bladder. The other actions may be appropriate, depending on the findings with the bladder scanner

Which patient statement to the nurse is most consistent with the diagnosis of venous insufficiency? A. "I can't get my shoes on at the end of they day." B. "I can't ever seem to get my feet warm enough." C. "I have burning leg pains after I walk two blocks." D. "I wake up during the night because my legs hurt."

A. "I can't get my shoes on at the end of they day." Rationale: Because the edema associated with venous insufficiency increases when the patient has been standing, shoes will feel tighter at the end of the day. The other patient statements are characteristic of peripheral artery disease.

The patient is admitted to the emergency department having difficulty with respiratory, vasomotor, and cardiac function. Which portion of the brain is affected to cause these manifestations? A. Medulla B. Cerebellum C. Parietal lobe D. Wernicke's area

A. Medulla Rationale: The medulla contains the vital centers concerned with respiratory, vasomotor, and cardiac function. The cerebellum maintains trunk stability and equilibrium but is not related to respiratory or cardiac function. The parietal lobe interprets spatial information and controls the sensory cortex. Wernicke's area is responsible for language comprehension

The nurse is obtaining a subjective data assessment from a woman reported as a sexual contact of a man with chlamydial infection. The nurse understands that symptoms of chlamydial infection in women A. are frequently absent B. are similar to those of genital herpes C. include a macular palmar rash in the later stages D. may involve chancres inside the vagina that are not visible

A. are frequently absent Rationale: Chlamydial infections are known as the silent disease because symptoms are absent or minor in most infected women and in many men

The RN in charge at a long term care facility could delegate which activities to unlicensed assistive personnel (UAP) (Select all that apply)? A. Assist the patient with eating B. Provide personal hygiene and skin care C. Check the environment for safety hazards D. Assist the patient to the bathroom at regular intervals E. Monitor for skin breakdown and swallowing difficulties

A.B.D. Assist the patient with eating; provide personal hygiene and skin care; assist the patient to the bathroom at regular intervals Rationale: All caregivers are responsible for the patient's safety. Basic care activities, such as those associated with personal hygiene and activities of daily living (ADLs) can be delegated to unlicensed assistive personnel (UAP). The RN will perform ongoing assessments and develop and revise the plan of care as needed. The RN will assess the patient's safety risk factors, provide education, and make referrals. The licensed practical nurse (LPN) could check the patient's environment for potential safety hazards.

A patient with a head injury opens his eyes to verbal stimulation, curses when stimulated, and does not respond to a verbal command to move but attempts to push away a painful stimulus. The nurse records the patient's Glasgow Coma Scale score as A. 9 B. 11 C. 13 D. 15

B. 11 Rationale: The patient has scores of 3 for eye opening, 3 for verbal response, and 5 for best motor response

Which nursing action has the highest priority for a patient who was admitted 16 hours earlier with a C5 spinal cord injury? A. Cardiac monitoring for bradycardia B. Assessment of respiratory rate and effort C. Administration of low molecular weight heparin D. application of pneumatic compression devices to legs

B. Assessment of respiratory rate and effort Rationale: Edema around the area of injury may lead to damage above the C4 level, so the highest priority is assessment fo the patient's respiratory function. The other actions are also appropriate for preventing deterioration or complications but are not as important as assessment for respiratory effort

A 25 year old female patient is at the clinic and says that she has white vaginal drainage and itching. Which etiology would be suspected? A. Cancer B. Candidiasis C. Trichomonas vaginalis D. Bacterial vaginosis

B. Candidiasis Rationale: Candidiasis is a white, thick, or curdy discharge and causes itching and inflammation. Cancer would be more likely to produce a bloody discharge. Trichomonas vaginalis produces a malodorous frothy green or yellow discharge. Bacterial vaginosis infection produces copious amounts of thin gray or white drainage with a fishy odor

Which problem can the nurse expect for patient who has a positive Romberg test result? A. Pain B. Falls C. Aphasia D. Confusion

B. Falls Rationale: A positive Romberg test result indicates that the patient has difficulty maintaining balance when standing with the eyes closed. The Romberg does not test for orientation, thermoregulation, or discomfort

Following a dilation and curettage (D&C), for what complication is it most important for the nurse to assess the patient? A. Infection B. Hemorrhage C. Urinary retention D. Perforation of the bladder

B. Hemorrhage Rationale: The risk for bleeding is increased following a dilation and curettage (D&C) because the endometrial lining is scraped and injury to the uterus can occur. The nurse should closely assess the amount of bleeding with frequent pad checks for the first 24 hours. Infection following D&C is uncommon, and the urinary system is not affected. Infection could occur after a biopsy. Urinary retention occurs with BPH

Which nursing action should be included in the plan of care after endovascular repair of an abdominal aortic aneurysm? A. Record hourly chest tube drainage B. Monitor fluid intake and urine output C. Assess the abdominal incision for redness D. Teach the patient to plan for a long recovery period

B. Monitor fluid intake and urine output Rationale: Because renal artery occlusion can occur after endovascular repair, the nurse should monitor parameters of renal function such as intake and output. Chest tubes will not be needed for endovascular surgery, the recovery period will be short, and there will not be an abdominal wound

A patient with myasthenia gravis is admitted to the hospital with respiratory insufficiency and severe weakness. What confirms a diagnosis of a myasthenia gravis? A. History and physical examination reveal weakness B. Serum acetylcholine receptor antibodies are present C. The patient's respiration is impaired because of muscle weakness D. EMG reveals an increased response with repeated stimulation of muscles

B. Serum acetylcholine receptor antibodies are present Rationale: Serum acetylcholine receptor antibodies will confirm a diagnosis of myasthenia gravis (MG). The history and physical revealing weakness is part of the diagnosis, but not the confirmation. Respiration impairment is a sign of MG, but not a confirmation of the diagnosis. The EMG will show muscle fatigue with a decreased response

After evacuation of an epidural hematoma, a patient's intracranial pressure (ICP) is being monitored with an intraventricular catheter. Which information obtained by the nurse is most important to communicate to the health care provider? A. Pulse 102 beats/min B. Temperature 101.6° F C. Intracranial pressure 15 mm Hg D.Mean arterial pressure 90 mm Hg

B. Temperature 101.6° F Rationale: Infection is a serious consideration with ICP monitoring, especially with intraventricular catheters. The temperature indicates the need for antibiotics or removal of the monitor. The ICP, arterial pressure, and apical pulse only require ongoing monitoring at this time

The individual with the lowest risk for sexual transmitted pelvic inflammatory disease is a woman who uses A. oral contraceptives B. barrier methods of contraception C. an intrauterine device for contraception D. Norplant implant or injectable Depo-Provera for contraception

B. barrier methods of contraception Rationale: The condom is considered to be the best form of protection against sexually transmitted infections (STIs)

Propranolol (Inderal), a B-adrenergic blocker that inhibits sympathetic nervous system activity, is prescribed for a patient who has extreme anxiety about public speaking. The nurse monitors the patient for A. dry mouth B. bradycardia C. constipation D. urinary retention

B. bradycardia Rationale: Inhibition fo the fight or flight response leads to a decreased heart rate. Dry mouth, constipation, and urinary retention are associated with peripheral nervous system blockade

Assisting the family to understand what is happening to the patient is an especially important role of the nurse when the patient has a tumor in which part of the brain A. ventricles B. frontal lobe C. parietal lobe D occipital lobe

B. frontal lobe Rationale: Frontal lobe tumors often lead to loss of emotional control, confusion, memory loss, disorientation, seizures, and personality and judgement changes that are very disturbing and frightening to the family. Physical symptoms, such as blindness, speech disturbances, or disturbances in sensation and perception that occur with other tumors, are more likely to be understood and accepted by the family

The nurse will plan to teach the patient scheduled for photovaporization of the prostate (PVP) A. that urine will appear bloody for several days B. how to care for an indwelling urinary catheter C. that symptom management improvement takes 2-3 weeks D. about complications associated with urethral stenting

B. how to care for an indwelling urinary catheter Rationale: The patient will have an indwelling catheter for 24 to 48 hours and will need teaching about catheter care. There is minimal bleeding with this procedure. Symptom improvement is almost immediate after PVP. Stent placement is not included in this procedure

The nurse is assessing the muscle strength of an older adult patient. The nurse knows the findings cannot be compared with those of a younger adult because A. nutritional status is better in young adults B. muscle bulk and strength decrease in older adults C. muscle strength should be the same for all adults. D. most young adults exercise more than older adults

B. muscle bulk and strength decrease in older adults Rationale: Changes associated with aging include decreases in muscle strength and agility in relation to decreased muscle bulk

An unconscious patient with increased ICP is on ventilatory support. Which arterial blood gas (ABG) measurement should prompt the nurse to notify the HCP? A. pH of 7.43 B. SaO2 of 94% C. PaO2 of 70mmHg D. PaCO2 of 35mmHg

C. PaO2 of 70mmHg Rationale: A PaO2 of 70mmHg reflects hypoxemia that may lead to further decreased cerebral perfusion. The goal is to keep PaO2 at greater than or equal to 100mmHg. The pH and SaO2 are within normal range and a PaCO2 of 35mmHg reflects a normal value

When assessing an aging adult woman, what does the nurse note as a normal finding? A. Rectocele B. Larger breasts C. Vaginal dryness D. Severe osteoporosis

C. Vaginal dryness Rationale: Vaginal dryness occurs with decreased estrogen and increased androgens circulating in the aging female. This also leads to breast and genital atrophy, reduction in bone mass, and increased atherosclerosis. A rectocele may occur and cause sexual or fecal elimination problems for the patient that will need treatment, but this is not a normal finding. Severe osteoporosis is not a normal change of aging.

The health care provider has prescribed IV norepinephrine (Levophed) for a patient in the ED with SCI. The nurse determines that the drug is having the desired effect when what is observed in patient assessment? A. Heart rate of 68bpm B. Respiratory rate of 24 C. Temperature of 96.8 D. Blood pressure of 106/82

D. Blood pressure of 106/82 Rationale: Norepinephrine is a vasopressor that is used to maintain blood pressure during states of hypotension that may occur during neurogenic shock associated with SCI. Atropine would be used to treat bradycardia. The temperature reflects some degree of poikilothermism, but this is not treated with medications

The health care provider prescribes the following interventions for a patient with acute prostatitis caused by Escherichia coli. Which intervention should the nurse question? A. Give trimethoprim/sulfamethoxazole 1 tablet daily for 28 days B. Administer ibuprofen 400mg every 8 hours as needed for pain C. Instruct the patient to avoid sexual intercourse until treatment is complete D. Catheterize the patient as needed if symptoms of urinary retention develop

D. Catheterize the patient as needed if symptoms of urinary retention develop Rationale: Although acute urinary retention may occur, insertion of a catheter through an inflamed urethra is contraindicated, and the nurse will anticipate that the health care provider will need to insert a suprapubic catheter. The other actions are appropriate

Which assessment information collected by the nurse may present a contraindication to a testosterone replacement therapy (TRT)? A. The patient has noticed a decrease in energy level for a few years B. The patient's symptoms have increased steadily over the past few years C. The patient has been using sildenafil (Viagra) several times every week D. The patient has had a gradual decrease in the force of his urinary stream

D. The patient has had a gradual decrease in the force of his urinary stream Rationale: The decrease in urinary stream may indicate benign prostatic hyperplasia (BPH) or prostate cancer, which are contraindications to the use of testosterone replacement therapy (TRT). The other patient data indicate that TRT may be a helpful therapy for the patient

What occurs during treatment of the patient with an intrauterine radioactive implant? A. All care should be provided by the same nurse B. The patient may ambulate in the room as desired C. There can be an unlimited number and duration of visitors D. The patient is restricted to bed rest with turning from side to side

D. The patient is restricted to bed rest with turning from side to side Rationale: To prevent displacement of the intrauterine implant, the patient is maintained on absolute bed rest with turning from side to side. Bowel elimination is discouraged during the treatment by cleaning the colon before implantation and urinary elimination is maintained by an indwelling catheter. Because the patient is radioactive, the principles of ALARA (as low as reasonably achievable) are used so that caregivers limit time and distance and use shielding for protection

What is the most important method of diagnosing functional headaches? A. CT scan B. Electromyography (EMG) C. Cerebral blood flow studies D. Thorough history of the headache

D. Thorough history of the headache Rationale: The primary way to diagnose and differentiate between headaches is with a careful history of the headaches, requiring assessment of specific details related to the headache. Electromyography (EMG) may reveal contraction of the neck, scalp, or facial muscles in tension type headaches, but this is not seen in all patients. CT scans and cerebral angiography are used to rule out organic causes of the headaches

A patient who has been diagnosed with stage 2 prostatic cancer chooses the option of active surveillance. The nurse will plan to A. vaccinate the patient with sipuleucel-T (Provenge) B. provide the patient with information about cryotherapy C. teach the patient about placement of intraurethral stents D. schedule the patient for annual prostate specific antigen testing

D. schedule the patient for annual prostate specific antigen testing Rationale: Patient's sho opt for active surveillance need to have annual digital rectal examinations and prostate specific antigen testing. Vaccination with sipuleucel-T, cryotherapy, and stent placement are options for patients who choose to have active treatment for prostate cancer.

Which characteristics describe transurethral resection of the prostate (TURP) (select all that apply)? A. Best used for a very large prostate gland B. Inappropriate for men with rectal problems C. Involves an external incision prostatectomy D. Uses transurethral incision into the prostate E. Most common surgical procedure to treat BPH F. Resectoscopic excision and cauterization or prostate tissue

E.F. Most common surgical procedure to treat BPH; Resectoscopic excision and cauterization or prostate tissue Rationale: The transurethral resection of the prostate (TURP) is the most common surgical procedure to treat BPH and uses a resectoscopic excision and cauterization of prostate tissue. Photovaporization of the prostate (PVP) or a simple open prostatectomy may be used for a very large prostate and has an external incision. TUMT is not approved for men with rectal problems. Transurethral incision into the prostate to expand the urethra for a small to moderate sized prostate is done with a tranurethral incision of the prostate (TURP).

Several patients call the urology clinic requesting appointments with the health care provider as soon as possible. Which patient will the nurse schedule to be seen first? A. 22-year-old who has noticed a firm, nontender lump on his scrotum B. 35-year-old who is concerned that his scrotum "feels like a bag of worms" C. 40-year-old who has pelvic pain while being treated for chronic prostatitis D. 70-year-old who is reporting frequent urinary dribbling after a prostatectomy

A. 22-year-old who has noticed a firm, nontender lump on his scrotum Rationale: The patient's age and symptoms suggest possible testicular cancer. Some forms of testicular cancer can be very aggressive, so the patient should be evaluated by the health care provider as soon as possible. Varicoceles do require treatment but not emergently. Ongoing pelvic pain is common with chronic prostatitis. Urinary dribbling is a common problem after prostatectomy

Which patient will the nurse plan on teaching about the Gardasil vaccine? A. A 24 year old female patient who has not been sexually active B. A 34 year old female patient who has multiple sexual partners C. A 24 year old female patient who is pregnant for the first time D. A 34 year old female patient who is in a monogamous relationship

A. A 24 year old female patient who has not been sexually active Rationale: Gardasil is recommended for female patients ages 9 through 26 years, preferably those who have never been sexually active. It is not recommended for women during pregnancy or for older women

Which patient is most appropriate for the intensive care unit (ICU) charge nurse to assign to a registered nurse (RN) who has floated from the medical unit? A. A 45 year old patient receiving antibiotics for meningococcal meningitis B. A 35 year old patient with intracranial pressure (ICP) monitoring after a head injury C. A 25 year old patient admitted with a skull fracture and craniotomy the previous day D. A 55 year old patient who has increased intracranial pressure (ICP) and is receiving hyperventilation therapy

A. A 45 year old patient receiving antibiotics for meningococcal meningitis Rationale: An RN who works on a medical unit will be familiar with administration of IV antibiotics and with meningitis. The patient recovering from a craniotomy, the patient with an ICP monitor, and the patient on a ventilator should be assigned to an RN familiar with the care of critically ill patients

What is the effect of finasteride (Proscar) in the treatment of BPH? A. A reduction in the size of the prostate gland B. Relaxation of the smooth muscle of the urethra C. Increased bladder tone that promotes bladder emptying D. Relaxation of the bladder detrusor muscle promoting urine flow

A. A reduction in the size of the prostate gland Rationale: Finasteride results in the suppression of dihydroxytestosterone (DHT) formation, which reduces the size of the prostate gland. Drugs affecting bladder tone are not indicated. A-Adrenergic receptor blockers are used to cause smooth muscle relaxation in the prostate, which improves urine flow

A male patient who has possible cerebral edema has a serum sodium level of 116 mEq/L (116 mmol/L) and a decreasing level of consciousness (LOC). He is now complaining of a headache. Which prescribed interventions should the nurse implement first? A. Administer IV 5% hypertonic saline. B. Draw blood for arterial blood gases (ABGs). C. Send patient for computed tomography (CT). D. Administer acetaminophen (Tylenol) 650 mg orally.

A. Administer IV 5% hypertonic saline. Rationale: The patient's low sodium indicates that hyponatremia may be causing the cerebral edema. The nurse's first action should be to correct the low sodium level. Acetaminophen (Tylenol) will have minimal effect on the headache because it is caused by cerebral edema and increased intracranial pressure (ICP). Drawing ABGs and obtaining a CT scan may provide some useful information, but the low sodium level may lead to seizures unless it is addressed quickly

How is urinary function maintained during the acute phase of spinal cord injury? A. An indwelling catheter B. Intermittent catheterization C. Insertion of a suprapubic catheter C. Use of incontinent pads to protect the skin

A. An indwelling catheter Rationale: During the acute phase of SCI, the bladder is atonic, causing urinary retention with the risk for reflux into the kidney or rupture of the bladder. An indwelling catheter is used to keep the bladder empty and monitor urinary output. Intermittent catheterization or other urinary drainage methods may be used in long term bladder management. Use of incontinent pads is inappropriate, because they do not help the bladder to empty

A patient has ICP monitoring with an intraventricular catheter. What is a priority intervention for the patient? A. Aseptic technique to prevent infection B. Constant monitoring of ICP waveforms C. Removal of CSF to maintain normal ICP D. Sampling CSF to determine abnormalities

A. Aseptic technique to prevent infection Rationale: An intraventricular catheter is a fluid coupled system that can provide direct access for microorganisms to enter the ventricles of the brain and aseptic technique is a very high nursing priority to decrease the risk for infection. Constant monitoring of ICP waveforms is not usually necessary and removal of CSF for sampling or to maintain normal ICP is done only when specifically ordered

A 28 year old patient reports anxiety, headaches with dizziness, and abdominal bloating occurring before her menstrual periods. Which action is best for the nurse to take at this time? A. Ask the patient to keep track of her symptoms in a diary for 3 months B. Suggest that the patient try aerobic exercise to decrease her symptoms C. Teach the patient about appropriate lifestyle changes to reduce premenstrual syndrome (PMS) symptoms D. Advise the patient to use nonsteroidal antiinflammatory drugs (NSAIDs) such as ibuprofen to control symptoms

A. Ask the patient to keep track of her symptoms in a diary for 3 months Rationale: The patient's symptoms indicate possible PMS, but they also may be associated with other diagnoses. Having the patient keep a symptom diary for 2 to 3 months will help in confirming a diagnosis of PMS. The nurse should not implement interventions for PMS until a diagnosis is made

Which action will the nurse include in the plan of care for a patient who is experiencing pain from trigeminal neuralgia? A. Assess fluid and dietary intake B. Apply ice packs for 20 minutes C. teach facial relaxation techniques D. spend time talking with the patient

A. Assess fluid and dietary intake Rationale: The patient with an acute episode of trigeminal neuralgia may be unwilling to eat or drink, so assessment of nutritional and hydration status is important. Because stimulation by touch is the precipitating factor for pain, relaxation of the facial muscles will not improve symptoms. Application of ice is likely to precipitate pain. The patient will not want to engage in conversation, which may precipitate attacks

A patient brought into the emergency department reports being sexually assaulted. The patient is confused about where she is and she has a laceration above the right eye. Which action should the nurse take first? A. Assess the patient's neurologic status B. Assist the patient to remove her clothing C. Ask the patient to describe what occurred during the assault D. Ask the sexual assault nurse examiner (SANE) to assess the patient

A. Assess the patient's neurologic status Rationale: The first priority is to treat urgent medical problems associated with the sexual assault. The patient's head injury may be associated with the head trauma such as a skull fracture or subdural hematoma. Therefore her neurologic status should be assessed first. The other nursing actions are also appropriate, but they are not as high in priority as assessment and treatment for acute physiologic injury

Admission vital signs for a brain-injured patient are blood pressure 128/68, pulse 110, and respirations 26. Which set of vital signs, if taken 1 hour after admission, will be of most concern to the nurse? A. Blood pressure 154/68, pulse 56, respirations 12 B. Blood pressure 134/72, pulse 90, respirations 32 C. Blood pressure 148/78, pulse 112, respirations 28 D. Blood pressure 110/70, pulse 120, respirations 30

A. Blood pressure 154/68, pulse 56, respirations 12 Rationale: Systolic hypertension with widening pulse pressure, bradycardia, and respiratory changes represent Cushing's triad. These findings indicate that the intracranial pressure (ICP) has increased and the brain herniation may be imminent unless immediate action is taken to reduce ICP. The other vital signs may indicate the need for changes in treatment, but they are not indicative of an immediately life-threatening process.

Which action will the nurse include in the plan of care for a patient who has a cauda equina spinal cord injury? A. Catheterize patient every 3 to 4 hours B. Assist patient to ambulate 4 times daily C. administer medications to reduce bladder spasm D. stabilize the neck when repositioning the patient

A. Catheterize patient every 3 to 4 hours Rationale: Patients with cauda equina syndrome have areflexive bladder, and intermittent catheterization will be used for emptying the bladder. Because the bladder is flacid, antispasmodic medications will not be used. The legs are flaccid with cauda equina syndrome, and the patient will be unable to ambulate. The head and neck will not need to be stabilized after a cauda equina injury, which affects the lumbar and sacral nerve roots

What is different when a lesion occurs in a lower motor neuron compared to in an upper motor neuron? A. Causes hyporeflexia and flaccidity B. Affects motor control of the lower body C. Arises in structures above the spinal cord D. Interferes with reflex arcs in the spinal cord

A. Causes hyporeflexia and flaccidity Rationale: The cell bodies of lower motor neurons that send impulses to skeletal muscles in the arms, trunk, and legs are located in the anterior horn of the spinal cord and lesions generally cause weakness or paralysis, decreased muscle tone, hyporeflexia, and flaccidity. Upper motor neurons include the brainstem and cerebral cortex motor neurons that influence skeletal muscle movement. Lesions at this point cause weakness and paralysis with hyperreflexia and spasticity

The nurse is developing a discharge teaching plan for a patient diagnosed with thromboangiitis obliterans (Buerger's disease). Which expected outcome has the highest priority for this patient? A. Cessation of all tobacco use B. Control of all serum lipid levels C. Maintenance of appropriate weight D. Demonstration of meticulous foot care

A. Cessation of all tobacco use Rationale: Absolute cessation of nicotine use is needed to reduce the risk for amputation in patients with Buerger's disease. Other therapies have limited success in treatment of this disease

A 68 year old male patient is brought to the emergency department (ED) by ambulance after being found unconscious on the bathroom floor by his spouse. Which action will the nurse take first? A. Check oxygen saturation B. Assess pupil reaction C. Palpate the head for injuries D. Verify Glasgow Coma Scale

A. Check oxygen saturation Rationale: Airway patency and breathing are the most vital functions and should be assessed first. The neurologic assessments should be accomplished next and additional assessment after that

An older patient with a history of an abdominal aortic aneurysm arrives at the emergency department (ED) with severe back pain and absent pedal pulses. Which action should the nurse take first? A. Check the blood pressure B. Draw blood for laboratory testing C. Assess for the presence of an abdominal bruit D. Determine any family history of heart disease

A. Check the blood pressure Rationale: Because the patient appears to be experiencing aortic dissection, the nurse's first action should be to determine the hemodynamic status by assessing blood pressure. The other actions may also be done, but they will not provide information to determine what interventions are needed immediately.

A patient tells the nurse about using acetaminophen (Tylenol) several times every day for recurrent bilateral headaches. Which action will the nurse plan to take first? A. Discuss the need to stop taking the acetaminophen B. Suggest the use of biofeedback for headache control C. Describe the use of botulism toxin (Botox) for headaches D. Teach the patient about magnetic resonance imaging (MRI)

A. Discuss the need to stop taking the acetaminophen Rationale: The headache description suggests that the patient is experiencing medication overuse headache. The initial action will be withdrawal of the medication. The other actions may be needed if headaches persist

The nurse is assessing the sexual-reproductive functional health of a 32 year old woman. Which question is most useful in determining the patient's sexual orientation and related risk factors? A. Do you have sex with men, women, or both? B. Which gender do you prefer to have sex with? C. What types of sexual activities do you prefer? D. Are you heterosexual, homosexual, or bisexual?

A. Do you have sex with men, women, or both? Rationale: This question is the most simply stated and will increase the likelihood of obtaining the relevant information about sexual orientation and possible risk factors associated with sexual activity. A patient who prefers sex with women may also have intercourse at times with men. The types of sexual activities engaged in may not indicate sexual orientation. Many patients who have sex with both men and women do not identify themselves as homosexual or bisexual.

A 22 year old patient tells the nurse at the health clinic that he has recently had some problems with erectile dysfunction. Which question should the nurse ask to assess for possible etiologic factors in this age group? A. Do you use recreational drugs or drink alcohol? B. Do you experience an unusual amount of stress? C. Do you have cardiovascular or peripheral vascular disease? D. Do you have a history of an erection that lasted for 6 hours or more?

A. Do you use recreational drugs or drink alcohol? Rationale: A common etiologic factor for erectile dysfunction (ED) in younger men is use of recreational drugs or alcohol. Stress, priapism, and cardiovascular illness also contribute to ED, but they are not common etiologic factors in younger men.

The patient comes to the HCP office with pain, edema, and warm skin oh her lower left leg. What test should the nurse expect to be ordered first? A. Duplex ultrasound B. Complete blood count C. Magnetic resonance imaging (MRI) D. Computed venography (phlebogram)

A. Duplex ultrasound Rationale: With manifestations of a VTE, the Duplex ultrasound is most widely used to diagnose VTE by identifying where a thrombus is located and its extent. D- dimer may also be drawn to determine if a VTE exists

For the patient undergoing a craniotomy, when should the nurse provide information about the use of wigs and hairpieces or other methods to disguise hair loss? A. During preoperative teaching B. If the patient asks about their use C. In the immediate postoperative period D. When the patient expresses negative feelings about his or her appearance

A. During preoperative teaching Rationale: To prevent undue concern and anxiety about hair loss and postoperative self-esteem disturbances, a patient undergoing cranial surgery should be informed preoperatively that the head is usually shaved in surgery while the patient is anesthetized and that a turban, scarf, or cap may be used after the dressings are removed postoperatively, and a wig also may be used after the incision has healed to disguise the hair loss. In the immediate postoperative period the patient is very ill and the focus is on maintaining neurologic function, but preoperatively the nurse should anticipate the patient's postoperative need for self-esteem and maintenance of appearance

A patient is admitted with possible botulism poisoning after eating home canned green beans. Which intervention ordered by the health care provider would the nurse question? A. Encourage oral fluids to 3L/day B. document neurologic symptoms C. position patient lying on the side D. observe respiratory status closely

A. Encourage oral fluids to 3L/day Rationale: The patient should be maintained on NPO status because neuromuscular weakness increases risk for aspiration. Side lying position is not contraindicated. Assessment of neurologic and respiratory status is appropriate

A patient who has bacterial meningitis is disoriented and anxious. Which nursing action will be included in the plan of care? A. Encourage the family members to remain at the bedside B. Apply soft restraints to protect the patient from injury C. Keep the room well lighted to improve patient orientation D. Minimize the contact with the patient to decrease sensory input

A. Encourage the family members to remain at the bedside Rationale: Patients with meningitis and disorientation will be calmed by the presence of someone familiar at the bedside. Restraints should be avoided because they increase agitation and anxiety. The patient requires frequent assessment for complications. The use of touch and a soothing voice will decrease anxiety for most patients. The patient will have photophobia, so the lights should be dim

A patient is admitted with a headache, fever, and general malaise. The HCP has asked that the patient be prepared for a lumbar puncture. What is a priority nursing action to avoid complications? A. Ensure that CT scan is performed prior to lumbar puncture B. Assess laboratory results for changes in the white cell counts C. Provide acetaminophen for the headache and fever before the procedure D. administer antibiotics before the procedure to treat potential meningitis

A. Ensure that CT scan is performed prior to lumbar puncture Rationale: A CT scan must be performed prior to lumbar puncture in a patient with possible increased intracranial pressure to avoid potential downward herniation of the brain when cerebrospinal fluid is drawn from the lumbar cistern

To prepare a woman who has been raped for physical examination, what should the nurse do first? A. Ensure that a signed informed consent is obtained from the patient. B. Provide a private place for the patient to talk about what happened to her C. Instruct the patient not to wash, eat, drink, or urinate before the examination D. Administer prophylaxis for sexually transmitted infections (STIs) and tetanus

A. Ensure that a signed informed consent is obtained from the patient. Rationale: Specific informed consent must be obtained from the rape victim before any examination can be made or rape data collected. Following consent, the patient is advised not to wash, eat, drink, or urinate before the examination so that evidence can be collected for medicolegal use. Prophylaxis for STIs, hepatitis B, and tetanus is administered following examination and follow up testing for pregnancy and human immunodeficiency virus (HIV) is done in several weeks

Surgical intervention is being considered for a patient with trigeminal neuralgia. The nurse recognizes that which procedure has the least residual effects with a positive outcome? A. Glycerol rhizotomy B. Gamma knife radiosurgery C. Microvascular decompression D. Percutaneous radiofrequency rhizotomy

A. Glycerol rhizotomy Rationale: Glycerol rhizotomy causes less sensory loss and fewer sensory aberrations with comparable pain relief and less danger than microvascular decompression and percutaneous radiofrequency rhizotomy, although these provide greater pain relief. Gamma knife radiosurgery provides precise high doses of radiation useful for persistent pain after other surgery

What is most important for the nurse to teach the female patient with genital warts? A. Have an annual Papanicolaou (Pap) test B. apply topical acyclovir faithfully as directed C. Have her sexual partner treated for the condition D. Use a contraceptive to prevent pregnancy, which may exacerbate the disease.

A. Have an annual Papanicolaou (Pap) test Rationale: Some types of genital warts are associated with cancer of the cervix, vagina, vulva, and throat or pharynx. Cancer of the penis, rectum, throat or pharynx may occur in men. Regular Papanicolaou (Pap) tests in women are critical in detecting early malignancies of the cervix. Oral acyclovir is used to treat HSV-2, but topical use has no value in treating viral STIs. Sexual partners of patients with HPV should be examined and treated, but because treatment does not destroy the virus, condoms should always be used during sexual activity. Genital warts often grow more rapidly during pregnancy, but pregnancy is not contraindicated

A patient with urinary obstruction from benign prostatic hyperplasia (BPH) tells the nurse, "My symptoms are much worse this week." Which response by the nurse is appropriate? A. Have you taken any over the counter (OTC) medications recently? B. I will talk to the doctor about a prostate specific antigen C. Have you talked to the doctor about surgery such as transurethral resection of the prostate (TURP)? D. The prostate gland changes in size from day to day, and this may be making your symptoms worse.

A. Have you taken any over the counter (OTC) medications recently? Rationale: Because the patient's increase in symptoms has occurred abruptly, the nurse should ask about OTC medications that might cause contraction of the smooth muscle in the prostate and worsen obstruction. The prostate gland does not vary in size from day to day. A TURP may be needed, but more assessment about possible reasons for the sudden symptom change is a more appropriate first response by the nurse. PSA testing is done to differentiate BPH from prostatic cancer

Following a lumbar puncture, for what should the nurse assess the patient? A. Headache B. Lower limb paralysis C. Allergic reactions to the dye D. Hemorrhage from the puncture site

A. Headache Rationale: A spinal headache, which may be caused by loss of CSF at the puncture site, is common following a lumbar puncturee or a myelogram, and nuchal rigidity may also occur as a result on meningeal irritation. The patient is not in danger of paralysis with a lumbar puncture, nor does hemorrhage from the site occur. Contrast media are not used with a lumbar puncture

What is a nursing intervention that is appropriate for the patient with a nursing diagnosis of anxiety related to lack of knowledge of the etiology and treatment of headache? A. Help the patient to examine lifestyle patterns and precipitating factors B. Administer medications as ordered to relieve pain and precipitating factors C. Provide a quiet, dimly lit environment to reduce stimuli that increase muscle tension and anxiety D. Support the patient's use of counseling or psychotherapy to enhance conflict resolution and stress reduction

A. Help the patient to examine lifestyle patterns and precipitating factors Rationale: When the anxiety is related to a lack of knowledge about the etiology and treatment of a headache, helping the patient to identify stressful lifestyle patterns and other precipitating factors and ways of avoiding them are appropriate interventions for anxiety. Interventions that teach alternate therapies to supplement drug therapy also give the patient some control over pain and are appropriate teaching regarding treatment of the headache. The other interventions may help to reduce anxiety, but they do not address the etiologic factor of anxiety

The wife of a man with moderate AD has a nursing diagnosis of social isolation related to diminishing social relationships and behavioral problems of the patient with AD. What is a nursing intervention that would be appropriate to provide respite care and allow the wife to have satisfactory contact with significant others? A. Help the wife to arrange for adult day care for the patient B. Encourage permanent placement of the patient in the Alzheimer's unit of a long term care facility C. Refer the wife to a home health agency to arrange daily home nursing visits to assist with the patient's care. D. Arrange for hospitalization of the patient for 3 or 4 days so that the wife can visit out of town friends and relatives

A. Help the wife to arrange for adult day care for the patient Rationale: Adult day care is an option to provide respite for caregivers and a protective environment for the patient during the early and middle stages of AD. There are also in home respite care providers. The respite from the demands of care allows the caregiver to maintain social contacts, perform normal tasks of living, and be more responsive to the patient's needs. Visits by home health nurses involve the caregiver and cannot provide adequate respite. Institutional placement is not always an acceptable option at earlier stages of AD, nor is hospitalization available for respite care

A Patient is diagnosed and treated for Gardnerella vaginalis infection at a clinic. For her treatment to be effective, what does the nurse tell the patient? A. Her sexual partner should also be examined and treated B. Her sexual partner must use a condom during intercourse C. She should wear minipads to prevent reinfection as long as she has vaginal drainage D. The vaginal suppository should be used in the morning so it will be fighting the infection all day

A. Her sexual partner should also be examined and treated Rationale: Gardnerella vaginalis infection is a bacterial vaginosis that may be sexually transmitted, and both partners may be infected. Treatment of the condition includes vaginal treatment with metronidazole (Flagyl) or clindamycin (Cleocin) or treatment via the oral route. Sexual activity is avoided until both partners are infection free. Minipads may be used to contain vaginal secretions, but they do not prevent reinfection. Vaginal suppositories and creams are used at bedtime so that the medication remains in the vagina for a long time

A 70 year old patient who has had a transurethral resection of the prostate (TURP) for benign prostatic hyperplasia (BPH) is being discharged from the hospital today, Which patient statement indicates a need for the nurse to provide additional instruction? A. I should call the doctor if I have incontinence at home B. I will avoid driving until I get approval from my doctor C. I should schedule yearly appointments for prostate examinations D. I will increase fiber and fluids in my diet to prevent constipation

A. I should call the doctor if I have incontinence at home Rationale: Because incontinence is common for several weeks after a TURP, the patient does not need to call the healthcare provider if this occurs., The other patient statements indicate that the patient has a good understanding of post-TURP instructions

A patient with a type A dissection of the arch of the aorta has a decreased LOC and weak carotid pulses. What should the nurse anticipate that initial treatment of the patient will include? A. Immediate surgery to replace the torn area with a graft B. Administration of anticoagulants to prevent embolization C. Administration of packed red blood cells (RBCs) to replace blood loss D. Administration of antihypertensives to maintain a mean arterial pressure of 70 to 80 mmHg

A. Immediate surgery to replace the torn area with a graft Rationale: Immediate surgery is indicated when complications (such as occlusion of the carotid arteries) occur. Otherwise, initial treatment for aortic dissection involves a period of lowering the BP and myocardial contractility to diminish the pulsatile forces in the aorta. Anticoagulants would prolong and intensify the bleeding and blood is administered only if the dissection ruptures

A patient with Alzheimer's disease (AD) dementia has manifestations of depression. The nurse knows that treatment of the patient with antidepressants will most likely do what? A. Improve cognitive function B. Not alter the course of either condition C. Cause interactions with the drugs used to treat the dementia D. Be contraindicated because of the central nervous system (CNS)-depressant effect of antidepressants

A. Improve cognitive function Rationale: Depression is often associated with AD, especially early in the disease when the patient has awareness of the diagnosis and progression of the disease. When dementia and depression occur together, intellectual deterioration may be more extreme. Depression is treatable, and use of antidepressants often improves cognitive function.

A patient has been taking phenytoin (Dilantin) for 2 years,. Which action will the nurse take when evaluating for adverse effects of the medication? A. Inspect the oral mucosa B. Listen to the lung sounds C. Auscultate the bowel sounds D. Check pupil reaction to light

A. Inspect the oral mucosa Rationale: Phenytoin can cause gingival hyperplasia, nut does not affect bowel sounds, lung sounds, or pupil reaction to light

A patient is seeking medical intervention for erectile dysfunction (ED). Why should he be thoroughly evaluated? A. It is important to determine if ED is reversible before treatment is started B. Psychologic counseling can reverse the problem in 80 to 90% of cases C. Most treatments for ED are contraindicated in patients with systemic diseases D. New invasive and experimental techniques currently used have known risks

A. It is important to determine if ED is reversible before treatment is started Rationale: Before treatment for ED is initiated, reversibility must be determined so that appropriate treatment can be planned. The actual cause may be determined, but this is more expensive. Only a small percentage of ED is caused by psychological factors. In the case of the 80-90% of ED that is of physiologic causes, interventions are directed at correcting or eliminating the cause or restoring function by medical means. Patients with systemic diseases can be treated medically if the cause cannot be eliminated. New invasive or experimental treatments are not widely used and should be limited to research centers

A patient in the emergency department reports that she has been sexually assaulted. Which action by the nurse will help to maintain the medicolegal chain of evidence? A. Labeling all specimens and other materials obtained from the patient. B. Assisting the patient in filling out the application for financial compensation C. Discussing the availability of the "morning after pill" for pregnancy prevention D. Educating the patient about baseline sexually transmitted infection (STI) testing

A. Labeling all specimens and other materials obtained from the patient. Rationale: The careful labeling of specimens and materials will assist in maintaining the chain of evidence. Assisting with paperwork, and discussing STIs and pregnancy prevention are interventions that might be appropriate after sexual assault, but they do not help maintain the legal chain of evidence

A woman who has multiple sclerosis (MS) asks the nurse about risks associated with pregnancy. Which response by the nurse is accurate? A. MS symptoms may be worse after the pregnancy B. Women with MS frequently have premature labor C. MS is associated with an increased risk for congenital defects D. Symptoms of MS are likely to become worse during pregnancy

A. MS symptoms may be worse after the pregnancy Rationale: During the postpartum period, women with MS are at greater risk for exacerbation of symptoms, There is no increased risk for congenital defects in infants born of mothers with MS. Symptoms of MS may improve during pregnancy. Onset of labor is not affected by MS

What is an appropriate outcome for a patient who undergoes an anterior colporrhaphy? A. Maintain normal bowel patterns B. Adjust to temporary ileal conduit C. Urinate within 8 hours postoperatively D. Experience healing of excoriated vaginal and vulvar tissue

A. Maintain normal bowel patterns Rationale: An anterior copporrhaphy involves repair of cystocele and an indwelling urinary catheter is left in place for several days postoperatively while healing occurs. Bowel function should not be altered and is maintained with a low residue diet and a stool softener if necessary to avoid straining and pressure on the incision

A key aspect of teaching for the patient on anticoagulant therapy includes which instructions? A. Monitor for and report any signs of bleeding B. Do not take acetaminophen (Tylenol) for a headache C. Decrease your dietary intake of foods containing vitamin K D. Arrange to have blood drawn twice a week to check drug effects

A. Monitor for and report any signs of bleeding Rationale: Patients taking anticoagulants should be taught to monitor and report any signs of bleeding, which can be a serious complication. Other important patient teaching includes maintenance of a consistent intake of foods containing vitamin K and avoidance of supplements that contain vitamin K. If a patient is taking warfarin, routine coagulation laboratory studies are necessary, although frequency is patient dependent, not necessarily twice a week.

A 29 year old female patient is diagnosed with chlamydia during a routine pelvic examination. The nurse knows that teaching regarding the management of the condition has been effective when the patient says which of the following? A. My partner will need to take antibiotics at the same time I do B. Go ahead and give me the antibiotic injection, so I will be cured C. I will use condoms during sex until I finish taking all the antibiotics D. I do not plan on having children, so treating the infection is not important

A. My partner will need to take antibiotics at the same time I do Rationale: Sex partners should be treated simultaneously to prevent reinfection. Chlamydia is treated with oral antibiotics. Abstinence from sexual intercourse is recommended for 7 days after treatment, and condoms should be recommended during all sexual contacts to prevent infection. Chronic pelvic pain, as well as infertility, can result from untreated chlamydia

Which infection, reported in the health history of a female patient who is having difficulty conceiving, will the nurse identify as a risk factor for infertility? A. N. gonorrhea B. Treponema pallidum C. Condyloma acuminatum D. Herpes simplex virus type 2

A. N. gonorrhea Rationale: Complications of gonorrhea include scarring of the fallopian tubes, which can lead to tubal pregnancies and infertility. Syphilis, genital warts, and genital herpes do not lead to problems with conceiving, although transmission to a fetus (syphilis) or newborn (genital warts or genital herpes) is a concern

The nurse is caring for a patient immediately after repair of an abdominal aortic aneurysm. On assessment, the patient has absent popliteal, posterior tibial, and dorsalis pedis pulses. The legs are cool and mottled. Which action should the nurse take first? A. Notify the surgeon and anesthesiologist B. Warp both the legs in a warming blanket C. Document the findings and recheck in 15 minutes D. Compare findings to the preoperative assessment of the pulses

A. Notify the surgeon and anesthesiologist Rationale: Lower extremity pulses may be absent for a short time after surgery because of vasospasm and hypothermia. Decreased or absent pulses together with a cool and mottled extremity may indicate embolization or graft occlusion. These findings should be reported to the surgeon immediately because this is an emergency situation. Because pulses are marked before surgery, the nurse would know whether pulses were present before surgery before notifying the health care providers about absent pulses. Because the patient's symptoms may indicate graft occlusion or multiple emboli and a possible need to return to surgery, it it not appropriate to wait 15 minutes before taking action. A warming blanket will not improve thee circulation to the patient's legs

A 22 year old patient tells the nurse that she has not had a menstrual period for the past 3 months. Which action is most important for the nurse to take? A. Obtain a urine specimen for a pregnancy test B. Ask about any recent stressful lifestyle changes C. Measure the patient's current height and weight D. Question the patient about prescribed medications

A. Obtain a urine specimen for a pregnancy test Rationale: Pregnancy should always be considered a possible cause of amenorrhea in women of childbearing age. The other actions are also appropriate, but it is important to check for pregnancy in this patient because pregnancy will require rapid implementation of actions to promote normal fetal development such as changes in lifestyle, folic acid intake, and so on.

Which surgical therapy for AAA is most likely to have the postoperative complication of renal injury? A. Open aneurysm repair (OAR) above the level of the renal arteries B. Excising only the weakened area of the artery and suturing the artery closed C. Bifurcated graft used in aneurysm repair when the AAA extends into the iliac arteries D. Endovascular graft procedure with an aortic graft inside the aneurysm via the femoral artery

A. Open aneurysm repair (OAR) above the level of the renal arteries Rationale: With the aortic cross clamping proximal and distal to the aneurysm, the open aneurysm repair (OAR) above the renal artery may cause kidney injury from the lack of blood flow during the surgery. The saccular aneurysm may involve excising only the weakened area of the artery and suturing the artery closed, but this will not decrease renal blood flow. Renal blood flow will not be directly obstructed using the bifurcated graft or the minimally invasive endovascular aneurysm repair

The nurse evaluates that treatment for the patient with an uncomplicated aortic dissection is successful when what happens? A. Pain is relieved B. Surgical repair is completed C. BP is increased to normal range D. Renal output is maintained at 30mL/hr

A. Pain is relieved Rationale: Relief of pain is an indication that the dissection has stabilized, and it may be treated conservatively for an extended time with drugs that lower BP and decrease myocardial contractility to diminish the pulsatile forces in the aorta. Anticoagulants would prolong and intensify the bleeding and blood is administered only if the dissection ruptures

The nurse is caring for a client who has a head injury. Which finding, when reported to the health care provider, should the nurse expect will result in new prescribed interventions? A. Pale yellow urine output of 1200mL over the past 2 hours B. Ventriculostomy drained 40mL of fluid in the past 2 hours C. Intracrainal pressure spikes to 16mmHg when patient is turned D. LICOX brain tissue oxygenation catheter shows PbtO2 if 38mmHg

A. Pale yellow urine output of 1200mL over the past 2 hours Rationale: The high urine output indicates that diabetes insipidus may be developing, and interventions to prevent dehydration need to rapidly implemented. The other data do not indicate a need for any change in therapy

After change-of-shift report, which patient should the nurse assess first? A. Patient with myasthenia gravis who is reporting increased muscle weakness B. Patient with a bilateral headache described as "like a band around my head" C. Patient with seizures who is scheduled to receive a dose of phenytoin (Dilantin) D. Patient with Parkinson's disease who has developed cogwheel rigidity of the arms

A. Patient with myasthenia gravis who is reporting increased muscle weakness Rationale: Because increased muscle weakness may indicate the onset of a myasthenic crisis, the nurse should assess this patient first. The other patients should also be assessed but do not appear to need immediate nursing assessments or actions to prevent life threatening complications

A couple seeks assistance from an infertility specialist for evaluation of their infertility. What does the nurse inform the couple that they can expect during the initial visit? A. Physical and psychosocial functioning examinations B. Assessment of the tubal patency with a hysterosalpingogram C. Pelvic ultrasound for the woman and semen analysis for the man D. Postcoital testing to evaluate sperm numbers and motility in cervical and vaginal secretions

A. Physical and psychosocial functioning examinations Rationale: The initial visit of a couple seeking assistance with infertility includes a history and physical for both partners, psychosocial functioning, testing for medical problems and sexually transmitted infections (STIs), a cervical Papanicolaou (Pap) test, possible semen analysis, and instruction for at home ovulation testing. A discussion of possible future testing options and cost is also done. If the couple decides to continue with treatment, further visits will include more intensive evaluation, including postcoital testing, a hysterosalpingogram, pelvic ultrasound, and midluteal progesterone and prolactin levels

In preparation for AAA repair surgery, what should the nurse include in patient teaching? A. Prepare the bowel on the night before the surgery with laxatives or an enema B. Use moisturizing soap to clean the skin three times a day before surgery C. Eat a high protein and high carbohydrate breakfast to help with healing postoperatively D. Take the prescribed oral antibiotic the morning of surgery before going to the operating room

A. Prepare the bowel on the night before the surgery with laxatives or an enema Rationale: Usually aortic surgery patient will have a bowel preparation, skin cleansing with an antimicrobial agent on the day before surgery, nothing by mouth after midnight on the day of the surgery, and IV antibiotics immediately before the incision is made. Patients with a history of cardiovascular disease will receive a B-adrenergic blocker preoperatively to reduce morbidity and mortality. Each surgeon's protocol may be different

Which topic will the nurse include in the preoperative teaching for a patient admitted for an abdominal hysterectomy? A. Purpose of ambulation and leg exercises B. Adverse effects of systemic chemotherapy C. Decrease in vaginal sensation after surgery D. Symptoms caused by the drop in estrogen level

A. Purpose of ambulation and leg exercises Rationale: Venous thromboembolism is a potential complication after the surgery, and the nurse will instruct the patient about ways to prevent it. Vaginal sensation is decreased after a vaginal hysterectomy but not after an abdominal hysterectomy. Most hysterectomies are not done for treatment of cancer. Unless the patient has cancer, chemotherapy, and radiation will not be prescribed. Because the patient will still have her ovaries, her estrogen will not decrease

The patient was in a traffic collision and is experiencing loss of function below C4. Which effect must the nurse be aware of to provide priority care for the patient? A. Respiratory diaphragmatic breathing B. Loss of all respiratory muscle function C. Decreased response of the sympathetic nervous system D. GI hypomotility with paralytic ileus and gastric distention

A. Respiratory diaphragmatic breathing Rationale: SCI below C4 will result in diaphragmatic breathing and usually hypoventilation from decreased vital capacity and tidal volume from intercostal muscle impairment. The nurse's priority actions will be to monitor rate, rhythm, depth, and effort of breathing to observe for changes from the baseline and identify the need for ventilation assistance. Loss of all respiratory muscle function occurs above C4, and the patient requires mechanical ventilation to survive. Although the decreased sympathetic nervous system response (from injuries above T6) and GI hypomotilitiy (paralytic ileus and gastric distention) will occur (with injuries aboe T5), they are not the patient's initial priority needs

A patient with possible viral meningitis is admitted to the nursing unit after lumbar puncture was performed in the emergency department. Which action prescribed by the health care provider should the nurse question? A. Restrict oral fluids to 1000mL/day B. Elevate the head of the bed 20 degrees C. Administer ceftriaxone (Rocephin) 1g IV every 12 hours D. Give ibuprofen (Motrin) 400mg every 6 hours as needed for headache

A. Restrict oral fluids to 1000mL/day Rationale: The patient with meningitis has increased fluid needs, so oral fluids should be encouraged. The other actions are appropriate. Slight elevation of the head of the bed will decrease headache without causing leakage of cerebrospinal fluid from the lumbar puncture site. Antibiotics should be administered until bacterial meningitis is ruled out by cerebral spinal fluid analysis

A patient in the outpatient clinic has a new diagnosis of peripheral artery disease (PAD). Which group of medications will the nurse plan to include when providing patient teaching about PAD management? A. Statins B. Antibiotics C. Thrombolytics D. Anticoagulants

A. Statins Rationale: Research indicates that statin use by patients with PAD improves multiple outcomes. There is no research that supports the use of the other drug categories in PAD

Which type of seizure is most likely to cause death for the patient? A. Status epilepticus B. Myoclonic seizures C. Subclinical seizures D. Psychogenic seizures

A. Status epilepticus Rationale: Status epilepticus is most dangerous because the continuous seizing can cause respiratory insufficiency, hypoxemia, cardiac dysrhythmia, hyperthermia, and systemic acidosis, which can all be fatal. Myoclonic seizures may occur in clusters and have a sudden, excessive jerk of the body that may hurl the person to the ground. Subclinical seizures may occur in a patient who is sedated, so there is no physical movement. Psychogenic seizures are psychiatric in origin and diagnoses with video electroencephalography (EEG) monitoring. They occur in patients with a history of emotional abuse or a specific traumatic episode

Patients with which STI are most likely to avoid obtaining and following treatment measures for their infection? A. Syphilis B. Gonorrhea C. HPV infection D. Genital Herpes

A. Syphilis Rationale: STIs such as syphilis, that can be treated with a single dose or short course antibiotic therapy often lead to a casual attitude about the outcome of this disease, which leads to nonadherence with instructions and delays in treatment. This is particularly true of diseases that initially show few distressing or uncomfortable symptoms, such as syphilis

A patient is admitted to the hospital with Guillain Barre syndrome. She had weakness in her feet and ankles that has progressed to weakness with numbness and tingling in both legs. During the acute phase of the illness, what should the nurse know about Guillain Barre syndrome? A. The most important aspect of care is to monitor the patient's respiratory rate and depth and vital capacity B. Early treatment with corticosteroids can suppress the immune response and prevent ascending nerve damage C. The most serious complication of this condition is ascending demyelination of the peripheral nerves and the cranial nerves D. Although voluntary motor neurons are damaged by the inflammatory response, the autonomic nervous system is unaffected by the disease

A. The most important aspect of care is to monitor the patient's respiratory rate and depth and vital capacity Rationale: The most serious complication of Guillain Barre syndrome is respiratory failure, and it is essential that respiratory rate and depth, ABGs, and vital capacity are monitored to detect involvement of the autonomic nerves that affect respiration. Corticosteroids do not appear to have an effect on the prognosis or duration of the disease. Rather, plasmapheresis or administration of high dose immunoglobulin does result in shortening the recovery time. The peripheral nerves of both the sympathetic and parasympathetic nervous systems are involved in the disease and may lead to orthostatic hypotension, hypertension, and abnormal vagal responses affecting the heart. Guillain Barre syndrome may affect the lower brainstem and cranial nerves VII, VI, III, XII, V, X, affecting facial, eye, and swallowing functions

After teaching a patient with newly diagnosed Raynaud's phenomenon about how to manage the condition, which action by the patient best demonstrates that the teaching has been effective? A. The patient exercises indoors during winter months B. The patient immerses hands in hot water when they turn pale C. The patient takes pseudoephedrine (Sudafed) for cold symptoms D. The patient avoids taking nonsteroidal antiinflammatory drugs (NSAIDs)

A. The patient exercises indoors during winter months Rationale: Patients should avoid temperature extremes by exercising indoors when it is cold. To avoid burn injuries, the patient should use warm rather than hot water to warm the hands. Pseudoephedrine is a vasoconstrictor and should be avoided. There is no reason to avoid taking NSAIDs with Raynaud's syndrome

A 54 year old man is recovering from a skull fracture with a subacute subdural hematoma that caused unconsciousness. He has return of motor control and orientation but appears apathetic and has reduced awareness of his environment. When planning discharge of the patient, what should the nurse explain to the patient and family? A. The patient is likely to have long-term emotional and mental changes that may require professional help B. Continuous improvement in the patient's condition should occur until he has returned to pretrauma status. C. The patient's complete recovery may take years, and the family should plan for his long term dependent care D. Role changes in family members will be necessary because the patient will be dependent on his family for care and support

A. The patient is likely to have long-term emotional and mental changes that may require professional help Rationale: Residual mental and emotional changes of brain trauma with personality changes are often the most incapacitating problems following head injury and are common in patients who have been comatose for longer than 6 hours. Families must be prepared for changes in the patient's behavior to avoid family-patient friction and maintain family functioning, and professional assistance may be required. There is no indication the patient will be dependent on others for care, but he likely will not return to pretrauma status

A patient who is hospitalized with pneumonia is disoriented and confused 3 days after admission. Which information indicates that the patient is experiencing delirium rather than dementia? A. The patient was oriented and alert when admitted B. The patient's speech is fragmented and incoherent C. The patient is oriented to person but disoriented to place and time D. The patient has a history of increasing confusion over several years

A. The patient was oriented and alert when admitted Rationale: The onset of delirium occurs acutely. The degree of disorientation does not differentiate between delirium and dementia. Increasing confusion for several years is consistent with dementia. Fragmented and incoherent speech may occur with either delirium or dementia

A 38 year old woman has newly diagnosed multiple sclerosis (MS) and asks the nurse what is going to happen to her. What is the best response by the nurse? A. You will have either periods of attacks and remissions or progression of nerve damage over time B. You need to plan for a continuous loss of movement, sensory functions, and mental capabilities C. You will most likely have a steady course of chronic progressive nerve damage that will change your personality D. It is common for people with MS to have an acute attack of weakness and then not to have any other symptoms for years

A. You will have either periods of attacks and remissions or progression of nerve damage over time Rationale: Most patients with multiple sclerosis (MS) have remissions and exacerbations of neurologic dysfunction or a relapsing-remitting initial course followed by progression with or without occassional relapses, minor remissions, and plateaus that progressively cause loss or motor, sensory, and cerebellar functions. Intellectual function generally remains intact, but patients may experience anger, depression, or euphoria. A few people have chronic progressive deterioration, and some may experience only occasional and mild symptoms for several years after onset

A patient has a nursing diagnosis of risk for ineffective cerebral tissue perfusion related to cerebral edema. An appropriate nursing intervention for the patient is A. avoiding positioning the patient with neck and hip flexion B. maintaining hyperventilation to a PaCO2 of 15 to 20 mm Hg C. clustering nursing activities to provide periods of uninterrupted rest D. routine suctioning to prevent accumulation of respiratory secretions

A. avoiding positioning the patient with neck and hip flexion Rationale: Nursing care activities that increase ICP include hip and neck flexion, suctioning, clustering care activities, and noxious stimuli. They should be avoided or performed as little as possible in the patient with increased ICP. Lowering the PaCO2 below 20mmHg can cause ischemia and worsening of ICP

A 50 year old man complains of recurring headaches. He describes these as sharp, stabbing, and located around his left eye. He also reports that his left eye seems to swell and get teary when these headaches occur. Based on this history, you suspect that he has A. cluster headaches B. tension headaches C. migraine headaches D. medication overuse headaches

A. cluster headaches Rationale: Cluster headaches involve repeated headaches that can occur for weeks to months, followed by periods of remission. The pain of cluster headache is sharp and stabbing; the intense pain lasts a few minutes to 3 hours. Cluster headaches can occur every other day and as often as eight times a day. The clusters occur with regularity, usually at the same time each day and during the same seasons of the year. Typically, a cluster lasts 2 weeks to 3 months, and the patient then goes into remission for months to years, The pain usually is located around the eye and radiates to the temple, forehead, cheek, nose, or gums. Other manifestations may include swelling around the eye, lacrimation (tearing), facial flushing or pallor, nasal congestion, and constriction of the pupil. During the headache, the patient is often agitated and restless, unable to sit still or relax

A 22 year old woman with multiple sexual partners seeks care after several weeks of experiencing painful and frequent urination and vaginal discharge. Although the results of a culture of cervical secretions are not yet available, the nurse explains to the patient that she will be treated as if she has gonorrhea and chlamydia to prevent A. damage to the fallopian tubes B. endocarditis and aortic aneurysms C. disseminated gonococcal infection D. polyarthritis and generalized adenopathy

A. damage to the fallopian tubes Rationale: Upward extension of gonorrhea or chlamydia commonly causes PID, which can cause adhesions and fibrous scarring, leading to tubal pregnancies and infertility. Disseminated gonococcal infection is rare, and endocarditis and aneurysms are associated with syphilis. Polyarthritis and adenopathy are not seen in gonorrhea or chlamydia

The nurse will inform a patient with cancer of the prostate that side effects of leuprolide (Lupron) may include A. flushing B. dizziness C. infection D. incontinence

A. flushing Rationale: Hot flashes may occur with decreased testosterone production. Dizziness may occur with the a-blockers used for a benign prostatic hyperplasia. Urinary incontinence may occur after prostate surgery, but it is not an expected side effect of medication. Risk for infection is increased in patient receiving chemotherapy

During the patient's process of grieving for the losses resulting from spinal cord injury, what should the nurse do? A. help the patient understand that working through the grief will be a lifelong process B. Assist the patient to move through all stages of the mourning and grief process to acceptance C. let the patient know that anger directed at the staff or the family is not a positive coping mechanism D. facilitate the grieving process so that it is completed by the time the patient is discharged from rehabilitation

A. help the patient understand that working through the grief will be a lifelong process Rationale: Working through the grief process is a lifelong process that is triggered by new experiences, such as marriage, child rearing, employment, or illness, which the patient must adjust to throughout life within the context of his or her disability. The goal of recovery is related to adjustment rather than complete acceptance, and many patients do not experience all components of the grief process. During the anger phase, patients should be allowed outbursts and the nurse may use humor to displace some of the patients anger

A patient hospitalized with a new diagnosis of Gullain Barre syndrome has numbness and weakness of both feet. The nurse will anticipate teaching the patient about A. infusion of immunoglobulin B. Intubation and mechanical ventilation C. administration of corticosteroid drugs D. insertion of a nasogastric (NG) feeding tube

A. infusion of immunoglobulin Rationale: Because Gullain Barre syndrome is in the earliest stages (as evidenced by the symptoms), use of high dose immunoglobulin is appropriate to reduce the extent and length of symptoms. Mechanical ventilation and tube feedings may be used later in the progression of the syndrome but are not needed now. Corticosteroid use is not helpful in reducing the duration or symptoms of the syndrome

The nurse advises a patient with myasthenia gravis (MG) to A. perform physically demanding activities early in the day. B. anticipate the need for weekly plasmapheresis treatments. C. do frequent weight-bearing exercise to prevent muscle atrophy. D. protect the extremities from injury due to poor sensory perception.

A. perform physically demanding activities early in the day. Rationale: Muscles are generarlly strongest in the morning, and activities involving muscle activity should be scheduled then. Plasmapheresis is not routinely scheduled but is used for myasthenia crisis or for situations in which corticosteroid therapy must be avoided. There is no decrease in sensation with MG, and muscle atrophy does not occur because although there is muscle weakness, there are still used

A patient with Bell's palsy refuses to eat while others are present because of embarrassment about drooling. The best response by the nurse to the patient's behavior is to A. respect the patient's feelings and arrange for privacy at mealtimes B. teach the patient to chew food on the unaffected side of the mouth C. offer the patient liquid nutritional supplements at frequent intervals D. discuss the patient's concerns with visitors who arrive at mealtimes

A. respect the patient's feelings and arrange for privacy at mealtimes Rationale: The patient's desire for privacy should be respected to encourage adequate nutrition and reduce patient embarrassment. Liquid supplements may help maintain nutrition but will reduce the patient's enjoyment of the taste of food. It would be inappropriate for the nurse to discuss the patient's embarrassment with visitors unless the patient wishes to share this information. Chewing on the unaffected side of the mouth will enhance nutrition and enjoyment of food but will not decrease the drooling

A patient with endometriosis asks why she is being treated with medroxyprogesterone, a medication that she thought was a contraceptive. The nurse explains that this theory A. suppress the menstrual cycle by mimicking pregnancy B. relieves symptoms such as vaginal atrophy and hot flashes C. prevents a pregnancy that could worsen the menstrual bleeding D. leads to permanent suppression of abnormal endometrial tissues

A. suppress the menstrual cycle by mimicking pregnancy Rationale: Medroxyprogesterone induces a pseudopregnancy, which suppresses ovulation and causes shrinkage of endometrial tissue. Menstrual bleeding does not occur during pregnancy. Vaginal atrophy and hot flashes are caused by synthetic androgens such as danazol or gonadotropin releasing hormone agonists such as leuprolide. Although hormonal therapies will control endometriosis while the therapy is used, endometriosis will recur once the menstrual cycle is reestablished

The nurse teaches the patient with any venous disorder that the best way to prevent venous stasis and increase venous return is to A. take short walks B. Sit with the legs elevated C. frequently rotate the ankles D. Continuously wear elastic compression stockings

A. take short walks Rationale: During walking, the muscles of the legs continuously knead the veins, promoting movement of venous blood toward the heart. Walking is the best measure to prevent venous stasis and will be increased gradually. Elevating the legs will decrease edema. The other methods will help venous return, but they do not provide the benefit that ambulation does

A patient is being evaluated for a possible spinal cord tumor. Which finding by the nurse requires the most immediate action? A. the patient has new onset weakness of both legs B. The patient complains of chronic severe back pain C. the patient starts to cry and says, I feel hopeless D. The patient expresses anxiety about having surgery

A. the patient has new onset weakness of both legs Rationale: The new symptoms indicate spinal cord compression, and emergency that requires rapid treatment to avoid permanent loss of function. The other patient assessments also need nursing action but do not require intervention as rapidly as the new onset weakness

After reviewing the health care record in the accompanying figure for a patient who has multiple risk factors for Alzheimer's disease (AD), which topic will be most important for the nurse to discuss with the patient? Patient history: Age 58; history of closed head injury; mother died at age 68 of Alzheimer's disease Habits: smokes 15 cigarettes daily; 1-2 glasses of wine weekly; rides a bike to and from work Laboratory results: Total cholesterol 220mg/dL; high density lipoprotein 80mg/dL; low density lipoprotein 103mg/dL A. tobacco use B. Family history C. Cholesterol level D. Head injury history

A. tobacco use Rationale: Tobacco use is a modifiable risk factor for AD. The patient will not be able to modify the increased risk associated with family history of AD and past head injury. While the total cholesterol is borderline high, the high HDL indicates that no change is needed in cholesterol management

When evaluating the discharge teaching for a patient with chronic peripheral artery disease (PAD), the nurse determines a need for further instruction when the patient says, "I will A. use a heating pad on my feet at night to increase the circulation." B. buy some loose clothes that do not bind across my legs or waist." C. walk to the point of pain, rest, and walk again for at least 30 minutes 3 times a week." D. change my position every hour and avoid long periods of sitting with my legs crossed."

A. use a heating pad on my feet at night to increase the circulation." Rationale: Because the patient has impaired circulation and sensation to the feet, the use of a heating pad could lead to burns. The other patient statements are correct and indicate that teaching has been successful

In assessing patients for STIs, the needs to know that many STIs can be asymptomatic . Which STIs can be asymptomatic (select all that apply) A. syphilis B. gonorrhea C. genital warts D. genital herpes E. chlamydial infection

A.B.C.D.E. syphilis, gonorrhea, genital warts, genital herpes, chlamydial infection Rationale: Syphilis (especially in later stages), gonorrhea, genital warts, genital herpes, and chlamydial infection can all be asymptomatic. Because of the high prevalence of asymptomatic STIs, screening of populations at high risk is needed to identify those who are infected

What manifestations are characteristic of the late or tertiary stage of syphilis (Select all that apply)? A. Heart failure B. Tabes dorsalis C. Aortic aneurysms D. Mental deterioration E. Generalized cutaneous rash F. Destructive skin, bone, and soft tissue lesions

A.B.C.D.F. Heart failure; Tabes dorsalis; Aortic aneurysms; Mental deterioration; Destructive skin, bone, and soft tissue lesions Rationale: In the tertiary (or late) stage of syphilis there can be cardiovascular problems (heart failure, aneurysms, valve insufficiency), gummas (chronic destructive lesions), and neurosyphilis manifestations (mental deterioration, tabes dorsalis, and speech disturbances). Generalized cutaneous rash occurs in the secondary stage of syphilis, a few weeks after the chancre appears

What are characteristics of vasospastic disease (Raynaud's phenomenon) (select all that apply)? A. Predominant in young females B. May be associated with autoimmune disorders C. Precipitated by exposure to cold, caffeine, and tobacco D. Involves small cutaneous arteries of the fingers and toes E. Inflammation of small and medium sized arteries and veins F. Episodes involve white, blue, and red color changes of fingertips

A.B.C.D.F. Predominant in young females; May be associated with autoimmune disorders; Precipitated by exposure to cold, caffeine, and tobacco; Involves cutaneous arteries of the fingers and toes; episodes involve white, blue, and red color changes of the fingertips Rationale: Raynaud's phenomenon is predominant in young females and may be associated with autoimmune disorders (e.g. rheumatoid arthritis, scleroderma, systemic lupus erythematosus). Incidents occur with cold, emotional upsets, and caffeine or tobacco use due to vasoconstrictive effects. Small cutaneous arteries are involved and cause color changes of the fingertips or toes. When conservative management is ineffective, it may be treated with nifedipine (Procardia)

Which diagnostic tests of the female reproductive system are operative procedures requiring surgical anesthesia (select all that apply) A. D&C B. Conization C. Culdoscopy D. Colposcopy E. Laparoscopy F. Endometrial biopsy

A.B.C.E. D&C; Conization; Culdoscopy; Laparoscopy Rationale: A D&C and laparoscopy are operative procedures requiring surgical anesthesia. A culdoscopy involves insertion of an endoscope through an incision made through the posterior fornix of the cul-de-sac and requires sedation, as does the removal of cervical tissue during a conization. Colposcopy and endometrial biopsies do not require surgical anesthesia

Which nonhormonal therapies will the nurse suggest for a healthy perimenopausal woman who prefers not to use hormone therapy (HT) (select all that apply)? A. Reduce coffee intake. B. Exercise several times a week. C. Take black cohosh supplements. D. Have a glass of wine in the evening. E. Increase intake of dietary soy products.

A.B.C.E. Reduce coffee intake; exercise several times a week; take black cohosh supplements; increase intake of dietary soy items Rationale: Reduction in caffeine intake, use of black cohosh, increasing dietary soy intake, and exercising three to four times weekly are recommended to reduce symptoms associated with menopause. Alcohol intake in the evening may increase the sleep problems associate with menopause

Stimulation of the parasympathetic nervous system results in (select all that apply) A. constriction of the bronchi B. dilation of skin blood vessels C. increased secretion of insulin D. increased blood glucose levels E. relaxation of the urinary sphincters

A.B.C.E. constriction of the bronchi; dilation of the skin blood vessels; increased secretion of insulin; relaxation of the urinary sphincters Rationale: Stimulation of the parasympathetic nervous system results in constriction of the bronchi, dilation of blood vessels to the skin, increased secretion of insulin, and relaxation of the urinary sphincter. Stimulation of the sympathetic nervous system results in increased blood glucose levels

Luteinizing hormone (LH) secretion by the anterior pituitary (select all that apply) A. results in ovulation B. causes follicles to complete maturation C. affects development of ruptured follicles D. directly inhibits both GnRH and FSH secretion E. stimulates testosterone production by interstitial cells of testes

A.B.C.E. results in ovulation; causes follicles to complete maturation; affects development of ruptured follicles; stimulates testosterone production by interstitial cells of testes Rationale: Luteinizing hormone (LH) causes follicles to complete maturation and undergo ovulation. LH also affects the development of a ruptured follicle. In men, LH (or ICSH) triggers testosterone production but the interstitial cells of the testes. Inhibin directly inhibits both GnRH and FSH secretion

The nurse in the outpatient clinic notes that the following patients have not received the human papillomavirus (HPV) vaccine. Which patients should the nurse plan to teach about benefits of the vaccine? (Select all that apply) A. A 24 year old male patient who has a history of genital warts B. An 18 year old male patient who has had one male sexual partner C. A 38 year old female patient who has never been sexually active D. A 20 year old female patient who has a newly diagnosed chlamydia infection E. A 30 year old female patient whose sexual partner has a history of genital warts

A.B.D. A 24 year old male patient who has a history of genital warts; An 18 year old male patient who has had one male sexual partner; A 20 year old female patient who has a newly diagnosed chlamydia infection Rationale: The HPV vaccines are recommended for male and female patients between ages 9 through 26 years. There are several types of HPV. Ideally, the vaccines are administered before patients are sexually active, but they offer benefit even to those who already have HPV infection because the vaccines protect against HPV types not already acquired

A patient with Parkinson's disease is admitted to the hospital for treatment of pneumonia. Which nursing interventions will be included in the plan of care (select all that apply)? A. Use an elevated toilet seat. B. Cut patient's food into small pieces. C. Serve high-protein foods at each meal. D. Place an armchair at the patient's bedside. E. Observe for sudden exacerbation of symptoms.

A.B.D. Use an elevated toilet seat; Cut patient's food into small pieces; Place an armchair at the patient's bedside Rationale: Because the patient with Parkinson's disease has difficulty chewing, food should be cut into small pieces. An armchair should be used when the patient is seated so that the patient can use the arms to assist with getting up from the chair. An elevated toilet seat will facilitate getting on and off the toilet. High protein foods will decrease the effectiveness of L-dopa. Parkinson's disease is a steadily progressive disease without acute exacerbations

The nurse is monitoring a patient for increased ICP following a head injury. Which of the following manifestations indicate an increased ICP (select all that apply) A. fever B. oriented to name only C. narrowing pulse pressure D. dilated right pupil greater than left pupil E. decorticate posturing to painful stimulus

A.B.D.E. fever; oriented to name only; right pupil dilated greater than left pupil; Decorticate posturing to painful stimulation Rationale: The first sign of increased ICP is a change in LOC. Other manifestations are dilated ipsilateral pupil, changes in motor response such as posturing, and fever, which may indicate pressure on the hypothalamus. Changes in vital signs would be an increased SBP with widened pulse pressure and bradycardia

The surgery area calls the transfer report for a 68 year old, postmenopausal, female patient who smokes and takes hormone therapy. She is returning to the clinical unit after a lengthy hip replacement surgery. Which factors present in this patient increase her risk for developing venous thromboembolism (VTE) related to Virchow's triad? (select all that apply) A. Smoking B. IV therapy C. Dehydration D. Estrogen therapy E. Orthopedic surgery F. Prolonged Immobilization

A.B.D.E.F. Smoking; IV therapy; Estrogen therapy; Orthopedic surgery; prolonged immobilization Rationale: This patient is a smoker and on hormone therapy, both of which increase blood hypercoagulability. She will have an IV, and her fractured hip can cause VTE by damaging the venous endothelium. She is an older patient who has had an orthopedic surgery and may have experienced prolonged immobility postinjury and through her "lengthy hip replacement surgery," which contributes to venous stasis. These are respresentative of Virchow's triad in this patient. The other options are also related to Virchow's triad but not present in this patient via the transfer report

Postoperative goals in caring for the patient who has undergone an abdominal hysterectomy include (select all that apply) A. monitoring urine output B. changing position frequently C. restricting all food for 24 hours D. observing perineal pad for bleeding E. encouraging leg exercises to promote circulation

A.B.E. monitoring urine output; changing position frequently; encouraging leg exercises to promote circulation Rationale: After an abdominal hysterectomy, postoperative care includes monitoring urinary output because urinary retention may occur from temporary bladder atony related to edema or nerve trauma. Frequent changes of position, avoidance of the high Fowler's position, and avoidance of pressure under the knees minimize the risk of deep vein thrombosis (DVT). Food and fluids may be restricted if the patient is nauseated. Leg exercises promote circulation. After an abdominal hysterectomy, the nurse observes the abdominal dressing for bleeding

The sister of a patient with AD asks the nurse whether prevention of the disease is possible. In responding, the nurse explains that there is no known way to prevent AD, but there are ways to keep the brain healthy. What is included in the ways to keep the brain healthy (Select all that apply)? A. Avoid trauma to the brain B. Recognize and treat depression early C. Avoid social gatherings to avoid infections D. Do not overtax the brain by trying to learn new skills E. Daily wine intake will increase circulation to the brain F. Exercise regularly to decrease the risk for cognitive decline

A.B.F. Avoid trauma to the brain; Recognize and treat depression early; Exercise regularly to decrease the risk for cognitive decline Rationale: Avoiding trauma to the brain, treating depression early, and exercising regularly can maintain cognitive function. Staying socially active, avoiding intake of harmful substances, getting enough sleep, a healthy diet, and challenging the brain to keep its connections active and create new ones also help to keep the brain healthy

What accurately describes prostate cancer detection and/or treatment (Select all that apply) A. The symptoms of pelvic or perineal pain, fatigue, and malaise may be present B. Palpation of the prostate reveals hard and asymmetric enlargement with areas of induration or nodules C. Orchiectomy is a treatment option for all patients with prostatic cancer except those with stage IV tumors D. The preferred hormonal therapy for treatment of prostate cancer includes estrogen and androgen receptor blockers E. Early detection of cancer of the prostate is increased with annual rectal examinations and serum prostatic acid phophatase (PAP) measurements F. An annual prostate examination is recommended starting at age 45 for African American men with a first degree relative with prostate cancer at an early age

A.B.F. The symptoms of pelvic or perineal pain, fatigue, and malaise may be present; Palpation of the prostate reveals hard and asymmetric enlargement with areas of induration or nodules; An annual prostate examination is recommended starting at age 45 for African American men with a first degree relative with prostate cancer at an early age Rationale: Pelvic or perineal pain, fatigue, malaise, and a hard asymmetric, enlarged prostate may be present with prostate cancer. Annual prostate examination is recommended starting at a younger age for African American men because of increased diagnosis and mortality from prostate cancer in this ethnic group. An orchiectomy may be done with prostatectomy or for metastatic stages of prostate cancer. Hormonal treatment includes androgen deprivation therapy, luteinizing hormone releasing agonists, and androgen receptor blockers. Early detection of prostate cancer is best detected with annual rectal exams and serum PSA. Elevated prostatic acid phosphatase (PAP) will be seen with metastasis, not a new diagnosis

A patient who has been treated for status epilepticus in the emergency department will be transferred to the medical nursing unit. Which equipment should the nurse have available in the patient's assigned room (Select all that apply)? A. Side rail pads B. Tongue blade C. Oxygen mask D. Suction tubing E. Urinary catheter F. Nasogastric tube

A.C.D. Side rail pads, Oxygen mask, Suction tubing Rationale: The patient is at risk for further seizures, and O2 and suctioning may be needed after any seizures to clear the airway and maximize oxygenation. The bed's side rails should be padded to minimize the risk for patient injury during a seizure. Use of tongue blades during a seizure in contraindicated. Insertion of a nasogastric (NG) tube is not indicated because the airway problem is not caused by vomiting or abdominal distention. A urinary catheter is not required unless there is urinary retention

What factors should be considered as priorities when taking the history of a patient with a neurologic problem (Select all that apply)? A. Avoid suggesting symptoms B. Include the CN assessment as the first assessment C. Mental status must be accurately assessed to ensure that the report history is factual D. Do a focused assessment of the neurologic system, as other body systems will not be affected E. The mode of onset and course of illness are especially important aspects of the nursing history

A.C.E. Avoid suggesting symptoms; Mental status must be accurately assessed to ensure that the report history is factual; The mode of onset and illness are especially important aspects of the nursing history Rationale: When taking the history of a patient with a neurologic problem, avoid suggesting symptoms or asking leading questions. The mode of onset and course of illness are especially important. Validate the history if the patient's mental status causes question as to the reliability of the history. The other options are part of the physical assessment and will depend on the patient's history and manifestations

Which factors decrease cerebral blood flow (select all that apply)? A. Increased ICP B. PaO2 of 45mmHg C. PaCO2 of 30mmHg D. Arterial blood pH of 7.3 E. Decreased mean arterial pressure (MAP)

A.C.E. Increased ICP; PaCO2 of 30mmHg; Decreased mean arterial pressure (MAP) Rationale: Cerebral blood flow is decreased when the MAP and the PaCO2 are decreased and ICP is increased. The other options increase cerebral blood flow

Significant information about a person's health history related to the reproductive system should include (Select all that apply) A. tobacco use B. intellectual status C. current pain level D. previous history of shingles E. previous sexually transmitted infections

A.C.E. tobacco use, current pain level, previous sexually transmitted infections Rationale: Tobacco use can affect the reproductive health of both men and women, including effects on fertility, birth outcomes, cancer, risk of pulmonary embolism for women on contraceptives. It is important to assess for pain and the intensity of pain as well as any previous STI.

Which factors would place a patient at a higher risk for prostate cancer? (Select all that apply) A. Older than 65 years B. Asian or Native American C. Long term use of an indwelling urethral catheter D. Father diagnosed and treated for early stage prostate cancer E. Previous history of undescended testicle and testicular cancer

A.D. Older than 65 years; Father diagnosed and treated for early stage prostate cancer Rationale: Age, ethnicity, and family history are risk factors for prostate cancer. The incidence of prostate cancer rises markedly after age 50, and more than 66% of men with this diagnosis are older than 65 years. The incidence of prostate cancer worldwide is higher in African Americans than in any other ethnic group (except Jamaican men of African descent). A family history of prostate cancer, especially cancer in first degree relatives (e.g. fathers, brothers), is associated with an increased risk

A 39 year old patient with a history of IV drug use is seen at a community clinic. The patient reports difficulty walking, stating, "I don't know where my feet are." Diagnostic screening reveals positive Venereal Disease Research Laboratory (VDRL) and fluorescent treponemal antibody absorption (FTA-Abs) test results. Based on the patient history, what will the nurse assess? (select all that apply) A. Heart sounds B. Genitalia for lesions C. Joints for swelling and inflammation D. Mental state for judgement and orientation E. Skin and mucous membranes for gummas

A.D.E. Heart sounds; mental state for judgement and orientation; skin and mucous membranes for gummas Rationale: The patient's clinical manifestations and laboratory tests are consistent with tertiary syphilis. Valvular insufficiency, gummas, and changes in mentation are other clinical manifestations of this stage

What manifestations of cognitive impairment are primarily characteristic of delirium (select all that apply)? A. Reduced awareness B. Impaired judgments C. Words difficult to find D. Sleep/wake cycle reversed E. Distorted thinking and perception F. Insidious onset with prolonged duration

A.D.E. Reduced awareness; Sleep/wake cycle reversed; Distorted thinking and perception Rationale: Manifestations of delirium include cognitive impairment with reduced awareness, reversed sleep/wake cycle, and distorted thinking and perception. The other options are characteristic of dementia

During assessment of the patient with trigeminal neuralgia, the nurse should (select all that apply) A. inspect all the aspects of the mouth and teeth B. assess the gag reflex and respiratory rate and depth C. lightly palpate the affected side of the face for edema D. test for temperature and sensation perception on the face E. ask the patient to describe factors that initiate an episode

A.D.E. inspect all the aspects of the mouth and teeth; test for temperature and sensation perception of the face; ask the patient to describe factors that initiate an episode Rationale: Assessment of the attacks, including the triggering factors, characteristics, frequency, and pain management techniques, helps the nurse plan for patient care. Painful episodes re usually triggered by light touch at a specific point (i.e. trigger zone) along the distribution of the nerve branches. Precipitating stimuli include chewing, tooth brushing, a hot or cold blast of air on the face, washing the face, yawning or talking. Touch and tickle seem to predominate as causative triggers, rather than pain or changes in ambient temperature. The nursing assessment should include the patient's nutritional status, hygiene (especially oral), and behavior (including withdrawal). As a result of the attacks, the patient may eat improperly, neglect hygienic practices, wear a cloth over the face, and withdraw from interaction with others

A patient is diagnosed with Bell's palsy. What information should the nurse teach the patient about Bell's palsy (select all that apply)? A. Bell's palsy affects the motor branches of the facial nerve B. Antiseizure drugs are the drugs of choice for treatment of Bell's palsy C. Nutrition and avoidance of hot foods or beverages are special needs of this patient D. Herpes simplex virus 1 is strongly associated as a precipitating factor in the development of Bell's palsy E. Moist heat, gentle massage, electrical stimulation of the nerve and exercises are prescribed to treat Bell's palsy F. An inability to close the eyelid, with an upward movement of the eyeball when closure is attempted, is evident

A.D.E.F. Bell's palsy affects the motor branches of the facial nerve; Herpes simplex virus 1 is strongly associated as a precipitating factor in the development of Bell's palsy; Moist heat, gentle massage, electrical stimulation of the nerve and exercises are prescribed to treat Bell's palsy; An inability to close the eyelid, with an upward movement of the eyeball when closure is attempted, is evident Rationale: Bell's palsy affects the motor branches of the facial nerve. Herpes simplex virus 1 or herpes zoster virus may be a precipitating factor. Moist heat, gentle massage, electrical nerve stimulation, and exercises are prescribed. Care must be taken to protect the eye with sunglasses, artificial tears or gel, and possibly taping the eyelid closed at night. Bell's palsy is treated with corticosteroids, usually prednisone, not antiseizure drugs. Oral hygiene is important, but avoidance of hot foods is not needed

Which characteristics describe the anticoagulant warfarin (Coumadin) (select all that apply)? A. Vitamin K is the antidote B. Protamine sulfate is the antidote C. My be administered orally or subcutaneously D. May be administered intravenously or subcutaneously E. Dosage monitored using international normalized ratio (INR) F. Dosage monitored using activated partial thromboplastin time (aPTT)

A.E. Vitamin K is the antidote; Dosage monitored using international normalized ratio (INR) Rationale: Warfarin (Coumadin) is a vitamin K antagonist, so vitamin K is the antidote. It is monitored with the international normalized ration (INR). It is only administered orally. Protamine sulfate is the antidote for unfractionated heparin (UH) and low molecular weight heparin (LMWH). UH can be administered subcutaneously or IV and is monitored with activated partial thromboplastin time (aPTT). Hirudin derivatives are given IV or subcutaneously, do not have an antidote, and are also monitored with aPTT. Argatroban, a synthetic thrombin inhibitor, is given only IV and is monitored with thrombin inhibitor, is given only IV and is monitored with aPTT. Factor Xa inhibitor fondaparinux (Arixtra) is given subcutaneously and does not require routine coagulation testing. Rivaroxaban (Xarelto), another factor Xa inhibitor is given orally

A patient with a venous thromboembolism (VTE) is started on enoxaparin (Lovenox) and warfarin (Coumadin). The patient asks the nurse why two medications are necessary. Which response by the nurse is most appropriate? A. "Taking two blood thinners reduces the risk for another clot to form." B. "Enoxaparin will work right away, but Coumadin takes several days to have an effect on preventing clots." C. "Enoxaparin will start to dissolve the clot, and Coumadin will prevent any more clots from forming." D. "Because of the risk for a blood clot in the lungs, it is important for you to take more than one blood thinner."

B. "Enoxaparin will work right away, but Coumadin takes several days to have an effect on preventing clots." Rationale: Low molecular weight heparin (LMWH) is used because of the immediate effect on coagulation and discontinued once the international normalized ratio (INR) value indicates that the warfarin has reached a therapeutic level. LMWH has not thrombolytic properties. The use of two anticoagulants is not related to the risk for pulmonary embolism, and two are not necessary to reduce the risk for another VTE. Anticoagulants do not thin the blood

The nurse has started discharge teaching for a patient who is to continue warfarin (Coumadin) following hospitalization for venous thromboembolism (VTE). The nurse determines that additional teaching is needed when the patient says which of the following? A. "I should get a Medic Alert device stating that I take Coumadin." B. "I should reduce the amount of green, leafy vegetables that I eat." C. "I will need routine blood tests to monitor the effects of the Coumadin." D. "I will check with my health care provider before I begin any new medications."

B. "I should reduce the amount of green, leafy vegetables that I eat." Rationale: Patients taking warfarin are taught to follow a consistent diet with regard to foods that are high in vitamin K, such as green, leafy vegetables. The other patient statements are accurate

While working in the outpatient clinic, the nurse notes that a patient has a history of intermittent claudication. Which statement by the patient would support this information? A. "When I stand too long, my feet start to swell." B. "My legs cramp when I walk more than a block." C. "I get short of breath when I climb a lot of stairs." D. "My fingers hurt when I go outside in cold weather."

B. "My legs cramp when I walk more than a block." Rationale: Cramping that is precipitated by a consistent level of exercise is descriptive of intermittent claudication. Finger pain associated with cold weather is typical of Raynaud's phenomenon. Shortness of breath that occurs with exercise is not typical of intermittent claudication, which is reproducible. Swelling associated with prolonged standing is typical of a venous disease

A 20 year old patient with PID is crying and tells the nurse that she is afraid she will not be able to have children as a result of the infection. What is the nurse's best response to this patient? A. "I would not worry about that now. Our immediate concern is to cure the infection you have." B. "PID increases the possibility of infertility. Would you like to talk about what it means to you?" C."Sterility following PID is possible but not common, and it is too soon to now what the effects will be." D. "The infection can cause more serious complications, such as abscesses and shock hat you should be more concerned about."

B. "PID increases the possibility of infertility. Would you like to talk about what it means to you?" Rationale: The risk for infertility following PID is high, and the nurse should allow time for the patient to express her feelings, clarify her concerns, and begin problem solving with regard to outcomes of the disease. Responses that do not allow for discussion of feelings and concerns and that tell the patient how she should feel or what she should worry about are not therapeutic

Family members of a patient who has a traumatic brain injury ask the nurse about the purpose of the ventriculostomy system being used for intracranial pressure monitoring. Which response by the nurse is best? A. "This type of monitoring system is complex and it is managed by skilled staff." B. "The monitoring system helps show whether blood flow to the brain is adequate." C. "The ventriculostomy monitoring system helps check for alterations in cerebral perfusion pressure." D. "This monitoring system has multiple benefits including facilitation of cerebrospinal fluid drainage."

B. "The monitoring system helps show whether blood flow to the brain is adequate." Rationale: Short and simple explanations should be given initially to patients and family members. The other explanations are either too complicated to be easily understood or may increase the family members' anxiety

A 34 year old patient who is discussing contraceptive options with the nurse says, "I want to have children but not for a few years." Which response by the nurse is accurate? A. "If you do not become pregnant within the next few years, you never will." B. "Women often have more difficulty becoming pregnant after about age 35." C. "Stop taking oral contraceptives several years before you want to have a child." D. "You have many more years of fertility left, so there is no rush to have children."

B. "Women often have more difficulty becoming pregnant after about age 35." Rationale: The probability of successfully becoming pregnant decreases after age 35 years, although some patients may have no difficulty in becoming pregnant. Oral contraceptives do not need to be withdrawn for several years for a woman to become pregnant. Although the patient may be fertile for many years, it would be inaccurate to indicate that there is no concern about fertility as she becomes older. Although the risk of infertility increases after age 35 years, not all patients have difficulty in conceiving

The patellar tendon is struck and the leg extends with contraction of the quadriceps. What grade should this response be given? A. 1/5 B. 2/5 C. 3/5 D. 4/5

B. 2/5 Rationale: This grade is a 2/5 as it is a normal patellar reflex response. Deep tendon grading is as follows: 0/5= absent; 1/5= weak response; 2/5= normal response; 3/5= brisk response; 4/5= hyperreflexia with nonsustained clonus; 5/5= hyperrelexia with sustained clonus

The patient is being monitored long term with a brain tissue oxygenation catheter. What range for the pressure of oxygen in brain tissue (PbtO2) will maintain cerebral oxygen supply and demand? A. 55%-75% B. 20 to 40 mmHg C. 70 to 150 mmHg D. 80 to 100 mmHg

B. 20 to 40 mmHg Rationale: The normal pressure of oxygen in brain tissue (PbtO2) is 20 to 40mmHg. The normal jugular venous oxygen saturation (SjvO2) is 55%-75% and indicates total venous brain tissue extraction of oxygen; this is used for short term monitoring. The MAP of 70 to 150mmHg is needed for effective autoregulation of CBF. The normal range for PaO2 is 80 to 100mmHg.

The patient at highest risk for venous thromboembolism (VTE) is A. A 62 year old man with spider veins who is having arthroscopic knee surgery B. A 32 year old woman who smokes, takes oral contraceptives, and is planning a trip to Europe C. A 26 year old woman who is 3 days postpartum and received maintenance IV fluids for 12 hours during her labor D. An active 72 year old man at home recovering from transurethral resection of the prostate for benign prostatic hyperplasia

B. A 32 year old woman who smokes, takes oral contraceptives, and is planning a trip to Europe Rationale: Three important factors (called Virchow's triad) in the etiology of venous thrombosis are (1) venous stasis, (2) damage of the endothelium (inner lining of the vein), and (3) hypercoagulability of the blood. Patients at risk for venous thrombosis usually have predisposing conditions for these three disorders. The 32 year old woman has the highest risk: long road trips without adequate exercise (venous stasis), tobacco use, and use of oral contraceptives. NOTE: The likelihood of hypercoaguability of blood is increased in women older than 35 years who use tobacco

The nurse in the clinic notes elevated prostate specific antigen (PSA) levels in the laboratory results of these patients. Which patient's PSA result is most important to report to the health care provider? A. A 38-year-old who is being treated for acute prostatitis B. A 48-year-old whose father died of metastatic prostate cancer C. A 52-year-old who goes on long bicycle rides every weekend D. A 75-year-old who uses saw palmetto to treat benign prostatic hyperplasia (BPH)

B. A 48-year-old whose father died of metastatic prostate cancer Rationale: The family history of prostate cancer and elevation of the patient for prostate cancer is needed. Elevations in PSA for the other patients are not unusual.

After receiving change of shift report, which patient admitted to the emergency department should the nurse assess first? A. A 67 year old patient who has a gangrenous foot ulcer with a weak pedal pulse B. A 50 year old patient who is complaining of sudden sharp and severe back upper back pain C. A 39 year old patient who has right calf tenderness, redness, and swelling after a plane ride D. A 58 year old patient who is taking anticoagulants for atrial fibrillation and has black stools

B. A 50 year old patient who is complaining of sudden sharp and severe back upper back pain Rationale: The patient's presentation of sudden sharp and severe upper back pain is consistent with dissecting throacic aneurysm , which will require the most rapid intervention. The other patients also require rapid intervention but do not before the patient with severe pain

The nurse in a health clinic receives requests for appointments from several patients. Which patient should be seen by the health care provider first? A. A 48-year-old man who has perineal pain and a temperature of 100.4° F B. A 58-year-old man who has a painful erection that has lasted over 6 hours C. A 38-year-old man who states he had difficulty maintaining an erection last night D. A 68-year-old man who has pink urine after a transurethral resection of the prostate (TURP) 3 days ago

B. A 58-year-old man who has a painful erection that has lasted over 6 hours Rationale: Priapism can cause complications such as necrosis or hydronephrosis, and this patient should be treated immediately. The other patients do not require immediate action to prevent serious complications

Several patients have been hospitalized for diagnosis of neurologic problems. Which patient will the nurse assess first? A. A patient with a transient ischemic attack (TIA) returning from carotid duplex studies B. A patient with a brain tumor who has just arrived on the unit after a cerebral angiogram C. A patient with a seizure disorder who has just completed an electroencephalogram (EEG) D. A patient prepared for a lumbar puncture whose health care provider is waiting for assistance

B. A patient with a brain tumor who has just arrived on the unit after a cerebral angiogram Rationale: Because cerebral angiograms require insertion of a catheter into the femoral artery, bleeding is a possible complication. The nurse will need to check the pulse, blood pressure, and the catheter insertion site in the groin as soon as the patient arrives. Carotid duplex studies and EEG are noninvasive. The nurse will need to assist with the lumbar puncture as soon as possible, but monitoring for hemorrhage after cerebral angiogram has a higher priority

A patient with stage 0 cervical cancer identified from a Papanicolaou (Pap) test asks the nurse what this finding means. The nurse's response should include which information? A. Malignant cells have extended beyond the cervix to the upper vagina B. Abnormal cells are present but are confined to the epithelial layer of the cervix C. Atypical cells characteristic of inflammation but not necessarily malignancy are present D. This is a common finding on Pap testing, and she will be examined frequently to see whether the abnormal cells spread beyond the cervix

B. Abnormal cells are present but are confined to the epithelial layer of the cervix Rationale: A stage 0 cervical cancer indicates cancer is situ that is confined to the epithelial layer of the cervix and requires treatment. Stage 0 is the least invasive. Stage I is confined to the cervix. Stage II has spread beyond the cervix to the upper two thirds of the vagina but not the tissues around the uterus. Stage III involves the pelvic wall, lower third of the vagina, and/or kidney problems. Stage IV indicates spread to distant organs

A hospitalized patient complains of a bilateral headache (4/10 on the pain scale) that radiates from the base of the skull. Which prescribed PRN medications should the nurse administer initially? A. Lorazepam (Ativan) B. Acetaminophen (Tylenol) C. Morphine sulfate (MS Contin) D. Butalbital and aspirin (Fiorinal)

B. Acetaminophen (Tylenol) Rationale: The patient's symptoms are consistent with a tension headache, and initial therapy usually involves a nonopioid analgesic such as acetaminophen, which is sometimes combined with a sedative or muscle relaxant. Lorazepam may be used in conjunction with acetaminophen but would not be appropriate as the initial monotherapy. Morphine sulfate and butalbital and aspirin would be more appropriate for a headache that did not respond to a nonopioid analgesic

The wife of a patient who is manifesting deterioration in memory asks the nurse whether her husband has AD. The nurse explains that a diagnosis of AD is usually made when what happens? A. A urine test indicates elevated levels of isoprostanes B. All other possible causes of dementia have been eliminated C. Blood analysis reveals increased amounts of B-amyloid protein D. A computed tomography (CT) scan of the brain indicates atrophy

B. All other possible causes of dementia have been eliminated Rationale: The only definitive diagnosis of AD can be made on examination of brain tissue during an autopsy, but a clinical diagnosis is made when all other possible causes of dementia have been eliminated. Patients with AD may have B-Amyloid proteins in the blood, brain atrophy, or isoprostanes in the urine, but these findings are not exclusive to those with AD

Following a generalized tonic-clonic seizure, the patient is tired and sleepy. What care should the nurse provide? A. Suction the patient before allowing him to rest. B. Allow the patient to sleep as long as he feels sleepy. C. Stimulate the patient to increase his level of consciousness. D. Check the patient's level of consciousness every 15 minutes for an hour.

B. Allow the patient to sleep as long as he feels sleepy. Rationale: In the postictal phase of generalized tonic clonic seizures, patients are usually very tired and may sleep for several hours and the nurse should allow the patient to sleep as long as necessary. Suctioning is performed only if needed, and decreased level of consciousness is not a problem postictally unless a head injury has occurred during the seizure

A patient with a seizure disorder is being evaluated for surgical treatment of the seizures. The nurse recognizes that what is one of the requirements for surgical treatment? A. Identification of scar tissue that is able to be removed B. An adequate trial of drug therapy that had unsatisfactory results C. Development of toxic syndromes from long term use of antiseizure drugs D. The presence of symptoms of cerebral degeneration from repeated seizures

B. An adequate trial of drug therapy that had unsatisfactory results Rationale: Most patients with seizure disorders maintain seizure control with medications, but if surgery is considered, three requirements must be met: the diagnosis of epilepsy must be confirmed, there must have been an adequate trial with drug therapy without satisfactory results, and a defined electroclinical syndrome. The focal point must be localized, but the presence of scar tissue is not required

Which actions could the nurse delegate to unlicensed assistive personnel (UAP) who are providing care for a patient who is at risk for venous thromboembolism? A. Monitor for any bleeding after anticoagulation therapy is started. B. Apply sequential compression device whenever the patient is in bed. C. Ask the patient about use of herbal medicines or dietary supplements. D. Instruct the patient to call immediately if any shortness of breath occurs.

B. Apply sequential compression device whenever the patient is in bed. Rationale: UAP training includes the use of equipment that requires minimal nursing judgement, such as sequential compression devices. Patient assessment and teaching require more education and critical thinking and should be done by the registered nurse (RN)

A 58 year old patient with erectile dysfunction (ED) tells the nurse that he is interested in using sildenafil (Viagra). Which action should the nurse take first? A. Assure the patient that ED is common with aging B. Ask the patient about any prescription drugs he is taking C. Tell the patient that Viagra does not always work for ED D. Discuss the common adverse effects of erectogenic drugs

B. Ask the patient about any prescription drugs he is taking Rationale: Because some medications can cause ED and patients using nitrates should not take sildenafil, the nurse should first assess for prescription drug use. The nurse may want to teach the patient about realistic expectations and adverse effects of sildenafil therapy, but this should not be the first action. Although ED does increase with aging, it may be secondary to medication use or cardiovascular disease.

A 27 year old patient who has testicular cancer is being admitted for a unilateral orchiectomy. The patient does not talk to his wife and speaks to the nurses only to answer the admission questions. Which action is appropriate for the nurse to take? A. Teach the patient and the wife that impotence is unlikely after unilateral orchiectomy B. Ask the patient if he has any questions or concerns about the diagnosis and treatment C. Inform the patient's wife that concerns about sexual function are common with this diagnosis D. Document the patient's lack of communication on the health record and continue preoperative care

B. Ask the patient if he has any questions or concerns about the diagnosis and treatment Rationale: The initial action by the nurse should be assessment for any anxiety or questions about the surgery or postoperative care. The nurse should address the patient, not the spouse, when discussing the diagnosis and any possible concerns. Without further assessment of patient concerns, the nurse should not offer teaching about complications after orchiectomy. Documentation of the patient's lack of interaction is not an adequate nursing action in this situation

A patient with continuous bladder irrigation following a prostatectomy tells the nurse that he has bladder spasms and leaking of urine around the catheter. What should the nurse do first? A. Slow the rate of the irrigation B. Assess the patency of the catheter C. Encourage the patient to try to urinate around the catheter D. Administer a belladonna and opium (B&O) suppository as prescribed

B. Assess the patency of the catheter Rationale: the nurse should first check for the presence of clots obstructing the catheter or tubing and remove them by irrigation. Then a belladonna and opium suppository is administered, if one is ordered. The flow rate of the irrigation fluid may be decreased if orders permit because fast flowing, cold fluid may also contribute to spasms. The patient should not try to void around the catheter because this will increase the spasms

On admission of a victim of sexual assault to the emergency department, what should be the first priority of the nurse? A. Contact a rape support person for the patient B. Assess the patient for urgent medical problems C. Question the patient about the details of the assault D. Inform the patient what procedures and treatments will be performed

B. Assess the patient for urgent medical problems Rationale: Sexual assault is an act of violence and the first priority of care for the patient should be assessment and treatment of serious injuries involving extragenital areas, such as fractures, subdural hematomas, cerebral concussions, and intraabdominal injuries. All the other options as well as preserving forensic evidence are appropriate treatments, but treatment for shock and urgent medical injuries is the first priority

A patient who has amyotrophic lateral sclerosis (ALS) is hospitalized with pneumonia. Which nursing action will be included in the plan of care? A. Observe for agitation and paranoia B. Assist with active range of motion (ROM) C. Give muscle relaxants as needed to reduce spasms D. Use simple words and phrases to explain procedures

B. Assist with active range of motion (ROM) Rationale: ALS causes progressive muscle weakness, but assisting the patient to perform active ROM will help maintain strength as long as possible. Psychotic manifestations such as agitation and paranoia are not associated with ALS. Cognitive function is not affected by ALS, and the patient's ability to understand procedures will not be impaired. Muscle relaxants will further increase muscle weakness and depress respirations

During the patient's acute postoperative period following repair of an AAA, the nurse should ensure that which goal is achieved? A. Hypothermia is maintained to decrease oxygen need B. BP and all peripheral pulses are evaluated at least every hour C. IV fluids are administered at a rate to maintain urine output of 100mL/hr D. The patient's BP is kept lower than baseline to prevent leaking at the incision line

B. BP and all peripheral pulses are evaluated at least every hour Rationale: The BP and peripheral pulses are evaluated every hour in the acute postoperative period to ensure that BP is adequate to maintain graft patency and that extremities are being perfused. BP is kept within normal range. If BP is too low, thrombosis of the graft may occur, if it is too high, it may cause leaking or rupture at the suture line. Hypothermia is induced during surgery but the patient is rewarmed as soon as surgery is completed. Fluid replacement to maintain urine output at 100mL/hr would increase the BP too much and only 30mL/hr of urine is needed to show adequate renal perfusion

After reviewing the electronic medical record shown in the accompanying figure for a patient who had transurethral resection of the prostate the previous day, which information requires the most rapid action by the nurse? History: Lower urinary tract symptoms for the last nine months, takes diuretic and beta-blocker for hypertension, antihypertensive drugs not prescribed after surgery Vital Signs: Temperature 99.0, Pulse 94 beats/minute, respirations 24 breaths/minute, BP 168/88 mmHg Physical assessment: Crackles heard at lung bases, reports frequent bladder spasms, nor urine draining from triple lumen catheter A. elevated temperature and pulse B. Bladder spasms and urine output C. Respiratory rate and lung crackles D. No prescription for antihypertensive drugs

B. Bladder spasms and urine output Rationale: Bladder spasms and lack of urine output indicate that the nurse needs to assess the continuous bladder irrigation for kinks and may need to manually irrigate the patient's catheter. The other information will also require actions, such as having the patient take deep breaths and cough and discussing the need for antihypertensive medication prescriptions with the health care provider, but the nurse's first action should be to address the problem with the urinary drainage system

When using intraventricular ICP monitoring, what should the nurse be aware of to prevent inaccurate readings? A. The P2 wave is higher than the P1 wave. B. CSF is leaking around the monitoring device. C. The stopcock of the drainage device is open to drain the CSF fluid D. The transducer of the ventriculostomy monitor is at the level of the upper ear.

B. CSF is leaking around the monitoring device. Rationale: An inaccurate ICP reading can be caused by CSF leaks around the monitor device, obstruction of the intraventricular catheter, kinks or bubbles in the tubing, and incorrect height of the transducer or drainage system relative to the patient's reference point, the tragus of the ear (cartilage projection anterior to the opening of the ear). The P2 wave being higher than the P1 wave indicates poor ventricular compliance. The drain of CSF drainage device should be closed for 6 minutes preceding the reading

What is the purpose of the dendrite? A. Provides gap in peripheral nerve axons B. Carries impulses to the nerve cell body C. Carries impulses from the nerve cell body D. Helps to repair damage to peripheral axons

B. Carries impulses to the nerve cell body Rationale: The dendrite carries impulses to the nerve cell body. The gap in the peripheral nerve axons is the node of Ranvier that allows an action potential to travel faster by jumping from node to node without transversing the insulated membrane segment. The axon carries impulses from the nerve cell body. Regeneration may occur with damage to peripheral axons

Which statement(s) accurately describe(s) mild cognitive impairment (select all that apply)? A. Always progresses to AD B. Caused by a variety of factors and many progress to AD C. Should be aggressively treated with acetycholinesterase drugs D. Caused by vascular infarcts that, if treated, will delay progression to AD E. Patient is usually not aware that there is a problem with his or her memory

B. Caused by a variety of factors and many progress to AD Rationale: Although some individuals with mild cognitive impairment (MCI) revert to normal cognitive function or do not go on to develop Alzheimer's disease (AD), those with MCI are at high risk for AD. No drugs have been approved fro the treatment of MCI. A person with MCI is often aware of a significant change in memory

What is the neurologic diagnostic test that has the highest risk of complications and requires frequent monitoring of neurologic and vital signs following the procedure? A. Electromyelogram B. Cerebral angiography C. Electroencephalogram D. Transcranial Doppler sonography

B. Cerebral angiography Rationale: Cerebral angiography involves the injection of contrast media through a catheter inserted into the femoral or brachial artery and passed into the base of a carotid or vertebral artery and is performed when vascular lesions or tumors are suspected. Allergic reactions to the contrast medium may occur, and vascular spasms or dislodgement of plaques is possible. Neurologic status and vital signs must be monitored every 15 to 30 minutes for 2 hours, every hour for the next six hours, and then every 2 hours for 24 hours following the test. Electromyelography, EEG and transcranial Doppler sonography are not invasive studies

A 20 year old male patient is admitted with a head injury after a collision while playing football. After noting that the patient has developed clear nasal drainage, which action should the nurse take? A. Have the patient gently blow the nose B. Check the drainage for glucose content C. Teach the patient that rhinorrhea is expected after a head injury D. Obtain a specimen of the fluid to send for culture and sensitivity

B. Check the drainage for glucose content Rationale: Clear nasal drainage in a patient with a head injury suggests a dural tear and cerebrospinal fluid (CSF) leakage. If the drainage is CSF, it will test positive for glucose. Fluid leaking from the nose will have normal nasal flora, so culture and sensitivity will not be useful. Blowing the nose is avoided to prevent CSF leakage.

A patient who is postoperative following repair of an AAA has been receiving IV fluids at 125mL/hr continuously for the last 12 hours. Urine output for the last 4 hours has been 60mL, 42mL 28mL, and 20mL, respectively. What is the priority action that the nurse should take? A. Monitor for a couple more hours B. Contact the HCP and report the decrease in urine output C. Send blood for electrolytes, blood urea nitrogen (BUN), and creatinine D. Decrease the rate of infusion to prevent blood leakage at the suture line

B. Contact the HCP and report the decrease in urine output Rationale: The decreasing urine output is evidence that either the present needs volume replacement or there is reduced renal blood flow. The HCP will want to be notified as soon as possible of this change in condition and will request results of daily blood urea nitrogen (BUN) and serum creatinine levels. The other options are incorrect

During the diagnosis and long-term management of a seizure disorder, what should the nurse recognize as one of the major needs of the patient? A. Managing the complicated drug regimen of seizure control B. Coping with the effects of negative social attitudes toward epilepsy C. Adjusting to the very restricted lifestyle required by a diagnosis of epilepsy D. Learning to minimize the effect of the condition in order to obtain employment

B. Coping with the effects of negative social attitudes toward epilepsy Rationale: One of the most common complications of a seizure disorder is the effect it has on the patient's lifestyle. This is because of the social stigma attached to seizures, which causes patient to hide their diagnosis and to prefer not to be identified as having epilepsy. Medication regimens usually require only once or twice daily dosing and the major restrictions of lifestyle usually involve driving and high risk environments. Job discrimination against the handicapped is prevented by federal and state laws and patients only need to identify their disease in case of medical emergencies

A 58 year old man is being assessed by the nurse. Which information would identify the need for further examination for benign prostatic hyperplasia? A. A mass on the scrotum or testes B. Difficulty starting a slow urinary stream C. Patient describes a single, small, painless blister D. A bulging inguinal ring while the patient bears down

B. Difficulty starting a slow urinary stream Rationale: A slow and difficult to start urinary stream indicates a need for further examination for benign prostatic hyperplasia (BPH). A mass could indicate cancer or other scrotal problems. A single, painless, small blister could indicate lymphogranuloma venereum or cancer. Palpating a bulging inguinal ring whikle the patient bears down is indicative of inguinal hernia

What is one focus of interprofessional care of patients with AD? A. Replacement of deficient acetylcholine in the brain B. Drug therapy for cognitive problems and undesirable behaviors C. The use of memory enhancing techniques to delay disease progression D. Prevention of other chronic diseases that hasten the progression of AD

B. Drug therapy for cognitive problems and undesirable behaviors Rationale: Because there is no cure for AD, collaborative management is aimed at controlling the undesirable manifestations that the patient may exhibit, and providing support for the family caregiver. Cholinesterase inhibitors help to increase acetylcholine (ACh) in the brain, but a variety of other drugs are also used to control behavior. Memory enhancing techniques have little or no effect in patients with AD, especially as the disease progresses. Patient with AD have limited ability to communicate health symptoms and problems, leading to a lack of professional attention for acute and other chronic illnesses

Which action will the public health nurse take to reduce the incidence of epidemic encephalitis in a community? A. Teach about prophylactic antibiotics after exposure to encephalitis B. Encourage the use of effective insect repellant during mosquito season C. Remind patients that most cases of viral encephalitis can be cared for at home D. Arrange to screen school age children for West Nile virus during the school year

B. Encourage the use of effective insect repellant during mosquito season Rationale: Epidemic encephalitis is usually spread by mosquitos and ticks. Use of insect repellant is effective in reducing risk. Encephalitis frequently requires that the patient be hospitalized in an intensive care unit during the initial stages. Antibiotic prophylaxis is not used to prevent encephalitis because most encephalitis is viral. West Nile virus is most common in adults over age 50 during the summer and early fall

After endotracheal suctioning, the nurse notes that the intracranial pressure for a patient with a traumatic head injury has increased from 14 to 17 mm Hg. Which action should the nurse take first? A. Document the increase in intracranial pressure. B. Ensure that the patient's neck is in neutral position. C. Notify the health care provider about the change in pressure. D. Increase the rate of the prescribed propofol (Diprivan) infusion.

B. Ensure that the patient's neck is in neutral position. Rationale: Because suctioning will cause a transient increase in ICP, the nurse should initially check for factors that might be contributing to the increase and observe the patient for a few minutes. Documentation is needed, but this is not the first action. There is no need to notify the health care provider about this expected reaction to suctioning. Propofol is used to control patient anxiety or agitation. There is no indication that anxiety has contributed to the increase in ICP

A high school teacher who has been diagnosed with epilepsy after having a generalized tonic clonic seizure tells the nurse, "I cannot teach any more. It will be too upsetting if I have a seizure at work." Which response by the nurse specifically addresses the patient's concern? A. You might benefit from some psychologic counseling B. Epilepsy usually can be well controlled with medications C. You will want to contact the Epilepsy foundation for assistance D. The Department fo Vocational Rehabilitation can help with work retraining

B. Epilepsy usually can be well controlled with medications Rationale: The nurse should inform the patient that most patients with seizure disorders are controlled with medication. The other information may be necessary if the seizures persist after treatment with antiseizure medications is implemented

A patient with moderate AD has a nursing diagnosis of impaired memory related to effects of dementia. What is an appropriate nursing intervention for this patient? A. Post clocks and calendars in the patient's environment B. Establish and consistently follow a daily schedule with the patient C. Monitor the patient's activities to maintain a safe patient environment D. Stimulate thought processes by asking the patient questions about recent activities

B. Establish and consistently follow a daily schedule with the patient Rationale: Adhering to a regular, consistent daily schedule helps the patient to avoid confusion and anxiety and is important both during hospitalization and at home. Clocks and calendars may be useful in early AD, but they have little meaning to a patient as the disease progresses. Questioning the patient about activities and events they cannot remember is threatening and may cause severe anxiety. Maintaining a safe environment for the patient is important but does not change the disturbed thought processes

A patient who was admitted to the emergency department with severe abdominal pain is diagnosed with an ectopic pregnancy. The patient begins to cry and asks the nurse to leave her alone to grieve. Which action should the nurse take next? A. Stay with the patient and encourage her to discuss her feelings B. Explain the reason for taking vital signs every 15 to 30 minutes C. Close the door to the patient's room and minimize disturbances D. Provide teaching about options for termination of the pregnancy

B. Explain the reason for taking vital signs every 15 to 30 minutes Rationale: Because the patient is at risk for rupture of the fallopian tube and hemorrhage, frequent monitoring of vital signs is needed. The patient has asked to be left alone, so staying with her and encouraging her to discuss her feelings are inappropriate reactions. Minimizing contact with her and closing the door of the room is unsafe because of the risk for hemorrhage. Because the patient has requested time to grieve, it would be inappropriate to provide teaching about options for pregnancy termination

What is the normal response to striking the triceps tendon with a reflex hammer? A. Forearm pronation B. Extension of the arm C. Flexion of the arm at the elbow D. Flexion and supination of the elbow

B. Extension of the arm Rationale: The normal response of the triceps reflex is extension of the arm or visible contraction of the triceps. The normal response of the biceps reflex is flexion of the arm at the elbow. Flexion and supination at the elbow are seen with the presence of the brachioradialis reflex.

When the patient has a rapidly growing brain tumor, what part of the brain slows expansion of cerebral brain tissue into the adjacent hemisphere? A. Ventricles B. Falx cerebri C. Arachnoid layer D. Tentorium cerebella

B. Falx cerebri Rationale: The falx cerebri is a fold of the dura that separates the cerebral hemispheres and slows expansion of brain tissue. The ventricles are filled with and produce CSF. The arachnoid layer is a delicate membrane that lies next to the dura mater with the subarachnoid space between the arachnoid and pia mater. The tentorium cerebella is a fold of dura that separates the cerebral hemispheres from the posterior fossa that contains the brainstem and cerebellum

Which nursing action will be most effective in ensuring daily medication compliance for a patient with mild dementia? A. Setting the medications up monthly in a medication box B. Having the patient's family member administer the medication C. Posting the reminder to take the medications in the patient's house D. Calling the patient weekly with a reminder to take the medication

B. Having the patient's family member administer the medication Rationale: Because the patient with mild dementia will have difficulty with learning new skills and forgetfulness, the most appropriate nursing action is to have someone else administer the drug. The other nursing actions will not be as effective in ensuring that the patient takes the medications

In counseling patients with SCIs regarding sexual function, how should the nurse advise a male patient with a complete lower motor neuron lesion? A. He may have uncontrolled reflex erections, but orgasm and ejaculation are usually not possible. B. He is most likely to have reflex erections and may experience orgasm if S2-S4 nerve pathways are intact C. He has a lesion with the greatest possibility of successful psychogenic erection with ejaculation and orgasm D. He will probably be unable to have either psychogenic or reflexogenic erections and no ejaculation or orgasm

B. He is most likely to have reflex erections and may experience orgasm if S2-S4 nerve pathways are intact Rationale: Patients with a complete lower motor neuron lesion are able to have reflex erections and use drugs to maintain erection for sexual satisfaction if S2-S4 nerve pathways are intact. Patients with complete upper motor neuron lesions usually have only reflex sexual function with rare ejaculation. Patients with incomplete lower motor neuron lesions have the highest possiblity of successful psychogenic erections with ejaculation, whereas patients with incomplete upper motor neuron lesions may experience reflex erections with ejaculation

A patient is admitted to the emergency department (ED) with SCI at the level of T2. Which finding is of most concern to the nurse? A. SpO2 of 92% B. Heart rate of 42bpm C. Blood pressure of 88/60mmHg D. Loss of motor and sensory function in arms and legs

B. Heart rate of 42bpm Rationale: Neurogenic shock associate3d with SCI above the level of T6 greatly decreases the effect of the sympathetic nervous system and bradycardia and hypotension occur. A heart rate of 42bpm is not adequate to meet the oxygen needs of the body. While low, the blood pressure is not at a critical point. The oxygen saturation is satisfactory and the motor and sensory losses are expected

What are the key manifestations of bacterial meningitis? A. Papilledema and psychomotor seizures B. High fever, nuchal rigidity, and severe headache C. Behavioral changes with memory loss and lethargy D. Jerky eye movements, loss of corneal reflex, and hemiparesis

B. High fever, nuchal rigidity, and severe headache Rationale: High fever, severe headache, nuchal rigidity, nausea, and vomiting, are key signs of meningitis. Other symptoms, such as papilledema, generalized seizures, hemiparesis, and decreased LOC, and cranial nerve dysfunction may occur as complications of increased ICP in meningitis

A patient with paraplegia has developed an irritable bladder with reflex emptying. Along with possible use of medications, what will be most helpful for the nurse to teach the patient? A. Hygiene care for an indwelling urinary catheter B. How to perform intermittent self catheterization C. To empty the bladder with manual pelvic pressure in coordination with reflex voiding patterns D. That a urinary diversion, such as an ileal conduit, is the easiest way to handle urinary elimination

B. How to perform intermittent self catheterization Rationale: Intermittent self catheterization 4 to 6 times a day is the recommended method of bladder management for the patient with a SCI and reflex neurogenic bladder because it more closely mimics normal emptying and has less potential for infection. The patient and family should be taught the procedure using clean technique at home and if the patient has use of the arms, self catherization should be performed. Indwelling catheterization is used during the acute phase to prevent overdistention of the bladder. Surgical urinary diversions are used if urinary complications occur.

A patient with a T4 spinal cord injury experiences neurogenic shock as a result of sympathetic nervous system dysfunction. What would the nurse recognize as characteristic of this condition? A. Tachycardia B. Hypotension C. Increased cardiac output D. Peripheral vasoconstriction

B. Hypotension Rationale: Neurogenic shock results from loss of vasomotor tone caused by injury and is characterized by hypotension and bradycardia. Loss of sympathetic nervous system innervation causes peripheral vasodilation, venous pooling, and a decrease in cardiac output. These effects are usually associated with a cervical or high thoracic injury (T6 or higher)

Which statement by patient who is being discharged from the emergency department (ED) after a concussion indicates a need for intervention by the nurse? A. I will return if I feel dizzy or nauseated B. I am going to drive home and go to bed C. I do not even remember being in an accident D. I can take acetaminophen (Tylenol) for my headache

B. I am going to drive home and go to bed Rationale: After a head injury, the patient should avoid driving and operating heavy machinery. Retrograde amnesia is common after a concussion. The patient can take acetaminophen for headache and should return if symptoms of increased intracranial pressure such as dizziness or nausea occur

The nurse is administering a mental status examination to a patient who has hypertension. The nurse suspects depression when the patient responds to the nurse's questions with A. Is that right B. I don't know C. Wait, let me think about it D. Who are those people over there

B. I don't know Rationale: Answers such as I don't know are more typical of depression than dementia. The response Who are those people over there is more typical of the distraction seen in a patient with delirium. The remaining two answers are more typical of a patient with mild to moderate dementia

A female patient returns to the clinic with a recurrent urethral discharge after being treated for a chlamydial infection 2 weeks ago. Which statement by the patient indicates the most likely cause of the recurrence of her infection? A. I took Vibramycin twice a day for a week B. I haven't told my boyfriend about my infection yet C. I had a couple of beers while I was taking the medication D. I've only had sexual intercourse once since m medication was finished

B. I haven't told my boyfriend about my infection yet Rationale: Notification and treatment of sexual partners are necessary to prevent recurrence and the ping pong effect of passing STIs between partners. Vibramycin is prescribed twice a day for 7 days, and although alcohol may cause more urinary irritation in the patient with chlamydia, it will not interfere with treatment. Avoiding sexual intercourse for 7 days after the medication is to prevent transmission

A 22 year old patient reports her concern about not having a menstrual period for the past 7 months. Which statement by the patient indicates a possible related factor to the amenorrhea? A. I should drink at least 3 glasses of nonfat milk every day B. I run 7 to 8 miles every day to manage my weight C. I am not sexually active but I currently have an IUD D. I was treated for a sexually transmitted infection 2 years ago

B. I run 7 to 8 miles every day to manage my weight Rationale: Intense endurance exercise can cause amenorrhea. The other statements by the patient do not suggest any urgent teaching needs

The nurse provides discharge teaching to a patient following a TURP and determines that the patient understands the instructions when he makes which statement? A. I should use daily enemas to avoid straining until healing is complete B. I will avoid heavy lifting, climbing, and driving until my follow up visit C. At least I don't have to worry about developing cancer of the prostate now D. Every day I should drink 10 to 12 glasses of liquids such as coffee, tea, or soft drinks

B. I will avoid heavy lifting, climbing, and driving until my follow up visit Rationale: Activities that increase intraabdominal pressure should be avoided until the surgeon approves these activities at a follow up visit. Stool softeners and high fiber diets may be used to promote bowel elimination, but enemas should not be used because they increase intraabdominal pressure and may initiate bleeding. Because TURP does not remove the entire prostate gland, the patient needs annual prostatic examinations to screen for cancer of the prostate. Fluid intake should be high, but caffeine and alcohol should not be used because they have a diuretic effect and increase bladder distention

The public health nurse is planning a program to decrease the incidence of meningitis in teenagers and young adults. Which action is most likely to to effective? A. Emphasize the importance of hand washing B. Immunize adolescents and college freshman C. Support serving healthy nutritional options in the college cafeteria D. Encourage adolescents and young adults to avoid crowds in the winter

B. Immunize adolescents and college freshman Rationale: The Neisseeria meningitides vaccination is recommended for children ages 11 and 12 years, unvaccinated teens entering high school, and college freshman. Hand washing may help decrease the spread of bacteria and good nutrition may increase resistance to infection, but those are not as effective as immunization. Because adolescents and young adults are in school or the workplace, avoiding crowds is not realistic

A patient who has benign prostatic hyperplasia (BPH) with urinary retention is admitted to the hospital with elevated blood urea nitrogen (BUN) and creatinine. Which prescribed therapy should the nurse implement first? A. Infuse normal saline at 50mL/hr B. Insert a urinary retention catheter C. Draw blood for a complete blood count D. Schedule pelvic magnetic resonance imaging

B. Insert a urinary retention catheter Rationale: The patient data indicate that the patient may have acute kidney injury caused by the BPH. The initial therapy will be to insert a catheter. The other actions are also appropriate, but they can be implemented after the urinary retention is resolved

The nurse is admitting a patient with a basal skull fracture. The nurse notes ecchymoses around both eyes and clear drainage from the patient's nose. Which admission order should the nurse question? A. Keep the head of the bed elevated B. Insert nasogastric tube to low suction C. Turn patient to the side every 2 hours D. Apply cold packs intermittently to face

B. Insert nasogastric tube to low suction Rationale: Rhinorrhea may indicate a dural tear with cerebrospinal fluid leakage. Insertion of a nasogastric tube will increase the risk for infections such as meningitis. Turning the patient, elevating the head, and applying cold packs are appropriate orders

A 32 year old patient who is diagnosed with chlamydia tells the nurse that she is very angry because her husband is her only sexual partner. Which response should the nurse make first? A. You may need professional counseling to help resolve your anger. B. It is understandable that you feel angry about contracting an infection C. Your feelings are justified and you should share them with your husband D. It is important that both you and your husband be treated for the infection

B. It is understandable that you feel angry about contracting an infection Rationale: This response expresses the nurse's acceptance of the patient's feelings and encourages further discussion and problem solving. The patient may need professional counseling, but more assessment of the patient is needed before making this judgement. The nurse should also assess further before suggesting that the patient share her feelings with the husband because problems such as abuse might be present in the relationship. Although it is important that both partners be treated, the patient's anger suggests that the feelins need to be acknowledged first,

Which instructions should the nurse in a teaching plan for an older patient newly diagnosed with peripheral artery disease (PAD)? A. Exercise only if you do not experience any pain B. It is very important that you stop smoking cigarettes C. Try to keep your legs elevated whenever you are sitting D. Put elastic compression stockings on early in the morning

B. It is very important that you stop smoking cigarettes Rationale: Smoking cessation is essential for slowing the progression of PAD to critical limb ischemia and reducing the risk of myocardial infarction and death. Circulation to the legs will decrease if the legs are elevated. Patients with PAD are taught to exercise to the point of feeling pain, rest, and then resume walking again. Support hose are not used for patients with PAD

The patient has just had a myelogram. What should be included in the nursing care of this patient? A. Restrict fluids until the patient is ambulatory B. Keep the patient positioned flat in bed for several hours C. Position the patient with the head of the bed elevated 30 degrees D. Provide mild analgesics for pain associated with the insertion of needles

B. Keep the patient positioned flat in bed for several hours Rationale: Following a myelogram (and a lumbar puncture), the patient is positioned flat in bed for several hours to avoid a spinal headache. Fluids are encouraged to help in the excretion of the contrast medium. Pain at the insertion site is rare and the most common complaint after a myelogram is a headache

The day shift nurse at the long-term care facility learns that a patient with dementia experienced sundowning late in the afternoon on the previous two days. Which action should the nurse take? A. Have the patient take a brief mid-morning nap. B. Keep window blinds open during the day C. Provide hourly orientation to time and place D. Move the patient to a quiet room in the afternoon

B. Keep window blinds open during the day Rationale: A likely cause of sundowning is a disruption in circadian rhythms, and keeping the patient active and in the daylight will help reestablish a more normal circadian pattern. Moving the patient to a different room might increase confusion. Taking a nap will interfere with nighttime sleep. Hourly orientation will not be helpful in a patient with dementia

When using the heel-to-shin test, for what abnormality is the nurse assessing the patient? A. Hypertonia B. Lack of coordination C. Extension of the toes D. Loss of proprioception

B. Lack of coordination Rationale: The heel to shin test assesses coordination and cerebellar function. Muscle tone is assessed by passively moving limbs through their range of motion and feeling slight resistance. Extensor plantar response is tested with plantar stimulation and my indicate an upper motor neuron lesion. Loss of proprioception (or position sense) is assessed by placing the thumb and forefinger on either side of the patient's forefinger of great toe and gently moving it up and down, then asking the patient to indicate the direction in which the digit was moved

Which nursing assessment finding in a patient who recently started taking hormone replacement therapy (HRT) requires discussion with the health care provider about a change in therapy? A. Breast tenderness B. Left calf swelling C. Weight gain of 3 lb D. Intermittent spotting

B. Left calf swelling Rationale: Unilateral calf swelling may indicate deep vein thrombosis caused by the changes in coagulation associated with HRT and would indicate that the HRT should be discontinued. Breast tenderness, weight gain, and intermittent spotting are common side effects of HRT and do not indicate a need for a change in therapy.

A patient who has severe Alzheimer's disease (AD) is being admitted to the hospital for surgery. Which interventions will the nurse include in the plan of care? A. Encourage the patient to discuss events from the past B. Maintain a consistent daily routine for the patient's care C. Reorient the patient to the date and time every 2 to 3 hours D. Provide the patient with current newspapers and magazines

B. Maintain a consistent daily routine for the patient's care Rationale: Providing a consistent routine will decrease anxiety and confusion for the patient. Reorientation to time and place will not be helpful to the patient with severe AD, and the patient will not be able to read. The patient with severe AD will probably not be able to remember events from the past

A nurse is caring for a patient newly diagnosed with chronic inflammatory demyelinating polyneuropathy (CIDP). Which statement can the nurse accurately use to teach the patient about CIDP? A. Corticosteroids have little effect on this disease B. Maintenance therapy will be needed to prevent relapse C. You will go into remission in approximately eight weeks D. You should be able to walk without help within 3 months

B. Maintenance therapy will be needed to prevent relapse Rationale: Unlike GBS, CIDP does not automatically resolve or go into remission. Most patients with GBS can walk without help within three months. Corticosteroids are a standard therapy for CIDP. Patients with CIDP require maintenance therapy to prevent relapse or progression

A patient with a metastatic tumor of the spinal cord is scheduled for removal of the tumor by laminectomy. In planning post operative care for the patient, what should the nurse recognize? A. Most cord tumors cause autodestruction of the cord as in traumatic injuries B. Metastatic tumors are commonly extradural lesions that are treated palliatively C. Radiation therapy is routinely administered following surgery for all malignant spinal cord tumors D. Because complete removal of intramedullary tumors is not possible

B. Metastatic tumors are commonly extradural lesions that are treated palliatively Rationale: Most metastatic or secondary tumors are extradural lesions in which treatment, including surgery, is palliative. Primary spinal tumors may be removed with the goal of cure. Most tumors of the spinal cord are slow growing, do not cause autodestruction, and, if removal is possible, can have complete function restored. Radiation is used to treat metastatic tumors that are sensitive to radiation and that have caused only minor neurologic deficits in the patient. Radiation is also used as adjuvant therapy to surgery for intramedullary tumors

Which type of headache is suspected when the headaches are unilateral and throbbing, preceded by a prodrome of photophobia, and associated with a family history of this type of headache? A. Cluster B. Migraine C. Frontal-type D. Tension-type

B. Migraine Rationale: Migraine headaches are frequently unilateral and usually throbbing. They may be preceded by a premonitory symptom or aura, and frequently there is a family history. Cluster headaches are also unilateral with severe bone crushing pain, but there is no premonitorry symptom or family history. Frontal type headache is not a functional type of headache but does describe the area of discomfort. Tension type headaches are bilateral with constant, squeezing tightness without premonitory symptoms or family history

When teaching the patient with PAD about modifying risk factors associated with the condition, what should the nurse emphasize? A. Amputation is the ultimate outcome if the patient does not alter lifestyle behaviors B. Modifications will reduce the risk of other atherosclerotic conditions such as stroke C. Risk reducing behaviors initiated after angioplasty can stop the progression of the disease D. Maintenance of normal body weight is the most important factor in controlling arterial disease

B. Modifications will reduce the risk of other atherosclerotic conditions such as stroke Rationale: PAD occurs as a result of atherosclerosis and the risk factors are the same as for other diseases associated with atherosclerosis, such as CAD, cerebrovascular disease, and aneurysms. Major risk factors are tobacco use, hyperlipidemia, elevated C-reactive protein, diabetes mellitus, obesity, and uncontrolled hypertension. The risk for amputation is high in patients with severe occlusive disease, but this is not the best approach to encourage patients to make lifestyle modifications

Vigorous control of fever in the patient with meningitis is required to prevent complications of increased cerebral edema, seizure frequency, neurologic damage, and fluid loss. What nursing care should be included? A. Administer analgesics as ordered B. Monitor LOC related to increased brain metabolism C. Rapidly decrease temperature with a cooling blanket D. Assess for peripheral edema from rapid fluid infusion

B. Monitor LOC related to increased brain metabolism Rationale: LOC must be monitored because it will decrease with the increased brain metabolism that the fever causes. Analgesics will not aid in lowering the body temperature, although acetaminophen will be used as an antipyretic. Rapid cooling may lead to shivering that increases metabolism. Monitoring cerebral edema will be done. Peripheral edema is unrelated and there will not be rapid fluid infusion for the fever. Fluid replacement will be calculated with 800mL/day for respiratory losses and 100mL for each degree of temperature above 100.4

A patient with abdominal and irregular vaginal bleeding is admitted to the hospital with a suspected ectopic pregnancy. Before actual diagnosis, what is the most appropriate action by the nurse? A. Provide analgesics for pain relief B. Monitor vital signs and bleeding frequently C. Offer support for the patient's emotional response to the loss of the pregnancy D. Explain thee need for frequent blood samples for B-human chorionic gonadotropin monitoring

B. Monitor vital signs and bleeding frequently Rationale: Ectopic pregnancy is a life threatening condition. If the fallopian tube ruptures, profuse bleeding can lead to hypovolemic shock. All of the interventions are indicated, but the priority is monitoring the vital signs and pain for evidence of bleeding

After a thymectomy, a patient with myasthenia gravis receives the usual dose of pyridostigmine (Mestinon). An hour later, the patient complains of nausea and severe abdominal cramps. Which action should the nurse take first? A. Auscultate the patient's bowel sounds. B. Notify the patient's health care provider. C. Administer the prescribed PRN antiemetic drug. D. Give the scheduled dose of prednisone (Deltasone).

B. Notify the patient's health care provider. Rationale: The patient's history and symptoms indicate a possible cholinergic crisis. The health care provider should be notified immediately, and it is likely that atropine will be prescribed. The other actions will be appropriate if the patient is not experiencing a cholinergic crisis

The nurse is admitting a patient newly diagnosed with peripheral artery disease. Which admission order should the nurse question? A. Cilostazol drug therapy B. Omeprazole drug therapy C. Use of treadmill for exercise D. Exercise to the point of discomfort

B. Omeprazole drug therapy Rationale: Because the antiplatelet effect of clopidogrel is reduced when it is used with omeprazole, the nurse should clarify this order with the health care provider. The other interventions are appropriate for a patient with peripheral artery disease

A healthy 24 year old patient who has been vaccinated against human papillomavirus (HPV) has a normal Pap test result. Which information will the nurse include in patient teaching when calling the patient with the results of the Pap test? A. You can wait until after age 30 before having another Pap test B. Pap testing is recommended every 3 years for women your age C. No further Pap testing is needed until you decide to become pregnant D. Yearly Pap testing is suggested for women with multiple sexual partners

B. Pap testing is recommended every 3 years for women your age Rationale: Women ages 21 to 29 years should get a Pap test every 3 years

The nurse who works in the vascular clinic has several patients with venous insufficiency scheduled today. Which patient should the nurse assign to an experienced licensed practical/vocational nurse (LPN/LVN)? A. Patient who has been complaining of increased edema and skin changes in the legs B. Patient who needs wound care for a chronic venous stasis ulcer on the right lower leg C. Patient who has a history of venous thromboembolism and is complaining of dyspnea D. Patient who needs teaching about elastic compression stockings for venous insufficiency

B. Patient who needs wound care for a chronic venous stasis ulcer on the right lower leg Rationale: LPN education and scope of practice includes wound care. The other patients, which require more complex assessments or education, should be managed by the RN

A patient undergoes an anterior and posterior (A&P) colporrhaphy for repair for a cystocele and rectocele. Which nursing action will be included in the postoperative care plan? A. Encourage a high fiber diet B. Perform urinary catheter care C. Repack the vagina with gauze daily D. Teach the patient to insert a pessary

B. Perform urinary catheter care Rationale: The patient will have a retention catheter for several days after surgery to keep the bladder empty and decrease strain on the suture. A pessary will not be needed after the surgery. Vaginal wound packing is not usually used after and A&P repair. A low residue diet will be ordered after posterior colporrhaphy

The home health registered nurse (RN) is planning care for a patient with a seizure disorder related to a recent head injury. Which nursing action can be delegated to a licensed practical/vocational nurse (LPN/LVN)? A. Make referrals to appropriate community agencies. B. Place medications in the home medication organizer. C. Teach the patient and family how to manage seizures. D. Assess for use of medications that may precipitate seizures.

B. Place medications in the home medication organizer. Rationale: LPN/LVN education includes administration of medications. The other activities require RN education and scope of practice

An 18 year old female patient who has been admitted to the emergency department after a motor vehicle crash is scheduled for chest and abdominal x-rays. Which information may alter the plans for the x-rays? A. Report of abdominal pain B. Positive result of hCG test C. Blood pressure of 172/88 mmHg D. Temperature of 102.1 F

B. Positive result of hCG test Rationale: Positive hCG testing indicates that the patient is pregnant and abdominal x-rays should be avoided if possible. The other information is also important to report promptly, but it will not affect whether the x-rays should be done

A patient who is suspected of having an epidural hematoma is admitted to the emergency department. Which action will the nurse expect to take? A. Administer IV furosemide (Lasix) B. Prepare the patient for craniotomy C. Initiate high dose barbiturate therapy D. Type and crossmatch for blood transfusion

B. Prepare the patient for craniotomy Rationale: The principal treatment for epidural hematoma is rapid surgery to remove the hematoma and prevent herniation. If intracranial pressure is elevated after surgery, furosemide or high dose barbiturate therapy may be needed, but these will not be of benefit unless the hematoma is removed. Minimal blood loss occurs with head injuries, and transfusion is usually not necessary

An unconscious male patient has just arrived in the emergency department with a head injury caused by a motor cycle crash. Which order should the nurse question? A. Obtain x-rays of the skull and spine B. Prepare the patient for lumbar puncture C. Send for computed tomography (CT) scan D. Perform neurologic checks every 15 minutes

B. Prepare the patient for lumbar puncture Rationale: After a head injury, the patient may be experiencing intracranial bleeding and increased intracranial pressure, and herniation of the brain could result if lumbar puncture is performed. Thee other orders are appropriate

A 28 year old female patient has been diagnosed with occipital lobe damage after a car accident. With what should the nurse expect the patient to need help? A. Being able to feel heat B. Processing visual images C. Identifying smells appropriately D. Being able to say what she means

B. Processing visual images Rationale: The occipital lobe is responsible for visual perception. This patient may experience inability to identify colors, hallucinations, visual loss, or total blindness. Heat is sensed with the sensory part of the brain. The olfactory nerve is responsible for identifying smells. Broca's area regulates verbal expression.

A normal male reproductive function that may be altered in a patient who undergoes a orchiectomy (removal of testes) is A. Production of GnRH B. Production of testosterone C. Production of progesterone D. production of seminal fluid

B. Production of testosterone Rationale: Orchiectomy is the removal of a testicle. Testosterone is produced in the testicles. Progesterone is produced in the female to maintain the rich vascular state of the uterus in preparation for fertilization and implantation. Seminal fluid is produced in the prostate. Production of GnRH takes place in the anterior pituitary

After talking with the HCP, the patient asks what the blood brain barrier does. What is the best description that the nurse can give the patient? A. Protects the brain from external trauma B. Protects against harmful blood borne agents C. Provides flexibility while protecting the spinal cord D. Forms the outer layer of protective membranes and the brain and spinal cord

B. Protects against harmful blood borne agents Rationale: The blood brain barrier physiologically protects the brain from harmful agents in the blood. The skull protects the brain from external trauma. The vertebral column allows flexibility while protecting thee spinal cord. The dura mater is the outer protective membrane

Which action will the emergency department nurse anticipate for a patient diagnosed with a concussion who did not lose consciousness? A. Coordinate the transfer of the patient to the operating room B. Provide discharge instructions about monitoring neurologic status C. Transport the patient to radiology for magnetic resonance imaging (MRI) D. Arrange to admit the patient to the neurologic unit for 24 hours of observation

B. Provide discharge instructions about monitoring neurologic status Rationale: A patient with a minor head trauma is usually discharged with instructions about neurologic monitoring and the need to return if neurologic status deteriorates. MRI, hospital admission, and surgery are not usually indicated in a patient with a concussion

Which intervention will the nurse include in the plan of care for a patient with moderate dementia who had a fractured hip repair 2 days ago? A. Provide complete personal hygiene care for the patient B. Remind the patient frequently about being in the hospital C. Reposition the patient frequently to avoid skin breakdown D. Place suction at the bedside to decrease the risk of aspiration

B. Remind the patient frequently about being in the hospital Rationale: The patient with moderate dementia will have problems with short and long term memory and will need reminding about the hospitalization. The other interventions would be used for a patient with severe dementia, who would have difficulty with swallowing, self care, and immobility

When caring for a patient with continuous bladder irrigation after having transurethral resection of the prostate, which action could the nurse delegate to unlicensed assistive personnel? A. Teach the patient how to perform Kegel exercises B. Report any complaints of pain or spasms to the nurse C. Monitor for increases in bleeding or presence of clots D. Increase the flow rate of the irrigation if clots are noted

B. Report any complaints of pain or spasms to the nurse Rationale: UAP education and role includes reporting patient concerns to supervising nurses. Patient teaching, assessments for complications, and actions such as bladder irrigation require more education and should be done by licensed nursing staff

A patient admitted with a diffuse axonal injury has a systemic blood pressure (BP) of 106/52 mmHg and an intracranial pressure of 14 mmHg. Which action should the nurse take first? A. Document the BP and ICP in the patient's record B. Report the BP and ICP to the health care provider C. Elevate the head of the patient's bed to 60 degrees D. Continue to monitor the patient's vital signs and ICP

B. Report the BP and ICP to the health care provider Rationale: Calculate the cerebral perfusion pressure (CPP: (CPP=Mean arterial pressure [MAP]- ICP.) MAP=DBP+1/3 (Systolic blood pressure [SBP]-Diastolic blood pressure [DBP]). Therefore, the MAP is 70 and the CPP is 56mmHg, which are below the normal values of 60-100 mmHg and are approaching the level of ischemia and neuronal death. Immediate changes in the patient's therapy such as fluid infusion or vasopressor administration are needed to improve the CPP. Adjustments in the head elevation should only be done after consulting with the health care provider. Continued monitoring and documentation will also be done, but they are not the first actions the nurse should take

A 20 year old patient who sustained a T2 spinal cord injury 10 days ago tells the nurse, I want to be transferred to another hospital where the nurses know what they are doing. Which action by the nurse is appropriate? A. Respond that abusive language will not be tolerated B. Request that the patient provide input for the plan of care C. perform care without responding to the patients comments D. Reassure the patient about the competence of the nursing staff

B. Request that the patient provide input for the plan of care Rationale: The patient is demonstrating behaviors consistent with the anger phase of the grief process, and the nurse should allow expression of anger and seek the patient's input into care. Expression of anger is appropriate at this stage, and should be accepted by the nurse. Reassurance about the competency of the staff will not be helpful in responding to the patient's concerns. Ignoring the patient's comments will increase the patient's anger and sense of helplessness

A patient has been diagnosed with cancer of the ovary. In planning care for the patient, the nurse recognizes that treatment of the patient depends on what? A. Results of a direct needle biopsy of the ovary B. Results of a laparotomy with multiple biopsies C. Whether the patient desires to maintain fertility D. The findings of metastasis by ultrasound or CT scan

B. Results of a laparotomy with multiple biopsies Rationale: Treatment of ovarian cancer is determined by staging from the results of laparotomy with multiple biopsies of the ovaries and other tissue throughout the pelvis and lower abdomen. The patient's desire for fertility is not a consideration because of the high mortality rate associated with ovarian cancer. Although diagnosis of ovarian tumors may be made by transvaginal ultrasound or CT scan, the treatment of ovarian cancer depends on the staging of the tumor

A patient with pelvic inflammatory disease (PID) is being treated with oral antibiotics as an outpatient. Which instruction will be included in patient teaching? A. Abdominal pain may persist for several weeks B. Return for a follow up appointment in 2 to 3 days C. Instruct a male partner to use a condom during sexual intercourse for the next week D. Nonsteroidal antiinflammatory drug (NSAID) use may prevent pelvic organ scarring

B. Return for a follow up appointment in 2 to 3 days Rationale: The patient is instructed to return for follow up in 48 to 72 hours. The patient should abstain from intercourse for 3 weeks. Abdominal pain should subside with effective antibiotic therapy. Corticosteroids may help prevent inflammation and scarring, but NSAIDs will not decrease scarring

A 31 year old patient who has been diagnosed with human papillomavirus (HPV) infection gives a health history that includes smoking tobacco, taking oral contraceptives, and having been treated twice for vaginal candidiasis. Which topic will the nurse include in patient teaching? A. Use of water soluble lubricants B. Risk factors for cervical cancer C. Antifungal cream administration D. Possible difficulties with conception

B. Risk factors for cervical cancer Rationale: Because HPV infection and smoking are both associated with increased cervical cancer risk, the nurse should emphasize the importance of avoiding smoking. An HPV infection does not decrease vaginal lubrication, decrease the ability to conceive, or require the use of antifungal creams

A patient seen in the outpatient clinic is diagnosed with mild cognitive impairment (MCI). Which action will the nurse include in the plan of care? A. Suggest a move into an assisted living facility B. Schedule the patient for more frequent appointments C. Ask family members to supervise the patient's daily activities D. Discuss the preventative use of acetylcholinesterase medications

B. Schedule the patient for more frequent appointments Rationale: Ongoing monitoring is recommended for patients with MCI. MCI does not usually interfere with activities of daily living, acetylcholinesterase drugs are not used for MCI, and an assisted living facility is not indicated for a patient with MCI

What is the fertility test that requires a couple to have no sexual intercourse for 2 to 3 days before the test? A. Urinary LH B. Semen analysis C. Endometrial biopsy D. Hysterosalpingogram

B. Semen analysis Rationale: A semen analysis requires a couple to have no sexual intercourse 2 to 3 days before and examination of semen for the volume, viscosity, number, mobility, and structure of sperm. Urinary LH is an over the counter test to identify midcycle LH surge that precedes ovulation by 1 to 2 days. An endometrial biopsy provides a sample of endometrium to evaluate its changes under the influence of progesterone. A hysterosalpingogram is a contrast x-ray of the uterine cavity and fallopian tubes. Intercourse does not affect these other three test results

A young woman is admitted to the hospital with acute pelvic inflammatory disease (PID). During the nursing history, the nurse notes which risk factor as being most significant for this patient? A. Lack of any method of birth control B. Sexual activity with multiple partners C. Use of a vaginal sponge for contraception D. Recent antibiotic induced monilial vaginitis

B. Sexual activity with multiple parnters Rationale: Sexual activity with multiple partners increase the risk for pelvic inflammatory disease (PID), and there is often a history of an acute infection of the lower genital tract caused by gonococcal or chlamydia microorganisms. The only significant contraceptive issue related to PID is that condom use will help to prevent STIs that may lead to PID

The patient is diagnosed with Brown Sequard syndrome after a knife wound to the spine. Which description accurately describes this syndrome? A. Damage to the most distal cord and nerve roots, resulting in flaccid paralysis of the lower limbs and areflexic bowel and bladder B. Spinal cord damage resulting in ipsilateral motor paralysis and contralateral loss of pain and sensation below the level of the injury C. rare cord damage resulting in loss of proprioception below the lesion level with retention of motor control and temperature and pain sensation D. Often caused by flexion injury with acute compression of cord resulting in complete motor paralysis and loss of pain and temperature sensation below the level of injury

B. Spinal cord damage resulting in ipsilateral motor paralysis and contralateral loss of pain and sensation below the level of the injury Rationale: Brown Sequard syndrome is characterized by ipsilateral loss of motor function and position and vibratory sense, and contralateral loss of pain and temperature sensation below the level of the injury. Damage to the most distal cord and nerve roots with flaccid paralysis of the lower limbs and areflexic bowel and bladder is seen with cauda equine syndrome or conus medullaris syndrome. Posterior cord syndrome is rare, with cord damage resulting in loss of proprioception below the lesion level but retention of motor control and temperature and pain sensation. Anterior cord syndrome is often caused by flexion injury, with acute compression of the cord resulting in complete motor paralysis and loss of pain and temperature sensation below the level of injury but touch, position, vibration, and motion remaining intact

A patient is admitted to the hospital with a diagnosis of abdominal aortic aneurysm. Which signs and symptoms would suggest that his aneurysm has ruptured? A. Rapid onset of shortness of breath and hemoptysis B. Sudden, severe low back pain and bruising along his flank C. Gradually increasing substernal chest pain and diaphoresis D. Sudden, patchy blue mottling on feet and toes and rest pain

B. Sudden, severe low back pain and bruising along his flank Rationale: The clinical manifestations of a ruptured aortic aneurysm include severe back pain, back or flank ecchymosis (Grey Turner's sign), and hypovolemic shock (tachycardia, hypotension, pale clammy skin, decreased urine output, altered level of consciousness, and abdominal tenderness).

Which intervention will the nurse include in the plan of care for a patient with primary restless legs syndrome (RLS) who is having difficulty sleeping? A. Teach about the use of antihistamines to improve sleep. B. Suggest that the patient exercise regularly during the day. C. Make a referral to a massage therapist for deep massage of the legs. D. Assure the patient that the problem is transient and likely to resolve.

B. Suggest that the patient exercise regularly during the day. Rationale: Nondrug interventions such as getting exercise are initially suggested to improve sleep quality in patients with RLS. Antihistamines may aggravate RLS. Massage does not alleviate RLS symptoms, and RLS is likely to progress in most patients

A patient with Parkinson's disease has bradykinesia. Which action will the nurse include in the plan of care? A. Instruct the patient in activities that can be done while lying or sitting B. Suggest that the patient rock from side to side to initiate leg movement C. Have the patient take small steps in a straight line directly in front of the feet D. Teach the patient to keep the feet in contact with the floor and slide them forward

B. Suggest that the patient rock from side to side to initiate leg movement Rationale: Rocking the body from side to side stimulates balance and improves mobility. The patient will be encouraged to continue exercising because this will maintain functional abilities. Maintaining a wide base of support will help with balance. Th patient should lift the feet and avoid a shuffling gait

A 51 year old woman suffered a wrist fracture when she slipped on the ice. She has her uterus and is interested in starting hormone replacement therapy (HRT), as she is also experiencing menopause symptoms. What should the nurse include when discussing the risks and benefits of HRT with this patient? A. Taking only progesterone is suggested for a woman with a uterus B. Taking both estrogen and progesterone may decrease her bone loss C. The risk of breast cancer and cardiovascular disease is decreased with HRT. D. Taking estrogen and progesterone will increase the risk of endometrial cancer

B. Taking both estrogen and progesterone may decrease her bone loss Rationale: Taking combination hormone replacement therapy (HRT) increases bone marrow density and decreases fractures. Both progesterone and estrogen are recommended for a menopausal woman with a uterus. Progesterone alone is not as effective as estrogen. The risk for breast cancer is increased, and the risk for endometrial cancer is decreased with combination HRT. Evidence does not support using HRT to prevent cardiovascular disease

Which action will the nurse plan to take for a patient with multiple sclerosis who has urinary retention caused by a flaccid bladder? A. Encourage a decreased evening intake of fluid B. Teach the patient how to use the Crede method C. Suggest the use of adult incontinence briefs for nighttime only D. Assist the patient to the commode every 2 hours during the day

B. Teach the patient how to use the Crede method Rationale: The Crede method can be used to improve bladder emptying. Decreasing fluid intake will not improve bladder emptying and may increase risk for urinary tract infection and dehydration. The use of incontinence briefs and frequent toileting will not improve bladder emptying

A patient with AD in a long term care facility is wandering the halls very agitated, asking for her mommy and crying. What is the best response by the nurse? A. Ask the patient, Why are you behaving this way B. Tell the patient, Let's go get a snack in the kitchen C. Ask the patient, Wouldn't you like to lie down now? D. Tell the patient, Just take some deep breaths and calm down

B. Tell the patient, Let's go get a snack in the kitchen Rationale: Patients with moderate to severe AD frequently become agitated, but because their short term memory loss is so pronounced, distraction is a very good way to calm them. Why questions are upsetting to them because they don't know the answer and they cannot respond to normal relaxation techniques

Which information will the nurse plan to include when teaching a young adult who has a family history of testicular cancer about testicular self examination? A. Testicular self examination should be done at least weekly B. Testicular self examination should be done in a warm room C. The only structure normally felt in the scrotal sac is the testis D. Call the health care provider if one testis is larger than the other

B. Testicular self examination should be done in a warm room Rationale: The testes will hang lower in the scrotum when the temperature is warm (e.g. during a shower), and it will be easier to palpate. The epididymis is also normally palpable in the scrotum. One testis is normally larger. Men at high risk should perform testicular self examination monthly

A 70 year old patient is admitted after falling from his roof. He has a spinal cord injury (SCI) at the C7 level. What findings during this assessment identify the presence of spinal shock? A. Paraplegia with a flaccid paralysis B. Tetraplegia with total sensory loss C. Total hemiplegia with sensory and motor loss D. Spastic tetraplegia with loss of pressure sensation

B. Tetraplegia with total sensory loss Rationale: At the C7 level, spinal shock is manifested by tetraplegia and sensory loss. The neurologic loss may be temporary or permanent. Paraplegia with flaccid paralysis would occur at the level of T1 or below. A hemiplegia occurs with central (brain) lesions affecting motor neurons and spastic tetraplegia occurs when spinal shock resolves

A week following SCI at T2, a patient experiences movement in his leg and tells the nurse that he is recovering some function. What is the nurse's best response to the patient? A. It is really still too soon to know if you will have a return of function B. That could be a really positive finding. Can you show me the movement C. That's wonderful. We will start exercising your legs more frequently now D. I'm sorry but the movement is only a reflex and does not indicate normal function

B. That could be a really positive finding. Can you show me the movement Rationale: When spinal shock ends, reflex movement and spasms will occur, which may be mistaken for return of function. However, with the resolution of edema, some normal function may also occur. It is important when movement occurs to determine whether the movement is voluntary and can be consciously controlled, which would indicate some return of function. If movement is is not voluntary, reflex return will be explained

The family caregiver for a patient with AD expresses an inability to make decisions, concentrate, or sleep. The nurse determines what about the caregiver? A. The caregiver is also developing signs of AD B. The caregiver is manifesting symptoms of caregiver role strain C. The caregiver needs a period of respite from care of the patient D. The caregiver should ask other family members to participate in the patient's care

B. The caregiver is manifesting symptoms of caregiver role strain Rationale: Family caregiver role strain is characterized by such symptoms of stress as the inability to sleep, make decisions, or concentrate. It is frequently seen in family members who are responsible for the care of the patient with AD. Assessment of the caregiver may reveal a need for assistance to increase coping skills, effectively use community resources, or maintain social relationships. Eventually the demands on a caregiver exceed the resources, and the person with AD may be placed in an institutional setting

A patient is scheduled for an induced abortion using instillation of hypertonic saline solution. Which information will the nurse plan to discuss with the patient before the procedure? A. The patient will require a general anesthetic B. The expulsion of the fetus may take 1 to 2 days C. There is a possibility that the patient may deliver a live fetus D. The procedure may be unsuccessful in terminating the pregnancy

B. The expulsion of the fetus may take 1 to 2 days Rationale: Uterine contractions take 12 to 36 hours to begin after the hypertonic saline is instilled. Because the saline is feticidal, the nurse does not need to discuss any possibility of a live delivery or that the pregnancy termination will not be successful. General anesthesia is not needed for this procedure.

What is the most common way to determine a diagnosis of chlamydial infection in a male patient? A. Cultures for chlamydial organisms are positive B. The nucleic acid amplification test (NAAT) is positive C. Gram stain smears and cultures are negative for gonorrhea D. Signs and symptoms of epididymitis or proctotis are also present

B. The nucleic acid amplification test (NAAT) is positive Rationale: The NAAT is more sensitive than other diagnostic tests, can be done with a urine sample, and has results within 24 hours. A cell culture can be used to detect chlamydia organisms, but it requires specific handling and is not as easy or as fast to perform as the NAAT. Gonorrhea and chlamydia have very similar symptoms in men and frequently occur together. Gram stain smears and cultures for N. Gonorrhoeae do not definitively diagnose chalmydia. Manifestations of epididymitis or proctitis may be present, as with other STIs, but are not diagnostic

The health care provider is considering the use of sumatriptan (Imitrex) for a 54 year old male patient with migraine headaches. Which information obtained by the nurse is most important to report to the health care provider? A. The patient drinks 1 to 2 cups of coffee daily B. The patient had a recent acute myocardial infarction C. The patient has had migraine headaches for 30 years D. The patient has taken topiramate (Topamax) for 2 months

B. The patient had a recent acute myocardial infarction Rationale: The triptans cause coronary artery vasoconstriction and should be avoided in patients with coronary artery disease. The other information will be reported to the health care provider, but none of it indicates that sumatriptan would be an appropriate treatment

A patient with a 10-week pregnancy is admitted to the emergency department with vaginal bleeding and abdominal cramping. What does the nurse recognize about this situation? A. The patient will recover quickly when the bleeding stops B. The patient is most likely experiencing a spontaneous abortion C. The patient will be scheduled for an immediate dilation and curettage (D&C) D. Treatment of the patient with bed rest is usually successful in preventing further bleeding

B. The patient is most likely experiencing a spontaneous abortion Rationale: In the presence of a confirmed pregnancy, uterine cramping with vaginal bleeding is the most important sign of spontaneous abortion. Other conditions causing vaginal bleeding, such as an incompetent cervix, do not usually cause cramping. There is no evidence that any medical treatment improves the outcome for spontaneous abortion. Blood loss can be significant, and the loss of the pregnancy may cause long term grieving. Dilation and curettage (D&C) (if needed) is performed after the abortion to minimize blood loss and reduce the chance of infection

A 39 year old patient with a suspected herniated intervertebral disc is scheduled for a myelogram. Which information communicated by the nurse to the health care provider before the procedure would change the procedural plans? A. The patient is anxious about the test results B. The patient reports a previous allergy to shellfish C. The patient has back pain when lying flat for more than 4 hours D. The patient drank apple juice 4 hours before the scheduled procedure

B. The patient reports a previous allergy to shellfish Rationale: A contrast medium containing iodine is injected into the subarachnoid space during a myelogram. The patient's allergy would contraindicate the use of this medium. The health care provider may need to modify the orders to prevent back pain, but this can be done after the procedure. Clear liquids are usually considered safe up to 4 hours before a diagnostic or surgical procedure. The patient's anxiety should be addressed, but procedural plans would not need to be changed

A 24 year old woman says she wants to begin using oral contraceptives. Which information from the nursing assessment is important to report to the health care provider before a prescription is considered? A. The patient quit smoking 5 months previously B. The patient's blood pressure is 150/86 mmHg C. The patient has not been vaccinated for rubella D. The patient has chronic iron deficiency anemia

B. The patient's blood pressure is 150/86 mmHg Rationale: Because hypertension increases the risk for mobidity and mortality in women taking oral contraceptives, the patient's blood pressure should be controlled before oral contraceptives are prescribed. The other information will not affect the choice of contraceptive

The patient with VTE is receiving therapy with heparin and asks the nurse whether the drug will dissolve the clot in her leg. What is the best response by the nurse? A. This drug will break up and dissolve the clot so that circulation in the vein can be restored B. The purpose of the heparin is to prevent growth of the clot or formation of new clots where the circulation is slowed C. Heparin won't dissolve the clot, but it will inhibit the inflammation around the clot and delay the development of new clots D. The heparin will dilate the vein, preventing turbulence of blood flow around the clot that may cause it to break off and travel to the lungs

B. The purpose of the heparin is to prevent growth of the clot or formation of new clots where the circulation is slowed Rationale: Anticoagulant therapy with heparin or warfarin (Coumadin) does not dissolve clots but prevents propagation of the clot, development of new thrombi, and embolization. Clot lysis occurs naturally through the body's intrinsic fibrinolytic system or by the administration of thrombolytic agents

Before undergoing a TURP, what should the patient be taught? A. Some degree of urinary incontinence is likely to occur. B. This surgery results in some degree of retrograde ejaculation. C. Erectile dysfunction is a common complication of this prostate surgery. D. An indwelling catheter will be used to maintain urinary output until healing is complete.

B. This surgery results in some degree of retrograde ejaculation. Rationale: Because of injury to the internal urinary sphincter, there is usually some degree of retrograde ejaculation following most transurethral surgeries, especially following TURP. Some semen travels back with ejaculation into the bladder and is eliminated with the next voiding. Urinary incontinence, erectile dysfunction, and continued catheterization are uncommon following TURP.

A patient has a lesion involving the fasciculus gracilis and fasciculus cuneatus of the spinal cord. The nurse should expect the patient to experience the loss of what? A. Pain and temperature sensations B. Touch, deep pressure, vibration, and position sense C. Subconscious information about body position and muscle tension D. Voluntary muscle control from the cerebral cortex to the peripheral nerves

B. Touch, deep pressure, vibration, and position sense Rationale: The fasciculus gracilis and fasciculus cutaneous tracts carry information and transmit impulses concerned with touch, deep pressure, vibration, position sense, and kinesthesia. Spinothalmic tracts carry pain and temperature sensations. The spinocerebellar tracts carry subconscious information about muscle tension and body position. Descending corticobulbar tracts carry impulses responsible to voluntary impulse from the cortex to the cranial and peripheral nerves

Which treatment for BPH uses a low wave radiofrequency to precisely destroy prostate tissue? A. Laser prostatectomy B. Transurethral needle ablation (TUNA) C. Transurethral microwave thermotherapy (TUMT) D. Transurethral electrovaporization of the prostate (TUVP)

B. Transurethral needle ablation (TUNA) Rationale: The transurethral needle ablation (TUNA) uses low wave radiofrequency to heat the prostate, causing necrosis. Laser prostatectomy uses a laser beam. Transurethral microwave thermotherapy (TUMT) uses microwave radiating heat to produce coagulative necrosis of the prostate and is not used for men with rectal problems. Transurethral electrovaporization of prostate (TUVP) uses electrosurgical vaporization and desiccation to destroy prostate tissue

Which equipment will the nurse obtain to assess vibration sense in a diabetic patient who has peripheral nerve dysfunction? A. Sharp pin B. Tuning fork C. Reflex hammer D. Calibrated compass

B. Tuning fork Rationale: Vibration sense is testing by touching the patient with a vibrating tuning fork. The other equipment is needed for testing of pain sensation, reflexes, and two point discrimination

Which information about a 76 year old patient should the nurse report as uncharacteristic of normal aging? A. Triceps reflex response graded at 1/5 B. Unintended weight loss of 15 pounds C. 10 mmHg orthostatic drop in blood pressure D. Patient complaint of chronic difficulty in falling asleep

B. Unintended weight loss of 15 pounds Rationale: Although changes in appetite are normal with aging, a 15lb weight loss requires further investigation. Orthostatic drops in blood pressure, changes in sleep patterns, and slowing of reflexes are normal changes in aging

A patient admitted with chest pain is also found to have positive Venereal Disease Research Laboratory (VDRL) and fluorescent treponemal antibody absorption (FAT-Abs) tests, rashes on the palms and the soles of the feet, and moist papules in the anal and vulvar area. Which action will the nurse include in the plan of care? A. Assess for arterial aneurysms B. Wear gloves for patient contact C. Place the patient in a private room D. Apply antibiotic ointment to the perineum

B. Wear gloves for patient contact Rationale: Exudate from any lesions with syphilis is highly contagious. Systemic antibiotics, rather than local treatment of lesions, are used to treat syphilis. The patient does not require a private room because the disease is spread through contact with the lesions. This patient has clinical manifestations of secondary syphilis and does not need to be monitored for manifestations of tertiary syphilis

A patient with newly diagnosed MS has been hospitalized for evaluation and initial treatment of the disease. Following discharge teaching, the nurse realizes that additional instruction is needed when the patient says what? A. It is important for me to avoid exposure to people with upper respiratory infectinos B. When I begin to feel better, I should stop taking the prednisone to prevent side effects C. I plan to use vitamin supplements and a diet high in fiber to help manage my condition D. I must plan with my family how we are going to manage my care if I become more incapacitated

B. When I begin to feel better, I should stop taking the prednisone to prevent side effects Rationale: Corticosteroids used in treating acute exacerbations of MS should not be abruptly stopped by the patient because adrenal insufficiency may result and prescribed tapering doses should be followed. Infections may exacerbate symptoms and should be avoided. High fiber diets with vitamin supplements are advocated. Long term planning for increasing disability is also important

Which action will the nurse include in the plan of care for a patient with impaired functioning of the left glossopharyngeal nerve (CN IX) and vagus nerve (CN X)? A. Assist to stand and ambulate B. Withhold oral fluids and food C. Insert an oropharyngeal airway D. Apply artificial tears every hour

B. Withhold oral fluids and food Rationale: The glossopharyngeal and vagus nerves innervate the pharynx and control the gag reflex. A patient with impaired function of these nerves is at risk for aspiration. An oral airway may be needed when a patient is unconscious and unable to maintain the airway, but it will not decrease aspiration risk. Taste and eye blink are controlled by the facial nerve. Balance and coordination are cerebellar functions

A patient is being evaluated for Alzheimer's disease (AD). The nurse explains to the patient's adult children that A. the most important risk factor for AD is a family history of the disorder B. a diagnosis of AD is made only after other causes of dementia are ruled out C. new drugs have been shown to reverse AD deterioration dramatically in some patients D. brain atrophy detected by magnetic resonance imaging (MRI) would confirm the diganosis of AD

B. a diagnosis of AD is made only after other causes of dementia are ruled out Rationale: The diagnosis of AD is usually one of exclusion. Age is the most important risk factor for development of AD. Drugs ay slow the deterioration but do not reverse the effects of AD. Brain atrophy is a common finding in AD, but it can occur in other diseases as well and does not confirm the diagnosis of AD

A patient is seen in the emergency department after diving into the pool and hitting the bottom with a blow to the face that hyperextended the neck and scraped the skin off the nose. The patient also described "having double vision" when looking down. During the neurologic exam, the nurse, the nurse finds the patient is unable to abduct either eye. The nurse recognizes this finding is related to A. a basal skull fracture B. a stretch injury to bilateral CN VI C. a stiff neck from the hyperextension injury D. facial swelling from the scrape on the bottom of the pool

B. a stretch injury to bilateral CN VI Rationale: Cranial nerves III (oculomotor), IV (trochlear), and VI (abducens) are responsible for eye movement. The lateral rectus eye muscle, the primary muscle responsible for lateral eye movement, in innervated by cranial nerve VI. The hyperextension injury, which can be experienced in diving accidents, or rear end motor vehicle crashes, can stretch these nerves bilaterally

Serum tumor markers that may be elevated on diagnosis of testicular cancer and used to monitor the response to therapy include A. tumor necrosis factor (TNF) and C-reactive protein (CRP) B. a-fetoprotein (AFP) and human chorionic gonadotropin (hCG) C. prostate specific antigen (PSA) and prostate acid phosphatase (PAP) D. carcinoembyronic antigen (CEA), antinuclear antibody (ANA) and HER-2

B. a-fetoprotein (AFP) and human chorionic gonadotropin (hCG) Rationale: a-fetoprotein (AFP) and human chorionic gonadotropin (hCG) are glycoproteins that may be elevated to testicular cancer. If they are elevated before surgical treatment, the levels are noted, and if response to therapy is positive, the levels will decrease. Lactate dehydrogenase (LDH) may also be elevated. Tumor necrosis factor (TNF) is a normal cytokine responsible for tumor surveillance and destruction. C-reactive protein (CRP) is found in inflammatory conditions and widespread malignancies. PSA and PAP are used for screening of prostatic cancer. Carcinoembryonic antigen (CEA) is a tumor marker for cancers of the GI system. Antinuclear antibody (ANA) is found most frequently in autoimmune disorders. HER-2 is used as a marker in breast cancer

Before administering botulinum antitoxin to a patient in the emergency department, it is most important for the nurse to A. obtain the patient's temperature B. administer an intradermal test dose C. document the neurologic symptoms D. ask the patient about an allergy to eggs

B. administer an intradermal test dose Rationale: To assess for possible allergic reactions, an intradermal test dose of the antitoxin should be administered. Although temperature, allergy history, and symptom assessment and documentation are appropriate, these assessments will not affect the decision to administer the antitoxin

A construction worker arrives at an urgent care center with a deep puncture wound from a rusty nail. The patient reports having had a tetanus booster 6 years ago. The nurse will anticipate A. IV infusion of tetanus immune globulin (TIG) B. administration of the tetanus diptheria (Td) booster C. intradermal injectio of an immune globulin test dose D. initiation of the tetanus diphtheria immunization series

B. administration of the tetanus diptheria (Td) booster Rationale: If the patient has not been immunized in the past 5 years, administration of thee Td booster is indicated because the wound is deep. Immune globulin administration is given by the IM route if the patient has no previous immunization. Administration of a series of immunization is not indicated. TIG is not indicated for this patient, and a test dose is not needed for immune globulin

When a 74-year-old patient is seen in the health clinic with new development of a stooped posture, shuffling gait, and pill rolling-type tremor, the nurse will anticipate teaching the patient about A. oral corticosteroids. B. antiparkinsonian drugs. C.magnetic resonance imaging (MRI). D. electroencephalogram (EEG) testing.

B. antiparkinsonian drugs. Rationale: The clinical diagnosis of Parkinson's is made when tremor, rigidity, and akinesia, and postural instability are present. The confirmation of the diagnosis is made on the basis of improvement when antiparkinsonian drugs are administered. MRI and EEG are not useful in diagnosing Parkinson's disease, and corticosteroid therapy is not used to treat it

The health care provider prescribes an infusion of heparin and daily partial thromboplastin time (PTT) testing for a patient with venous thromboembolism (VTE). The nurse will plan to A. decrease the infusion when the PTT value is 65 seconds B. avoid giving IM medications to prevent localized bleeding C. have vitamin K available in case reversal of the heparin is needed D. monitor posterior tibial and dorsalis pedis pulses with the doppler

B. avoid giving IM medications to prevent localized bleeding Rationale: Intramuscular injections are avoided in patients receiving anticoagulation to prevent hematoma formation and bleeding from the site. A PTT of 65 seconds is within the therapeutic range. Vitamin K is used to reverse Warfarin. Pulse quality is not affected by VTE

The most common early symptom of a spinal cord tumor is A. urinary incontinence B. back pain that worsens with activity C. paralysis below the level of involvement D, impaired sensation of pain, temperature, and light touch

B. back pain that worsens with activity Rationale: The most common early symptom of a spinal cord tumor is pain in the back, with radicular pain following the nerve(s) affected. The location of the pain depends on the level of compression. The pain worsens with activity, coughing, straining, and lying down

When caring for a 58 year old patient with persistent menorrhagia, the nurse will plan to monitor the A. estrogen level B. complete blood count (CBC) C. gonadotropin-releasing hormone (GNRH) level. D. serial human chorionic gonadotropin (hCG) results

B. complete blood count (CBC) Rationale: Because anemia is a likely complication of menorrhagia, the nurse will need to check the CBC. Estrogen and GNRH levels are checked for patients with other problems, such as infertility. Serial hCG levels are monitored in patients who may be pregnant, which is not likely for this patient

A nursing measure that is indicated to reduce the potential for seizures and increased intracranial pressure in the patient with bacterial meningitis is A. administering codeine for relief of head and neck pain B. controlling fever with prescribed drugs and cooling techniques C. keeping the room darkened and quiet to minimize environmental stimulation D. maintaining the patient on strict bed rest with the head of the bed slightly elevated

B. controlling fever with prescribed drugs and cooling techniques Rationale: Fever must be vigorously managed because it increases cerebral edema and the frequency of seizures. Neurologic damage may result from an extremely high temperature over a prolonged period. Acetaminophen or aspirin may be used to reduce fever; other measures, such as a cooling blanket or tepid sponge baths with water, may be effective in lowering the temperature

The nurse will plan to teach the patient who is incontinent of urine following a radical retropubic prostatectomy to A. restrict oral fluid intake B. do pelvic muscle exercises C. perform intermittent self-catheterization D. use belladonna and opium suppositories

B. do pelvic muscle exercises Rationale: Pelvic floor muscle training (Kegel) exercises are recommended to strengthen the pelvic floor muscles and improve urinary control. Belladonna and opium suppositories are used to reduce bladder spasms after surgery. Intermittent self catheterization may be taught before surgery if the patient has urinary retention, but it will not be useful in reducing incontinence after surgery. The patient should have a daily oral intake of 2 to 3 L

After scheduling a patient with a possible ovarian cyst for ultrasound, the nurse will teach the patient that she should A. expect to receive IV contrast during the procedure. B. drink several glasses of fluids before the procedure. C. experience mild abdominal cramps after the procedure. D. discontinue taking aspirin for 7 days before the procedure.

B. drink several glasses of fluids before the procedure. Rationale: A full bladder is needed for many ultrasound procedures, so the nurse will have the patient drink fluids before arriving for the ultrasound. The other instructions are not accurate for this procedure

The nurse plans care for the patient with increased intracranial pressure with the knowledge that the best way to position the patient is to A. keep the head of the bed flat B. elevate the head of the bed to 30 degrees C. maintain patient of the left side with the head supported on a pillow D. use a continuous rotation bed to continuously change patient position

B. elevate the head of the bed to 30 degrees Rationale: The nurse should maintain the patient with abnormal ICP in the head up position. Elevation of the head of the bed to 30 degrees enhances respiratory exchange and aids in decreasing cerebral edema. The nurse should position the patient to prevent extreme neck flexion, which can cause venous obstruction and contribute to elevation in ICP. Elevation of the head of the bed also reduces sagittal sinus pressure, promotes drainage from the head through the valveless venous system and jugular veins, and decreases the vascular congestion that can produce cerebral edema. However, raising the head of the bed above 30 degrees may decrease the cerebral perfusion pressure (CPP) by lowering systemic BP. The effects of elevation of the head of the bed on the ICP and CPP ust be evaluated carefully

The nurse will explain to the patient who has a T2 spinal cord transection injury that A. use of the shoulders will be limited B. function of both arms should be retained C. total loss of respiratory function may occur D. tachycardia is common with this type of injury

B. function of both arms should be retained Rationale: The patient with a T2 injury can expect to retain full motor and sensory function of the arms. Use of only the shoulders is associated with cervical spinal injury. Loss of respiratory function occurs with cervical spine injuries. Bradycardia is associated with injuries above the T6 level

The health care provider prescribes finasteride (Proscar) for a patient who has benign prostatic hyperplasia (BPH). When teaching the patient about the drug, the nurse informs him that A. he should change position from lying to standing slowly to avoid dizziness B. his interest in sexual activity may decrease while he is taking the medication C. improvement in the obstructive symptoms should occur within about 2 weeks D. he will need to monitor his blood pressure frequently to assess for hypertension

B. his interest in sexual activity may decrease while he is taking the medication Rationale: A decrease in libido is a side effect of finasteride because of the androgen suppression that occurs with the drug. Although orthostatic hypotension may occur if the patient is also taking a medication for erectile dysfunction, it should not occur with finasteride alone. Improvement in symptoms of obstruction takes about 6 months . The medication does not cause hypertension.

The nurse is admitting a patient who has a neck fracture at the C6 level to the intensive care unit. Which assessment findings indicate neurogenic shock? A. involuntary and spastic movement B. hypotension and warm extremities C. hyperactive reflexes below the injury D. lack of sensation or movement below the injury

B. hypotension and warm extremities Rationale: Neurogenic shock is characterized by hypotension, bradycardia, and vasodilation leading to warm skin temperature. Spasticity and hyperactive reflexes do not occur at this stage of spinal cord injury. Lack of movement and sensation indicate spinal cord injury but not neurogenic shock

When obtaining a health history and physical assessment for a 36-year-old female patient with possible multiple sclerosis (MS), the nurse should A. assess for the presence of chest pain. B. inquire about urinary tract problems. C. inspect the skin for rashes or discoloration. D. ask the patient about any increase in libido.

B. inquire about urinary tract problems. Rationale: Urinary tract problems with incontinence or retention are common symptoms of MS. Chest pain and skin rashes are not symptoms of MS. A decrease in libido is common with MS

Which patient assessment will help the nurse identify potential complications of trigeminal neuralgia? A. have the patient clench the jaws B. inspect the oral mucosa and teeth C. palpate the face to compare skin temperature bilaterally D. identify trigger zones by lightly touching the affected side

B. inspect the oral mucosa and teeth Rationale: Oral hygiene is frequently neglected because of fear of triggering facial pain and may lead to gum disease, dental caries, or an abscess. Having the patient clench the facial muscles will not be useful because the sensory branches (rather than the motor branches) of the nerve are affected by trigeminal neuralgia. Light touch and palpation may be triggers for pain and should be avoided

A patient with paraplegia resulting from a T9 spinal cord injury has a neurogenic reflexic bladder. Which action will the nurse include in the plan of care? A. Teach the patient the Crede method B. instruct the patient how to self catheterize C. Catheterize for residual urine after voiding D. Assist the patiet to the toilet every 2 hours

B. instruct the patient how to self catheterize Rationale: Because the patient's bladder is spastic and will empty in response to overstretching of the bladder wall, the most appropriate method is to avoid incontinence by emptying the bladder at regular intervals through intermittent catheterization. Assisting the patient to the toilet will not be helpful because the bladder will not empty. The Crede method is more appropriate for a bladder that is flaccid, such as occurs with areflexic neurogenic bladder. Catheterization after voiding will not resolve the patient's incontinence

During the admitting neurologic examination, the nurse determines the patient has speech difficulties as well as weakness of the right arm and lower face. The nurse would expect a CT scan to show pathology in the distribution of the A. basilar artery. B. left middle cerebral artery. C. right anterior cerebral artery. D. left posterior communicating artery.

B. left middle cerebral artery. Rationale: The left middle cerebral artery feeds the lateral frontal lobes. The motor strip in this distribution controls the contralateral arm, lower half of the face and motor speech

A young patient who is trying to become pregnant asks the nurse how to determine when she is most likely to conceive. The nurse explains that A. ovulation is unpredictable unless there are regular menstrual periods B. ovulation prediction kits can provide accurate information about ovulation C. she will need to bring a specimen of cervical mucus to the clinic for testing D. she should take her body temperature daily and have intercourse when it drops

B. ovulation prediction kits can provide accurate information about ovulation Rationale: Ovulation prediction kits indicate when luteinizing hormone (LH) levels first rise. Ovulation occurs about 28 to 36 hours after the first rise of LH. This information can be used to determine the best time for intercourse. Body temperature rises at ovulation. Postcoital cervical smears are used in infertility testing, but they do not predict the best time for conceiving and are not obtained by the patient. Determination of the time of ovulation can be predicted by basal body temperature charts or ovulation prediction kits and is not dependent on regular menstrual periods

After change of shift report on the neurology unit, which patient will the nurse assess first? A. patient with Bell's palsy who has herpes vesicles in front of the ear B. patient with botulism who is drooling and experiencing difficulty swallowing C. patient with neurosyphilis who has tabes dorsalis and decreased deep tendon reflexes D. patient with an abscess caused by injectable drug use who needs tetanus immune globulin

B. patient with botulism who is drooling and experiencing difficulty swallowing Rationale: The patient's diagnosis and difficulty swallowing indicate the nurse should rapidly assess for respiratory distress. The information about the other patient is consistent with their diagnosis and does not indicate any immediate need for assessment or intervention

While the nurse performs range of motion (ROM) on an unconscious patient with increased ICP, the patient experiences severe decerebrate posturing reflexes. What should the nurse do first? A. use restraints to protect the patient from injury while posturing B. perform the exercises less frequently because posturing indicates increased ICP C. administer central nervous system (CNS) depressants to likely sedate the patient. D. continue the exercises because they are necessary to maintain musculoskeletal function

B. perform the exercises less frequently because posturing indicates increased ICP Rationale: If reflex posturing occurs during range of motion (ROM) or positioning of the patient, these activities should be done less frequently until the patient's condition stabilizes because posturing can cause increases in ICP and may indicate herniation. Neither restrains nor central nervous system (CNS) depressants would be indicated

After a 26-year-old patient has been treated for pelvic inflammatory disease, the nurse will plan to teach about A. use of hormone therapy (HT). B. potential complication of infertility C. irregularities in the menstrual cycle D. changes in secondary sex characteristics

B. potential complication of infertility Rationale: Pelvic inflammatory disease may cause scarring of the fallopian tubes and result in difficulty in fertilization or implantation of a fertilized egg. Because ovarian function is not affected, the patient will not require HT, have irregular menstrual cycles, or experience changes in secondary sex characteristics

Which action will the nurse take when caring for a patient who develops tetanus from injectable substance use? A. avoid use of sedatives B. provide a quiet environment C. provide range of motion exercises daily D. check pupil reaction to light every 4 hours

B. provide a quiet environment Rationale: In patients with tetanus, painful seizures can be precipitated by jarring, loud noises, or bright lights, so the nurse will minimize noise and avoid shining light in the patient's eyes. Range of motion exercises may also stimulate the patient and cause seizures. Although the patient has a history of injectable drug use, sedative medications, will be needed to decrease spasms

When the nurse is developing a rehabilitation plan for a 30 yera old patient with a C6 spinal cord injury, an appropriate goal is that the patient will be able to A. drive a car with powered hand controls B. push a manual wheelchair on a flat surface C. turn and reposition independently when in a bed D. transfer independently to and from a wheelchair

B. push a manual wheelchair on a flat surface Rationale: The patient with a C6 injury will be able to use the hands to push a wheelchair on flat, smooth surfaces. Because flexion of the thumb and fingers in minimal, the patient will not be able to grasp a wheelchair during transfer, drive a car with powered hand controls, or turn independently in bed

A patient with tremor is being evaluated for Parkinson's disease. The nurse explains to the patient that Parkinson's disease can be confirmed by A. CT and MRI scans B. relief of symptoms with administration of dopaminergic agents C. the presence of tremors that increase during voluntary movement D. cerebral angiogram that reveals the presence of cerebral atheroscleroris

B. relief of symptoms with administration of dopaminergic agents Rationale: Although clinical manifestations are characteristic in PD, no laboratory or diagnostic tests are specific for the condition. A diagnosis is made when the presence of tremor, rigidity, akinesia, and postural instability occur with asymmetric onset, and it is confirmed wit a positive response to antiparkinsonian drugs. Research regarding the role of genetic testing and MRI to diagnose PD is ongoing. Essential tremors increase during voluntary movement,whereas the tremors of PD are more prominent at rest

Provide emotional support to a patient with an STI by A. offering information on how safer sexual practices can prevent STIs. B. showing concern when listening to the patient who expresses negative feelings. C. reassuring the patient that the disease is highly curable with appropriate treatment. D. helping the patient who received an STI from his or her sexual partner in forgiving the partner.

B. showing concern when listening to the patient who expresses negative feelings. Rationale: The diagnosis of an STI may be met with a variety of emotions, such as shame, guilt, anger, and even a desire for vengeance. Provide counseling, and try to help the patient verbalize feelings. Support and a willingness to listen to the patient's concerns are needed

A 32 year old patient has oral contraceptives prescribed for endometriosis. The nurse will teach the patient to A. expect to experience side effects such as facial hair B. take the medication every day for the next 9 months C. take calcium supplements to prevent developing osteoporosis during therapy. D. Use a second method of contraception to ensure that she will not become pregnant

B. take the medication every day for the next 9 months Rationale: When oral contraceptives are prescribed to treat endometriosis, the patient should take the medications continuously for 9 months. Facial hair is a side effect of synthetic androgens. The patient does not need to use additional contraceptive methods. The hormones in oral contraceptives will protect against osteoporosis

A patient with a head injury has bloody drainage from the ear. What should the nurse do to determine if CSF is present in the drainage? A. examine the tympanic membrane for a tear B. test the fluid for a halo sign on a white dressing C. tests the fluid with a glucose identifying strip or stick D. collect 5 mL of fluid in a test tube and send it to the laboratory for analysis

B. test the fluid for a halo sign on a white dressing Rationale: Testing clear drainage for CSF in nasal or ear drainage may be done with a Dextrostik or Tes-Tape strip, but if blood is present, the glucose in the bood will produce an unreliable result. To test bloody drainage, the nurse should test the fluid for a "halo" or "ring" that occurs when a yellowish ring encircles blood dripped onto a white pad or towel within a few minutes

Two days following a spinal cord injury, a patient asks continually about the extent of impairment that will result from the injury. What is the best response by the nurse? A. you will have more normal function when spinal shock resolves and the reflex arc returns B. the extent of your injury cannot be determined until the secondary injury to the cord is resolved C. when your condition is more stable, an MRI will be done that can reveal the extent of the cord damage D. because long-term rehabilitation can affect the return of function, it will be years before we can tell when the complete effect will be

B. the extent of your injury cannot be determined until the secondary injury to the cord is resolved Rationale: Until the edema and necrosis at the site of the injury are resolved in 72 hours or more after the injury, it is not possible to determine how much cord damage is present from the initial injury, how much secondary injury occurred, or how much the cord was damaged by edema that extended above and below the level of the original injury. The return reflexes signals the end of spinal shock and the reflexes may be inappropriate and excessive, causing spasms that complicate rehabilitation

A patient with gonorrhea is treated with a single IM dose of ceftriaxone (Rocephin) and is given a prescription for doxycycline (Vibramycin) 100mg bid for 7 days. The nurse explains to the patient that this combination is prescribed to A. prevent reinfection during treatment B. treat any coexisting chlamydial infection C. eradicate resistant strains of N. gonorrhea D. prevent the development of resistant organisms

B. treat any coexisting chlamydial infection Rationale: Because there is a high incidence of co-infection with gonorrhea and chlamydia, patients are usually treated for both. The other explanations about the purpose of the antibiotic combination are not accurate

The nurse assessing a patient with newly diagnosed trigeminal neuralgia will ask the patient about A. visual problems caused by ptosis B. triggers leading to facial discomfort C. poor appetite caused by loss of taste D. weakness on the affected side of the face

B. triggers leading to facial discomfort Rationale: The major clinical manifestations of trigeminal neuralgia is severe facial pain triggered by cutaneous stimulation of the nerve. Ptosis, loss of taste, and facial weakness are not characteristics of trigeminal neuralgia

A patient has a 6cm thoracic aortic aneurysm that was discovered during routine chest x ray. When obtaining an admission history from the patient, it will be most important for the nurse to ask about A. low back pain B. trouble swallowing C. abdominal tenderness D. changes in bowel habits

B. trouble swallowing Rationale: Difficulty swallowing may occur with a thoracic aneurysm because of the pressure on the esophagus. The other symptoms will be important to assess for in patients with abdominal aortic aneurysms

Which information shown in the accompanying figure and obtained by the nurse about a 72 year old man who is complaining of erectile dysfunction is most important to communicate to the health care provider? History: Takes three medications for hypertension; recent knee surgery Physical Exam: Uncircumcised, foreskin easily retractable; left testis hangs lower than right; decreased pubic hair Vital Signs: Temperature 97.6; pulse 64bpm; respirations 22 breaths per minutes; blood pressure 134/70 mmHg A. Recent knee surgery B. use of antihypertensives C. low position of left testis D. Pulse and blood pressure level

B. use of antihypertensives Rationale: Many medications used for hypertension can cause erectile dysfunction. More infomation is needed regarding the specific medications. The other assessment data will not impact erectile function (recent knee surgery) or are normal for a 70 year old man (physical examination data and vital signs)

The nurse performing a focused examination to determine possible causes of infertility will assess for A. hydrocele B. varicocele C. epididymitis D. paraphimosis

B. varicocele Rationale: Persistent varicoceles are commonly associated with infertility. Hydrocele, epididymitis, and paraphimosis are not risk factors for infertility

A 48 year old male patient who has been diagnosed with gonococcal urethritis tells the nurse he had recent sexual contact with a woman but says she did not appear to have any disease. In responding to the patient, the nurse explains that A. women do not develop gonorrhea infections but can serve as carriers to spread the disease to men B. women may not be aware they have gonorrhea because they often do not have symptoms of infection C. women develop subclinical cases of gonorrhea that do not cause tissue damage or clinical manifestations D. when gonorrhea infections occur in women, the disease affects only the ovaries and not the genital organs

B. women may not be aware they have gonorrhea because they often do not have symptoms of infection Rationale: Many women with gonorrhea are asymptomatic or have minor symptoms that are overlooked. The disease may affect both the genitals and the other reproductive organs and cause complications such as pelvic inflammatory disease. Women who can transmit the disease have active infections

A 58 year old patient is on the medical surgical unit after undergoing a radical vulvectomy for vulvar carcinoma. The greatest risk to the patient at this time is A. self-care deficit B. wound infection C. inadequate nutrition D. ineffective sexual pattern

B. wound infection Rationale: Complex and meticulous wound care is needed to prevent infection and delayed wound healing. The patient may be at risk for other problems, but they are not the greatest concerns in the immediate postoperative time period

Which actions could the nurse delegate to a licensed practical/vocational (LPN/LVN) who is part of the team caring for a patient with Alzheimer's disease (select all that apply)? A. Develop a plan to minimize difficult behavior B. Administer the prescribed memantine (Namenda) C. Remove potential safety hazards from the patient's environment D. Refer the patient and caregivers to appropriate community resources E. Help the patient and caregivers choose memory enhancement methods F. Evaluate the effectiveness of the prescribed enteral feedings on patient nutrition

B.C. Administer the prescribed memantine (Namenda); Remove potential safety hazards from the patient's environment LPN/LVN education and scope of practice includes medication administration and monitoring for environmental safety in stable patients. Planning of interventions such as ways to manage behavior or improve memory, referrals, and evaluation of the effectiveness of interventions require registered nurse (RN)-level of education and scope of practice

Which nursing actions can the nurse working in a women's health clinic delegate to experienced unlicensed assistive personnel (UAP) (select all that apply) A. Call a patient with the results of an endometrial biopsy B. Assist the health care provider with performing a Pap test C. Draw blood for CA-125 levels for a patient with ovarian cancer D. Question a patient about use of medications that may cause amenorrhea E. Teach the parent of a 10 year old about human papilloma virus (HPV) vaccine Gardisil

B.C. Assist the health care provider with performing a Pap test; Draw blood for CA-125 levels for a patient with ovarian cancer Rationale: Assisting with a Pap test and drawing blood (if trained) are skills that require minimal critical thinking and judgement and can be safely delegated to a UAP. Patient teaching, calling a patient who may have questions about results of diagnostic testing, and risk factor screening all require more education and critical thinking and should be done by the registered nurse (RN)

Which clinical manifestations are seen in patients with either Buerger's disease or Raynaud's phenomenon (select all that apply)? A. Intermittent fevers B. Sensitivity to cold temperatures C. Gangrenous ulcers on fingertips D. Color changes of fingers and toes E. Episodes of superficial vein thrombosis

B.C.D. Sensitivity to cold temperatures; Gangrenous ulcers on fingertips; Color changes of fingers and toes Rationale: Both Buerger's and Raynaud's phenomenon have the following clinical manifestations in common: cold sensitivity, ischemic and gangrenous ulcers on fingertips, and color changes of the distal extremity (fingers or toes)

A patient is diagnosed with cluster headaches. The nurse knows that which characteristics are associated with this type of headache (select all that apply)? A. Family history B. Alcohol is the only dietary trigger C. Severe, sharp, penetrating head pain D. Abrupt onset lasting 5 to 180 minutes E. Bilateral pressures or tightness sensation F. May be accompanied by unilateral ptosis or lacrimation

B.C.D.F. Cluster headaches have only alcohol as a dietary trigger and have an abrupt onset lasting 5 minutes to 3 hours with severe, sharp, penetrating pain. Cluster headaches may be accompanied by unilateral ptosis, lacrimation, rhinitis, facial flushing, or pallor and commonly recur several times each day for several weeks, with months or years between clustered attacks. Family history and nausea, vomiting, or irritability may be seen with migraine headaches. Bilateral pressure occurring between migraine headaches and intermittent occurrence over long periods of time are characteristic of tension type headaches

The spouse of a 67-year-old male patient with early stage Alzheimer's disease (AD) tells the nurse, "I am exhausted from worrying all the time. I don't know what to do." Which actions are best for the nurse to take next (select all that apply)? A. Suggest that a long-term care facility be considered. B. Offer ideas for ways to distract or redirect the patient. C. Teach the spouse about adult day care as a possible respite. D. Suggest that the spouse consult with the physician for antianxiety drugs. E. Ask the spouse what she knows and has considered about dementia care options.

B.C.E. Offer ideas for ways to distract or redirect the patient; Teach the spouse about adult day care as a possible respite; Ask the spouse what she knows and has considered about dementia care options Rationale: The stress of being a caregiver can be managed with a multicomponent approach. This includes respite care, learning ways to manage challenging behaviors, and further assessment of what the spouse may already have considered for care options. The patient is in the early stages and does not need long term placement. Antianxiety medications may be appropriate, but other measures should be tried first

A patient with PAD has a nursing diagnosis of ineffective peripheral tissue perfusion, What should be included in the teaching plan for this patient? (Select all that apply) A. Apply cold compresses when the legs become swollen B. Wear protective footwear and avoid hot or cold extremes C. Walk at least 30 minutes per day, at least 3 times per week D. Use nicotine replacement therapy, as a substitute for smoking E. Inspect lower extremities for pulses, temperature, and any injury

B.C.E. Wear protective footwear and avoid hot or cold extremes; Walk at least 30 minutes per day, at least 3 times per week; Inspect lower extremities for pulses, temperature, and any injury Rationale: Protecting feet and legs from injury is important. Walking exercise increases oxygen extraction in the legs and improves skeletal muscle metabolism. The patient with PAD should walk at least 30 minutes a day, at least 3 times per week. Exercise should be stopped when pain occurs and resumed when the pain subsides. the lower extremities should be assessed at regular intervals for changes. Cold compresses and nicotine in all forms causes vasocontriction and should be avoided

Which descriptions are characteristic of encephalitis (select all that apply)? A. Increased CSF production B. is an inflammation of the brain C. almost always has a viral cause D. Most frequently caused by bacteria E. May be transmitted by insect vectors F. Involves inflammation of the tissues surrounding the brain and spinal cord

B.C.E. is an inflammation of the brain; almost always has a viral cause; may be transmitted by insect vectors Rationale: Encephalitis is usually caused by a virus that inflames the brain and can be transmitted by ticks and mosquitos. The other options are characteristics of meningitis

What are characteristics of LH? (Select all that apply) A. Maintains implanted egg B. Completes follicle maturation C. Stimulates testosterone production D. Required for female sex characteristics E. Needed for growth of mammary glands F. Called interstitial cell stimulating hormone (ICSH) in men

B.C.F. Completes follicle maturation; stimulates testosterone production; called interstitial cell stimulating hormone (ICSH) in men Rationale: LH completes follicle maturation in women and stimulates testosterone production. LH is call interstitial cell stimulating hormone (ICSH) in men. Progesterone maintains an implanted egg. Estrogen develops and maintains secondary sex characteristics of women as well as the proliferative phase of thee menstrual cycle after menstruation and the uterine changes essential to pregnancy. Prolactin stimulates the growth of the mammary glands

FSH is secreted by the anterior pituitary gland in both women and men. Which characteristics describe FSH? (Select all that apply) A. Produced by testes B. Elevated at onset of menopause C. Responsible for spermatogenesis D. Needed for male sex characteristics E. Stimulated by elevated estrogen levels F. Stimulates growth and maturity of ovarian follicles

B.C.F. Elevated at onset of menopause; responsible for spermatogenesis; stimulates growth and maturity of ovarian follicles Rationale: FSH is elevated at menopause. In men it is responsible for spermatogenesis. FSH stimulates growth and maturity of ovarian follicles. Testosterone is produced by the testes and produces male sex characteristics. Gonadotropin releasing hormone (GnRH) is stimulated by elevated estrogen levels and decreased testosterone levels.

The nurse should explain to the patient who has erectile dysfunction (ED) that (select all that apply) A. the most common cause is benign prostatic hypertrophy B. ED may be due to medications or conditions such as diabetes C. only men who are over 65 years or older benefit from PDE5 inhibitors D. there are medications and devices that can be used to help with erections E. this condition is primarily due to anxiety and best treated with psychotherapy

B.D. ED may be due to medications or conditions such as diabetes; there are medications and devices that can be used to help with erections Rationale: Erectile dysfunction may be due to a variety of causes, including medications, other chronic disease, or may be due to psychological issues. Treatment options include medications (such as PDE5 inhibitors, vacuum erection devices, or penile implants). PD5E inhibitors are not restricted to any specific age group

Which are clinical findings in a person with an acute lower extremity VTE (select all that apply)? A. Pallor and coolness of foot and calf B. Mild to moderate calf pain and tenderness C. Grossly diminished or absent pedal pulses D. Unilateral edema and induration of the thigh E. Palpable cord along a superficial varicose vein

B.D. Mild to moderate calf pain and tenderness; Unilateral edema and induration of the thigh Rationale: The patient with lower extremity venous thromboembolism (VTE) may or may not have unilateral leg edema, extremity pain, a sense of fullness in the thigh or calf, paresthesias, warm skin, erythema, or a systemic temperatures greater than 100.4. If the calf is involved, it may be tender to palpation

The nurse explains to the patient with chronic bacterial prostatitis who is undergoing antibiotic therapy that (select all that apply) A. all patients require hospitalization B. Pain will lessen once treatment has ended C. course of treatment is generally 2 to 4 weeks D. Long term therapy may be indicated in immunocompromised patient E. If the condition is unresolved and untreated, he is at risk for prostate cancer

B.D. pain will lessen once treatment has ended; long term therapy may be indicated in immunocompromised patient Rationale: Patients with chronic bacterial prostatitis are usually given antibiotics for 4 to 12 weeks. Antibiotics may be given for a lifetime if the patient is immunocompromised. Although patients with chronic bacterial prostatitis tend to experience much discomfort, the pain resolves as the infection is treated. If the patient with acute bacterial prostatitis has high fever or other signs of impending sepsis, hospitalization,m and IV antibiotics are prescribed

A primary HSV infection differs from recurrent HSV episodes in that (select all that apply) A. only primary infections are sexually transmitted B. symptoms are less severe during recurrent episodes. C. transmission of the virus to a fetus is less likely during primary infection. D. systemic manifestations such as fever and myalgia are more common in primary infection. E .lesions from recurrent HSV are more likely to transmit the virus than lesions from primary HSV.

B.D. symptoms are less severe during recurrent episodes; systemic manifestations such as fever and myalgia are more common in primary infection Rationale: Primary herpes simplex virus (HSV) infections (i.e. genital herpes) tend to be associated with local inflammation and pain, and they are accompanied by systemic manifestations of fever, headache, malaise, myalgia, and regional lymphadenopathy. The symptoms of recurrent genital herpes episodes are less severe, and the lesions usually heal within 8 to 12 days

Which therapies for BPH are done on an outpatient basis (select all that apply)? A. Intraprostatic urethral stents B. Transurethral needle ablation (TUNA) C. Photovaporization of the prostate (PVP) D. Transurethral incision of prostate (TUIP) E. Transurethral microwave therapy (TUMT)

B.D.E. Transurethral needle ablation (TUNA), Transurethral incision of prostate (TUIP), Transurethral microwave therapy (TUMT) Rationale: TUNA, TUIP, TUMT are currently done on an outpatient basis or in a HCP's office

Which CNs are involved with oblique eye movements (select all that apply)? A. Optic (CN II) B. Trochlear (CV IV) C. Trigeminal (C V) D. Abducens (CN VI) E. Oculomoter (CN III)

B.D.E. Trochlear (CN IV); Abducens (CN VI); Oculomoter (CN III) Rationale: The trochlear (CN IV), abducens (CN VI), and oculomotor (CN III) nerves cause oblique eye movement. The optic nerve (CN II) is for vision. The trigeminal (CN V) provides sensation from the face and the motor function of mastication

What are characteristics of peripheral artery disease (Select all that apply)? A. Pruritus B. Thickened, brittle nails C. Dull ache in calf or thigh D. Decreased peripheral pulses E. Pallor on elevation of the legs F. Ulcers over bony prominences on toes and feet

B.D.E.F. Thickened, brittle nails; Decreased peripheral pulses; pallor on elevation of the legs; ulcers over bony prominences on toes and feet Rationale: PAD is manifested as thick, brittle nails; decreased peripheral pulses, pallor when the legs are elevated; ulcers over bony prominences on the toes and feet; and paresthesia. The other options are characteristic of venous disease and paresthesia could occur with venous thromboembolism (VTE)

Which conditions characterize critical limb ischemia (select all that apply)? A. cold feet B. Arterial leg ulcers C. Venous leg ulcers D. Gangrene of the leg E. No palpable peripheral pulses F. Rest pain lasting more than 2 weeks

B.D.F. Arterial leg ulcers; Gangrene of the leg; Rest pain lasting more than 2 weeks Rationale: Arterial leg ulcers and/or gangrene of the leg due to PAD and chronic ischemic rest pain lasting more than 2 weeks characterize critical limb ischemia. Optimal therapy is revascularization via bypass surgery

For a patient with moderate cognitive impairment, the HCP is trying to differentiate between a diagnosis of dementia and dementia with Lewy bodies (DLB). What observation by the nurse support a diagnosis of DLB (select all that apply)? A. Tremors B. Fluctuating cognitive ability C. Disturbed behavior, sleep, and personality D. Symptoms of pneumonia, including congested lung sounds E. Bradykinesia, rigidity, and postural instability without tremor

B.E. Fluctuating cognitive ability; Bradykinesia, rigidity, and postural instability without tremor Rationale: Dementia with Lewy bodies (DLB) is diagnosed with dementia with the following symptoms; (1) exttrapyramidal signs such as bradykinesia, rigidity, and postural instability but not always a tremor, (2) fluctuating cognitive ability, and (3) hallucinations. The extrapyramidal signs plus tremors would more likely indicate Parkinson's disease. Disturbed behavior, sleep, personality, and eventually memory are characteristics of frontotemporal lobe degeneration (FTLD)

What are causes of vasogenic cerebral edema (select all that apply)? A. Hydrocephalus B. Ingested toxins C. Destructive lesions or trauma D. Local disruption of cell membranes E. Fluid flowing from intravascular to extravascular space

B.E. Ingested toxins; Fluid flowing from intravascular to extravascular space Rationale: A variety of insults, such as brain tumors, abscesses, and ingested toxins, lead to vasogenic cerebral edema. It is the most common type of cerebral edema. It is the most common type of cerebral edema and characterized by leakage of macromolecules from the capillaries into the surrounding extracellular space. This results in an osmotic gradient that favors the flow of fluid from the intravascular to the extravascular space. Hydrocephalus causes interstitial cerebral edema. Destructive lesions or trauma destroy cell membranes and cause cytotoxic cerebral edema

Which question should the nurse ask when assessing a 60 year old patient who has a history of benign prostatic hyperplasia (BPH)? A. "Have you noticed any unusual discharge from your penis?" B. "Has there been any change in your sex life in the past year?" C. "Has there been a decrease in the force of your urinary system?" D. "Have you been experiencing any difficulty in achieving an erection?"

C. "Has there been a decrease in the force of your urinary system?" Rationale: Enlargement of the prostate blocks the urethra, leading to urinary changes such as a decrease in the force of the urinary stream. The other questions address possible problems with infection or sexual difficulties, but they would not be helpful in determining whether there were functional changes caused by BPH.

When a 31-year-old male patient returns to the clinic for follow-up after treatment for gonococcal urethritis, a purulent urethral discharge is still present. When trying to determine the reason for the recurrent infection, which question is most appropriate for the nurse to ask the patient? A. "Did you take the prescribed antibiotic for a week?" B. "Did you drink at least 3 quarts of fluids every day?" C. "Were your sexual partners treated with antibiotics?" D. "Do you wash your hands after using the bathroom?"

C. "Were your sexual partners treated with antibiotics?" Rationale: A common reason for recurrence of symptoms is reinfection because infected partners have not been simultaneously treated. Because gonorrhea is treated with one dose of antibiotic, antibiotic therapy for a week is not needed. An adequate fluid intake is important, but a low fluid intake is not a likely cause for failed treatment. Poor hygiene may cause complications such as ocular trachoma but will not cause a failure of treatment

A 47 year old patient asks whether she is going into menopause if she has not had a menstrual period for 3 months. Which response by the nurse is appropriate? A. "Have you thought about using hormone replacement therapy?" B. "Most women feel a little depressed about entering menopause." C. "What was your menstrual pattern before your periods stopped?" D. "Because you are in your mid 40s, it is likely that you are menopausal."

C. "What was your menstrual pattern before your periods stopped?" Rationale: The initial response by the nurse should be to assess the patient's baseline menstrual pattern. Although many women do enter menopause in the mid 40s, more information about this patient is needed before telling her that it is likely she is menopausal. Although hormone therapy may be prescribed, further assessment of the patient is needed before discussing therapies for menopause. Because the response to menopause is very individual, the nurse should not assume that the patient is experiencing any adverse emotional reactions

Which patient in the women's health clinic will the nurse expect to teach about an endometrial biopsy? A. A 55 year old patient who has 3 to 4 alcoholic drinks each day B. A 35 year old patient who has used oral contraceptives for 15 years C. A 25 year old patient who has a family history of hereditary nonpolyposis colorectal cancer D. A 45 year old patient who has had six previous full term pregnancies and two spontaneous abortions

C. A 25 year old patient who has a family history of hereditary nonpolyposis colorectal cancer Rationale: Patients with a personal or family history of hereditary nonpolyposis colorectal cancer are at increased risk for endometrial cancer. Alcohol addiction does not increase this risk. Multiple pregnancies and oral contraceptive use offer protection from endometrial cancer

The following patients call the outpatient clinic. Which phone call should the nurse return first? A. A 44 year old patient who has a bloody discharge after a hysteroscopy earlier today B. A 64 year old patient who is experiencing shoulder pain after a laparoscopy C. A 34 year old patient who is short of breath after having a pelvic CT with contrast dye D. A 54 year old patient who has severe breast tenderness following a needle aspiration breast biopsy

C. A 34 year old patient who is short of breath after having a pelvic CT with contrast dye Rationale: The patient's dyspnea suggests a delayed reaction to the iodine dye used for the CT scan. The other patient's symptoms are not unusual after the procedures they had done.

Following an ascending aortic aneurysm, what is an important finding that the nurse should report immediately to the HCP? A. Shallow respirations and poor coughing B. Decreased drainage from the chest tubes C. A change in the level of consciousness (LOC) an inability to speak D. Lower extremity pulses that are decreased from the preoperative baseline

C. A change in the level of consciousness (LOC) an inability to speak Rationale: During repair of an ascending aortic aneurysm, the blood supply to the carotid arteries may be interrupted, leading to neurologic complications manifested by a decreased level of consciousness (LOC) and altered pupil responses to light as well as changes in facial symmetry, speech, upper extremity movement, and hand grasp quality. The thorax is opened for ascending aortic surgery, and shallow breathing, poor cough, and decreasing chest drainage are expected. Lower limb pulses may normally be decreased or absent for a short time following surgery

Which statement accurately describes the female reproductive system? A. Fertilization of an ovum by sperm occurs in the uterus B. Only the ectocervix is used for obtaining Papanicolaou (Pap) tests C. A middle aged woman is considered to be in menopause when she has not had a menstrual period for 1 year D. The normal process of reabsorption of immature oocytes throughout the female life span is caused by follicle stimulating hormone (FSH) and luteinizing hormone (LH)

C. A middle aged woman is considered to be in menopause when she has not had a menstrual period for 1 year Rationale: Menopause is identified after 1 year of amenorrhea. The ovum is fertilized by the sperm in the fallopian tube. A Papnicolaou (Pap) test includes cells from both the endocervix and ectocervix. The reabsorption of oocytes by the body is called atresia. Follicle stimulating hormone (FSH) stimulates the initial stage of folicular maturation, while luteinizing hormone (LH) must be present for complete maturation and ovulation to occur.

Although an 18-year-old girl knows that abstinence is one way to prevent STIs, she does not consider that as an alternative. She asks the nurse at the clinic if there are other measures for preventing STIs. What should the nurse teach her? A. Abstinence is the only way to prevent STIs. B. Voiding immediately after intercourse will decrease the risk for infection. C. A vaccine can prevent genital warts and cervical cancer caused by some strains of HPV. D. Thorough hand washing after contact with genitals can prevent oral-genital spread of STIs.

C. A vaccine can prevent genital warts and cervical cancer caused by some strains of HPV. Rationale: A vaccine is available for HPV types 6, 11, 16, 18 that protects against genital warts and cervical cancer. Although sexual abstinence is the most certain method of avoiding all STIs, it is not usually a feasible alternative. Undamaged condoms also protect against infection. Conscientious hand washing and voiding after intercourse are positive hygienic measures that will help to prevent secondary infections but will not prevent STIs

A premarital blood test of syphilis reveals that a woman has a positive Venereal Disease Research Laboratory (VDRL) test. How should the nurse advise the patient? A. A single dose of penicillin will cure the syphilis B. She should question her fiance about prior sexual contacts C. Additional testing to detect specific antitreponemal antibodies is necessary D. A lumbar puncture to evaluate cerebrospinal fluid (CSF) is necessary to rule out active syphiliis

C. Additional testing to detect specific antitreponemal antibodies is necessary Rationale: Many other disease or conditions may cause false positive test results on notreponemal Venereal Disease Research Laboratory (VDRL) or rapid plasma reagent (RPR) tests and additional testing is needed before a diagnosis is confirmed or treatment is administered. The diagnosis is confirmed by specific treponemal tests, such as the fluorescent antibody absorption (FTA-Abs) test or the TP-PA test. Analysis of CSF is used to diagnose asymptomatic neurosyphilis

Which prescribed intervention will the nurse implement first for a patient in the emergency department who is experiencing continuous tonic-clonic seizures? A. Give phenytoin (Dilantin) 100 mg IV. B. Monitor level of consciousness (LOC). C. Administer lorazepam (Ativan) 4 mg IV. D. Obtain computed tomography (CT) scan

C. Administer lorazepam (Ativan) 4 mg IV. Rationale: To prevent ongoing seizures, the nurse should administer rapidly acting antiseizure medication such as benzodiazepines. A CT scan is appropriate, but prevention of any seizure activity during the CT scan is necessary. Phenytoin will also be administered, but it is not rapidly acting. Patients who are experiencing tonic clonic seizures are nonresponsive, although the nurse should assess LOC after the seizure

What are the priority nursing interventions after an abdominal aortic aneurysm repair? A. Assessment of cranial nerves and mental status B. Administration of heparin and monitoring of PT C. Administration of IV fluids and monitoring of kidney function D. Elevation of the legs and application graduated compression stockings

C. Administration of IV fluids and monitoring of kidney function Rationale: Postoperative priorities include administration of IV fluids and maintenance of renal perfusion. An adequate blood pressure is important for maintaining graft patency, and administration of IV fluids and blood components (as indicated) is essential for adequate blood flow. The nurse should evaluate renal function by measuring hourly urine output and monitoring daily blood urea nitrogen (BUN) and serum creatinine levels. Irreversible renal failure may occur after aortic surgery, particularly in individuals at high risk

A 22 year old patient seen at the health clinic with a severe migraine headache tells the nurse about having similar headaches recently. Which initial action should the nurse take? A. Teach about the use of triptan drugs B. Refer the patient for stress counseling C. Ask the patient to keep a headache diary D. Suggest the use of muscle relaxation techniques

C. Ask the patient to keep a headache diary Rationale: The initial nursing action should be further assessment of the precipitating causes of the headaches, quality, and location of pain. Stress reduction, muscle relaxation, and the triptan drugs may be helpful, but more assessment is needed first

When administering the Mini-Cog exam to a patient with possible Alzheimer's disease, which action will the nurse take? A. Check the patient's orientation to time and date B. Obtain a list of the patient's prescribed medications C. Ask the person to use a clock drawing to indicate a specific time D. Determine the patient's ability to recognize a common object such as a pen

C. Ask the person to use a clock drawing to indicate a specific time Rationale: In the Mini-Cog, patients illustrate a specific time stated by the examiner by drawing the time on a clock face. The other actions may be included n assessment for Alzheimer's disease but are not part of the Mini-Cog exam

A patient admitted with dermal ulcers who has a history of a T3 spinal cord injury tells the nurse, I have a pounding headache and I feel sick to my stomach. Which action should the nurse take first? A. Check for a fecal impaction B. Give the prescribed antiemetic C. Assess the blood pressure (BP) D. Notify the health care provider

C. Assess the blood pressure (BP) Rationale: The BP should be assessed immediately in a patient with an injury at T6 level or higher who complains of a headache to determine if autonomic hyperreflexia is occurring. Notification of the patient's health care provider is appropriate after the BP is obtained. Administration of an antiemetic is indicated if autonomic hyperreflexia is ruled out as the cause of nausea. After checking the BP, the nurse may assess for a fecal impaction using lidocaine jelly to prevent further increased BP

A 44 year old woman undergoing a total abdominal hysterectomy asks whether she will need to take estrogen until she reaches the age of menopause. What is the best response by the nurse? A. Yes, it will help to prevent the more intense symptoms caused by surgically induced menopause B. Your are close enough to normal menopause that you probably won't need additional estrogen C. Because your ovaries won't be removed, they will continue to secrete estrogen until your normal menopause D. There are so many risks associated with estrogen replacement therapy that it is best to begin menopause now

C. Because your ovaries won't be removed, they will continue to secrete estrogen until your normal menopause Rationale: A total hysterectomy involves the removal of the uterus and cervix, but the fallopian tubes and ovaries are left intact. Although menstruation is terminated, normal ovarian production of estrogen continues. When the uterus, tubes, and ovaries are removed, it is called a total hysterectomy and bilateral salpingo-oophorectomy (TAH-BSO)

The nurse teaches the patient taking antiseizure drugs that this method is most commonly used to measure compliance and monitor for toxicity. A. A daily seizure log B. Urine testing for drug levels C. Blood testing for drug levels D. Monthly electroencephalography (EEG)

C. Blood testing for drug levels Rationale: Serum levels of antiseizure drugs are monitored regularly to maintain therapeutic levels of the drug, above which patients are likely to experience toxic effects and below which seizures are likely to occur. Many newer drugs do not require a drug level monitoring because of large therapeutic ranges. A daily seizure log and urine testing for drug levels will not measure compliance or testing for drug levels will not measure compliance or monitor for toxicity. EEGs have limited value in diagnosis of seizures and even less value in monitoring seizure control

Which information will the nurse include when teaching a patient who has developed a small vesicovaginal fistula 2 weeks into the postpartum period? A. Take stool softeners to prevent fecal contamination of the vagina B. Limit fluid intake to minimize the quantity of urinary drainage C. Change the perineal pad frequently to prevent perineal skin breakdown D. Call the health care provider immediately if urine drains from the vagina

C. Change the perineal pad frequently to prevent perineal skin breakdown Rationale: Because urine will leak from the vagina, the patient should plan to use perineal pads and change them frequently. A high fluid intake is recommended to decrease the risk for urinary tract infections. Fecal contamination is no a concern with vesicovaginal fistulas

A patient with increased intracranial pressure after a head injury has a ventriculostomy in place. Which action can the nurse delegate to unlicensed assistive personnel (UAP) who regularly work in the intensive care unit? A. Document the intracranial pressure every hour B. Turn and reposition the patient every 2 hours C. Check capillary blood glucose level every 6 hours D. Monitor cerebrospinal fluid color and volume hourly

C. Check capillary blood glucose level every 6 hours Rationale: Experienced UAP can obtain capillary blood glucose levels when they have been trained and evaluated in the skill. Monitoring and documentation of cerebrospinal fluid (CSF) color and intracranial pressure (ICP) require registered nurse (RN)- level education and scope of practice. Although repositioning patients is frequently delegated to UAP, repositioning a patient with a ventriculostomy is complex and should be supervised by the RN

Which nursing action will be included in the plan of care for a patient who has had cerebral angiography? A. Monitor for headache and photophobia B. Keep patient NPO until gag reflex returns C. Check pulse and blood pressure frequently D. Assess orientation to person, place, and time

C. Check pulse and blood pressure frequently Rationale: Because a catheter is inserted into an artery (e.g. femoral artery) during cerebral angiography, the nurse should assess for bleeding after this procedure that can affect pulse and blood pressure. The other nursing assessments are not needed after angiography

A patient who is scheduled for a routine gynecologic examination tells the nurse that she has had intercourse during the past year with several men. The nurse will plan to teach about the reason for A. contraceptive use B. antibiotic therapy C. Chlamydia testing D. pregnancy testing

C. Chlamydia testing Chlamydia testing is recommended annually for women with multiple sex partners. There is no indication that the patient needs teaching about contraceptives, pregnancy testing, or antibiotic therapy.

A patient who has been recently diagnosed with benign prostatic hyperplasia (BPH) tells the nurse that he does not want to have a transurethral resection of the prostate (TURP) because it might affect his ability to have sexual intercourse. Which action should the nurse take? A. Discuss alternative methods of sexual expression B. Teach about medication for erectile dysfunction C. Clarify that TURP does not commonly affect erection D. Offer reassurance that fertility is not affected by TURP

C. Clarify that TURP does not commonly affect erection Rationale: ED is not a concern with TURP, although retrograde ejaculation is likely, and the nurse should discuss this with the patient., Erectile function is not usually affected by a TURP, so the patient will not need information about penile implants or reassurance that other forms of sexual expression may be used. Because the patient has not asked about fertility, reassurance about sperm production does not address his concerns

Fertility and normal reproductive function can be maintained when a cancer of the cervix is treated with which therapy? A. External radiation therapy B. Internal radiation implants C. Conization or laser surgery D. Cryotherapy or subtotal hysterectomy

C. Conization or laser surgery Rationale: Conization (an excision of a cone shaped section of the cervix) and laser treatment both are effective to locally remove or destroy malignant cells of the cervix and preserve fertility. Radiation treatments frequently impair ovarian and uterine function and lead to sterility. A subtotal hysterectomy would be contraindicated in the treatment of cervical cancer because the cervix would be left intact in this procedure

The nurse in the infertility clinic is explaining in vitro fertilization (IVF) to a couple. The woman tells the nurse that they cannot afford IVF on her husband's salary. The man replies that if his wife worked outside the home, they would have enough money. Which nursing diagnosis is appropriate? A. Decisional conflict related to inadequate financial resources B. Ineffective sexuality patterns related to psychological stress C. Defensive coping related to anxiety about lack of conception D. Ineffective denial related to frustration about continued infertility

C. Defensive coping related to anxiety about lack of conception Rationale: The statements made by the couple are consistent with the diagnosis of defensive coping. No data indicate that ineffective sexuality and ineffective denial are problems. Although the couple is quarreling about finances, the data do not provide information indicating that the finances are inadequate

Which observation of the patient made by the nurse is most indicative of Parkinson's disease? A. Large, embellished handwriting B. Weakness of one leg resulting in a limping walk C. Difficulty rising from a chair and beginning to walk D. Onset of muscle spasms occurring with voluntary movement

C. Difficulty rising from a chair and beginning to walk Rationale: The akinesia of PD prevents automatic movements and activities such as beginning to walk, rising from a chair, or even swallowing saliva cannot be executed unless they are consciously willed. Handwriting is affected by the tremor and results in the writing trailing off at the end of words. Specific limb weakness and muscle spasms are not characteristic of PD

A patient at the clinic for a routine health examination mentions that she is exhausted because her legs bother her so much at night that she cannot sleep. The nurse questions the patient further about her leg symptoms with what knowledge about restless leg syndrome? A. The condition can be readily diagnosed with EMG B. Other more serious nervous system dysfunctions may be present C. Dopaminergic agents are often effective in managing the symptoms D. Symptoms can be controlled by vigorous exercise of the legs during the day

C. Dopaminergic agents are often effective in managing the symptoms Rationale: Restless legs syndrome that is not related to other pathologic processes, such as diabetes mellitus or rheumatic disorders, may be caused by a dysfunction in the basal ganglia circuits that use the neurotransmitter dopamine, which controls movements. Dopamine precursors and dopamine agonists, such as those used for parkinsonism, are effective in managing sensory and motor symptoms. Polysomnography studies during sleep are the only test that have diagnostic value, and although exercise should be encouraged, excessive leg exercise does not have an effect on the symptoms

A 46 year old service counter worker undergoes sclerotherapy for treatment of superficial varicose veins at an outpatient center. Which instructions should the nurse provide to the patient before discharge? A. Sitting at the work counter, rather than standing, is recommended B. Exercise, such as walking or jogging, can cause recurrence of varicosities C. Elastic compression stockings should be applied before getting out of bed D. Taking an aspirin daily will help prevent clots from forming around venous valves.

C. Elastic compression stockings should be applied before getting out of bed Rationale: Elastic compression stockings are applied with the legs elevated to reduce pressure in the lower legs. Walking is recommended to prevent recurrent varicosities. Sitting and standing are both risk factors for varicose veins and venous insufficiency. An aspirin a day is not adequate to prevent venous thrombosis and would not be recommended for a patient who had just had sclerotherapy

A patient with spinal cord injury is experiencing severe neurologic deficits. What is the most likely mechanics of injury for this patient? A. Compression B. Hyperextension C. Flexion rotation D. Extension rotation

C. Flexion rotation Rationale: The major mechanisms of SCI are flexion, hyperextension, flexion rotation, extension rotation, and compression. The flexion rotation injury is the most unstable because spinal ligaments are torn. This injury most often contributes to severe neurologic deficits

A patient is considering the use of combined estrogen-progesterone hormone replacement therapy (HRT) during menopause. Which information will the nurse include during their discussion? A. Use of estrogen-containing vaginal creams provides the same benefits as oral HRT B. Increased risk of colon cancer in women taking HRT requires frequent colonoscopy C. HRT decreases osteoporosis risk and increases the risk for cardiovascular disease and breast cancer D. Use of HRT for up to 10 years to prevent symptoms such as hot flashes is generally considered safe

C. HRT decreases osteoporosis risk and increases the risk for cardiovascular disease and breast cancer Rationale: Data from the Women's Health Initiative indicate an increased risk for cardiovascular disease and breast cancer in women taking combination HRT but a decrease in hip fractures. Vaginal creams decrease symptoms related to vaginal atrophy and dryness, but they do not offer the other benefits of HRT, such as decreased hot flashes. Most women who use HRT are placed on short term treatment and are not treated for up to 10 years. The incidence of colon cancer decreases in women taking HRT

On admission to the ambulatory surgical center, a patient with BPH informs the nurse that he is going to have a laser treatment of his enlarged prostate. The nurse plans patient teaching with the knowledge that the patient will need to know what? A. The effects of general anesthesia B. The possibility of short-term incontinence C. Home management of an indwelling catheter D. Monitoring for postoperative urinary retention

C. Home management of an indwelling catheter Rationale: Because of edema, urinary retention, and delayed sloughing of tissue that occurs with a laser prostatectomy, the patient will have a postprocedure urinary catheter for up to 7 days. The procedure is done under local anesthetic, and incontinence or urinary retention is not usually a problem with laser prostatectomy

Which chronic neurologic disorder involves a deficiency of the neurotransmitters acetylcholine and γ-aminobutyric acid (GABA) in the basal ganglia and extrapyramidal system? A. Myasthenia gravis B. Parkinson's disease C. Huntington's disease D. Amyotrophic lateral sclerosis (ALS)

C. Huntington's disease Rationale: Huntington's disease (HD) involves deficiency of acetylcholine and y-aminobutyric acid (GABA) in the basal ganglia and extrapyramidal system that causes the opposite symptoms of parkinsonism. Myasthenia gravis involves autoimmune antibody destruction of cholinergic receptor at the neuromuscular junction. Amyotrophic lateral sclerosis (ALS) involves degeneration of motor neurons in the brainstem and spinal cord

Which CN is tested with tongue protrusion? A. Vagus (CN X) B. Olfactory (CN I) C. Hypoglassal (CN XII) D. Glosspharyngeal (CN IX)

C. Hypoglassal (CN XII) Rationale: The hypoglossal (CN XII) is tested with tongue protrusion. The vagus (CN X) and glossopharyngeal (CN IX) nerves are tested with the gag reflex. The olfactory (CN I) nerve is tested with odor identification

After the nurse has taught a patient with a newly diagnosed sexually transmitted infection (STI) about expedited partner therapy, which patient statement indicates that the teaching has been effective? A. I will tell my partner that it is important to be examined at the clinic B. I will have my partner take the antibiotics of any STI symptoms occur C. I will make sure that my partner takes all of thee prescribed medication D. I will have my partner use a condom until I have finished the antibiotics

C. I will make sure that my partner takes all of thee prescribed medication Rationale: With expedited partner therapy, the patient is given a prescription or medications for the partner. The partner does not need to be evaluated by the health care provider but is presumed to be infected and should be treated concurrently with the patient. Use of a condom will not treat the presumed STI in the partner

Which statement by a 24 year old patient indicates that the nurse's teaching about management of primary genital herpes has been effective? A. I will use acyclovir ointment on the area to relieve the discomfort B. I will use condoms for intercourse until the medication is all gone C. I will take the acyclovir (Zovirax) every 8 hours for the next week D. I will need to take all of the medication to be sure the infection is cured

C. I will take the acyclovir (Zovirax) every 8 hours for the next week Rationale: The treatment regimen for primary genital herpes infections includes acyclovir 400mg 3 times daily for 7 to 10 days. The patient is taught to abstain from intercourse until the lesions are gone. (Condoms should be used even when the patient is asymptomatic). Acyclovir ointment is not effective in treating lesions or reducing pain. Herpes infection is chronic and recurrent

A patient admitted to the hospital following a generalized tonic clonic seizure asks the nurse what caused the seizure. What is the best response by the nurse? A. So many factors can cause epilepsy that it is impossible to say what caused your seizure. B. Epilepsy is an inherited disorder. Does anyone else in your family have a seizure disorder C. In seizures, some type of trigger causes sudden, abnormal bursts of electrical brain activity D. Scar tissue in the brain alters the chemical balance, creating uncontrolled electrical discharges

C. In seizures, some type of trigger causes sudden, abnormal bursts of electrical brain activity Rationale: A seizure is a paroxysmal, uncontrolled discharge of neurons in the brain, which interrupts normal function, but the factor that causes the abnormal firing is not clear. Seizures may be precipitated by many factors, and although scar tissue may make the brain neurons more likely to fire, it is not the usual cause of seizures. Epilepsy is established only by a pattern of spontaneous, recurring seizures

A patient with Parkinson's disease is started on levodopa. What should the nurse explain about this drug? A. It stimulates dopamine receptors in the basal ganglia B. It promotes the release of dopamine from brain neurons C. It is a precursor of dopamine that is converted to dopamine in the brain D. It prevents the excessive breakdown of dopamine in the peripheral tissues

C. It is a precursor of dopamine that is converted to dopamine in the brain Rationale: Peripheral dopamine does not cross the blood brain barrier, but its precursor, levodopa, is able to enter the brain, where it is converted to dopamine, increasing the supply that is deficient in PD. Bromocriptine is used to treat PD to stimulate dopamine receptors in the basal ganglia. Amantadine stimulates dopamine release and blocks the reuptake of dopamine into presynaptic neurons. Carbidopa and entacapone are agents that are usually administered with levodopa to prevent the levodopa from being metabolized in the peripheral tissues before it can reach the brain

A patient who is unconscious has ineffective cerebral tissue perfusion and cerebral tissue swelling. Which nursing intervention will be included in the plan of care? A. Encourage coughing and deep breathing B. Position the patient with knees and hips flexed C. Keep the head of the bed elevated to 30 degrees D. Cluster nursing interventions to provide rest periods

C. Keep the head of the bed elevated to 30 degrees Rationale: The patient with increased intracranial pressure (ICP) should be maintained in the head up position to help reduce ICP. Extreme flexion of the hips and knees increases abdominal pressure, which increases ICP. Because the stimulation associated with nursing interventions increases ICP, clustering interventions will progressively elevate ICP. Coughing increases intrathoracic pressure and ICP

For what is the serum estradiol laboratory test used? A. Detect pregnancy B. Detect prostate cancer C. Measure ovarian function D. Identify secondary gonadal failure

C. Measure ovarian function Rationale: Serum estradiol measures ovarian function to assess estrogen-secreting tumors and precocious female puberty or to confirm perimenopausal status in women. It may also be indicative of testicular tumors in men. Pregnancy is detected with urinary or serum human chorionic gonadotropin (hCG). Prostate cancer is detected with prostate specific antigen (PSA). Gonadal failure secondary to pituitary dysfunction is identified with urine or serum FSH studies

During assessment of a patient admitted to the hospital with an acute exacerbation of MS, what should the nurse expect to find? A. Tremors, dysphagia, and ptosis B. Bowel and bladder incontinence and loss of memory C. Motor impairment, visual disturbances, and paresthesias D. Excessive involuntary movements, hearing loss, and ataxia

C. Motor impairment, visual disturbances, and paresthesias Rationale: Specific neurologic dysfunction of MS is caused by destruction of myelin and replacement with glial scar tissue at specific areas in the nervous system. Motor, sensory, cerebellar, and emotional dysfunctions, including paresthesias as well as patchy blindness, blurred vision, pain radiating along the dermatome of the nerve, ataxia, and severe fatigue, are the most common manifestations of MS. Constipation and bladder dysfunctions, short term memory loss, sexual dysfunction, anger and depression or euphoria may also occur. Excessive involuntary movements and tremors are not seen in MS

Which assessment finding for a patient who has been admitted with a right calf venous thromboembolism (VTE) requires immediate action by the nurse? A. Erythema of right lower leg B. Complaint of right calf pain C. New onset of shortness of breath D. Temperature of 100.4

C. New onset of shortness of breath Rationale: New onset of dyspnea suggests a pulmonary embolus, which will require rapid actions such as O2 administration and notification of the health care provider. The other findings are typical of VTE

The nurse is preparing to admit a newly diagnosed patient and experiencing tonic clonic seizures. What could the nurse delegate to unlicensed assistive personnel (UAP)? A. Complete the admission assessment B. Assess the details of the seizure event C. Obtain the suction equipment from the supply cabinet D. Place a padded tongue blade on the wall above the patient's bed

C. Obtain the suction equipment from the supply cabinet Rationale: The unlicensed assistive personnel (UAP) is able to obtain equipment from the supply cabinet or department. The RN may need to provide a list of equipment and should set up the equipment and ensure proper functioning. The RN is responsible for the initial history and assessment as well as assessing and documenting seizure events. Padded tongue blades are no longer used, and no effort should be made to place anything in the patient's mouth during a seizure

Which information about a patient who has a new prescription for phenytoin (Dilantin) indicates that the nurse should consult with the health care provider before administration of the medication? A. Patient has tonic clonic seizures B. Patient experiences an aura before seizures C. Patient has minor elevations in the liver function tests D. Patient's most recent blood pressure is 156/92 mmHg

C. Patient has minor elevations in the liver function tests Rationale: Many older patients (especially with compromised liver function) may not be able to metabolize phenytoin. The health care provider may need to choose another antiseizure medication. Phenytoin is an appropriate medication for patients with tonic clonic seizures, with or without aura. Hypertension is not a contraindication for phenytoin therapy

Which hospitalized patient will the nurse assign to the room closest to the nurses' station? A. Patient with Alzheimer's disease who has long-term memory deficit B. Patient with vascular dementia who takes medications for depression C. Patient with new-onset confusion, restlessness, and irritability after surgery D. Patient with dementia who has an abnormal Mini-Mental State Examination

C. Patient with new-onset confusion, restlessness, and irritability after surgery Rationale: This patient's history and clinical manifestations are consistent with delirium. The patient is at risk for safety problems and should be placed near the nurses' station for ongoing observation. The other patients have chronic symptoms that are consistent with their diagnoses but are not at immediate risk for safety issues

A disturbed body image needing nursing interventions to assist the patient and family to cope may be seen in any patient undergoing gynecologic surgery. With which surgery will this most likely be expected to occur? A. Vaginectomy B. Hemivulvectomy C. Pelvic exenteration D. Radical hysterectomy

C. Pelvic exenteration Rationale: A pelvic exenteration is the most radical gynecologic surgery and results in removal of the uterus, ovaries, fallopian tubes, vagina, bladder, urethra, and pelvic lymph nodes and, in some situations, also the descending colon, rectum, and anal canal. There are urinary and fecal diversions on the abdominal wall, the absence of a vagina, and the onset of menopausal symptoms al, all of which result in severe altered body structure and changes in body image. The patient and family will need much understanding and support during the long recovery period, including verbalization of feelings

A patient with Alzheimer's disease (AD) who is being admitted to a long term care facility has had several episodes of wandering away from home. Which action will the nurse include in the plan of care? A. Reorient the patient several times daily B. Have the family bring in familiar items C. Place the patient in a room close to the nurse's station D. Ask the patient why the wandering episodes have occurred

C. Place the patient in a room close to the nurse's station Rationale: Patients at risk for problems with safety require close supervision. Placing the patient near the nurse's station will allow nursing staff to observe the patient more closely. The use of why questions can be frustrating for patients with AD because they are unable to understand clearly or verbalize the reason for wandering behaviors. Because of the patient's short term memory loss, reorientation will not help prevent wandering behavior. Because the patient had wandering behavior at home, familiar objects will not prevent wandering

A patient has an incomplete left spinal cord lesion at the level of T7, resulting in Brown-Sequard syndrome. Which nursing action should be included in the plan of care? A. Assessment of the patient for right arm weakness B. Assessment of the patient for increased right leg pain C. Positioning the patient's left leg when turning the patient D. Teaching the patient to look at the right leg to verify its position

C. Positioning the patient's left leg when turning the patient Rationale: The patient with Brown-Sequard syndrome has loss of motor function on the ipsiateral side and will require the nurse to move the left leg. Pain sensation will be lost in the patient's right leg. Arm weakness will not be a problem for a patient with a T7 injury. The patient will retain position sense for the right leg

The patient comes to the emergency department (ED) with cortical blindness and visual field defects. Which type of head injury does the nurse suspect? A. Cerebral contustion B. Orbital skull fracture C. Posterior fossa fracture D. Frontal lobe skull fracture

C. Posterior fossa fracture Rationale: The posterior fossa fracture causes occipital bruising resulting in cortical blindness or visual field defects. A cerebral contusion is bruising of brain tissue within a focal area. An orbital skull fracture would cause periorbital ecchymosis (raccoon eyes) and possible optic nerve injury. A frontal lobe skull fracture would expose the brain to contaminants through the frontal air sinus and thee patient would have CNS rhinorrhea or pneumocranium

A surgical repair is panned for a patient who has a 5.5cm abdominal aortic aneurysm (AAA). On physical assessment of the patient, what should the nurse expect to find? A. Hoarseness and dysphagia B. Severe back pain with flank ecchymosis C. Presence of a bruit in the periumbilical area D. Weakness in the lower extremities progressing to paraplegia

C. Presence of a bruit in the periumbilical area Rationale: Although most abdominal aortic aneurysms (AAAs) are asymptomatic, on physical examination a pulsatile mass in the periumbilical area slightly to the left of the midline may be detected and bruits may be audible with a stethoscope placed over the aneurysm. Hoarseness and dysphagia may occur with aneurysms of the ascending aorta and aortic arch. Severe back pain with flank ecchymosis is usually present on rupture of an AAA and neurovascular loss in the lower extremities may occur from a thoracic aneurysm

Which information about a 30 year old patient who is hospitalized after a traumatic brain injury requires the most rapid action by the nurse? A. Intracranial pressure of 15mmHg B. Cerebrospinal fluid (CSF) drainage of 25mL/hr C. Pressure of oxygen in brain tissue (PbtO2) is 14mmHg D. Cardiac monitor shows sinus tachycardia at 120 beats/min

C. Pressure of oxygen in brain tissue (PbtO2) is 14mmHg Rationale: The PbtO2 should be 20 to 40 mmHg. Lower levels indicate brain ischemia. CSF is produced at a rate of 20 to 30 mL/hr. The reason for the sinus tachycardia should be investigated, but the elevated heart rate is not as concerning as the decrease in PbtO2.

A young woman who runs vigorously as a form of exercise has not had a menstrual period in more than 6 months. What should the nurse teach her? A. Normal periods will return when she stops running B. Uterine balloon therapy may be necessary to promote uterine sloughing of the overgrown endometrium C. Progesterone or combined oral contraceptives should be used to prevent persistent overgrowth of the endometrium D. Unopposed progesterone production causes an overgrowth of the endometrium that increases her risk for endometrial cancer

C. Progesterone or combined oral contraceptives should be used to prevent persistent overgrowth of the endometrium Rationale: When ovulation does not occur, estrogen continues to be unopposed by progesterone and excessive buildup of the endometrium occurs. To prevent the risk of endometrial cancer by the buildup of the endometrium or to prevent heavy menstrual bleeding from an unstable endometrium, progesterone or combined oral contraceptives are prescribed to ensure regular shedding of the endometrial lining. Balloon therapy to treat heavy menstrual bleeding is contraindicated in women who desire future fertility and does not apply to amenorrhea

A patient with MS has a nursing diagnosis of self care deficit related to muscle spasticity and neuromuscular deficits. In providing care for the patient, what is most important for the nurse to do? A. Teach the family members how to care adequately . for the patient's needs B. Encourage the patient to maintain social interactions to prevent social isolation C. Promote the use of assistive devices so that the patient can participate in self care activities D. Perform all activities of daily living (ADLs) for the patient to conserve the patient's energy

C. Promote the use of assistive devices so that the patient can participate in self care activities Rationale: The main goal in care of the patient with MS is to keep the patient active and maximally functional and promote self care as much as possible to maintain independence. Assistive devices encourage independence while preserving the patient's energy. No care activity that the patient can do for himself or herself should be performed by others. Involvment of the family in the patient's care and maintenance of social interactions are also important but are not he priority care

When caring for a patient following a radical prostatectomy with a perineal approach, what is the priority nursing intervention the nurse should use to prevent complications? A. Use chemotherapy to prevent metastasis B. Administer sildenafil (Viagra) as needed for erectile dysfunction C. Provide wound care after each bowel movement to prevent infection D. Insert a smaller indwelling urinary catheter to prevent urinary retention

C. Provide wound care after each bowel movement to prevent infection Rationale: A prostatectomy performed with a perineal approach has a high risk for infection because of the proximity of the wound to the anus, so wound care is the priority. Chemotherapy is usually not the first choice of drug therapy following surgery, nor is sildenafil. The catheter size would not be changed but the catheter would be removed. Urinary incontinence is a bigger problem than retention

During assessment of a patient with dementia, the nurse determines that the condition is potentially reversible when finding out what about the patient? A. Has long-standing abuse of alcohol B. Has a history of Parkinson's disease C. Recently developed symptoms of hypothyroidism D. Was infected with human immunodeficiency virus (HIV) 15 years ago

C. Recently developed symptoms of hypothyroidism Rationale: Hypothyroidism can cause dementia, but it is a treatable condition if it has not been long standing. The other conditions are causes of irreversible dementia

What functions does the thalamus have? A. Registers auditory input B. Integrates past experiences C. Relays sensory and motor input to and from the cerebrum D. Controls and facilitates learned and automatic movements

C. Relays sensory and motor input to and from the cerebrum Rationale: The thalamus relays sensory and motor input to and from the cerebrum and basal ganglia. Auditory input is registered by the superior temporal gyrus. Past experiences are integrated by the anterior temporal lobe. The basal ganglia controls and facilitates learned and automatic movements associated with skeletal muscle activity

The nursing care area is very busy with new surgical patients. Which care could the RN delegate to the unlicensed assistive personnel (UAP) for a patient with VTE? A. Assess the patient's use of herbs B. Measure the patient for elastic compression stockings C. Remind the patient to flex and extend the legs and feet every 2 hours D. Teach the patient to call the emergency response system (ERS) with signs of pulmonary embolus

C. Remind the patient to flex and extend the legs and feet every 2 hours Rationale: The RN could delegate to the unlicensed assistive personnel (UAP) the task to remind the patient to flex and extend the legs and feet every 2 hours while in bed. Measuring for elastic compression stockings may be delegated to the licensed practical nurse (LPN). The RN must assess and teach the patient

A patient with SCI has spinal shock. The nurse plans care for the patient based on what knowledge? A. Rehabilitation measures cannot be initiated until spinal shock has resolved B. The patient will need continuous monitoring for hypotension, tachycardia, and hypoxemia C. Resolution of spinal shock is manifested by spasticity, reflex return, and neurogenic bladder D. Patient will have complete loss of motor and sensory functions below the level of the injury, but autonomic functions are not affected

C. Resolution of spinal shock is manifested by spasticity, reflex return, and neurogenic bladder Rationale: Spinal shock occurs in many people with acute SCI. In spinal shock, the entire cord below the level of the lesion fails to function, resulting in a flaccid paralysis and hypomotility of most processes without any reflex activity. Return of reflex activity, although spastic, signals the end of spinal shock. Rehabilitation activities are not contraindicated during spinal shock and should be instituted if the patient's cardiopulmonary status is stable. Neurogenic shock results from loss of vascular tone caused by the injury and is manifested by hypotension, peripheral vasodilation, and decreased cardiac output (CO). Sympathetic function is impaired below the level of the injury because sympathetic nerves leave the spinal cord at the thoracic and lumbar areas and cranial parasympathetic nerves predominate in control over respirations, heart, and all vessels and organs below the injury, which includes autonomic functions

Which assessment is most important for the nurse to make regarding a patient with myasthenia gravis? A. Pupil size B. Grip strength C. Respiratory effort D. Level of consciousness

C. Respiratory effort Rationale: Because respiratory insufficiency may be life threatening, it will be most important to monitor respiratory function. The other data also will be assessed but are not as critical

During care of a patient in myasthenic crisis, maintenance of what is the nurse's first priority for the patient? A. Mobility B. Nutrition C. Respiratory function D. Verbal communication

C. Respiratory function Rationale: The patient with myasthenic crisis has severe weakness and fatiguability of all skeletal muscles, affecting the patient's ability to breathe, swallow, talk, and move. However, the priority of nursing care is monitoring and maintaining adequate ventilation

Which type of macroglial cells myelinate peripheral nerve fibers? A. Neurons B. Astrocytes C. Schwann cells D. Ependymal cells

C. Schwann cells Rationale: Schwann cells are the macroglial cells that myelinate peripheral nerve fibers. Neurons are not glial cells. Astrocytes provide structural support to neurons and form the blood brain barrier with the endothelium of blood vessels. Ependymal cells line the brain ventricles and aid in secretion of CSF

An infertile couple has used at home ovulation testing using basal body temperature without conceiving. The nurse understands that what will be used first to treat this infertile couple? A. Surgery to reduce endometriosis B. Intrauterine insemination with sperm from the husband C. Selective estrogen receptor modulator (clomiphene [Clomid]) D. Assisted reproductive technologies (e.g. in vitro fertilization)

C. Selective estrogen receptor modulator (clomiphene [Clomid]) Rationale: Drug therapy will be used before more invasive treatments. Drugs may include selective estrogen receptor modulators, menotropin (human menopausal gonadotropin), follicle stimulating hormone agonists, gonadotropin releasing hormone (GnRH) antagonists, GnRH agonists, or human chorionic gonadotropin (hCG). If the husband's reproductive system in functioning, intrauterine insemination with his sperm may later be done. The assisted reproductive technologies may be used if this was not successful. The surgery for endometriosis could be done if this was diagnosed, but that is not included in this question

During the nursing assessment of the patient with a type B aortic dissection, what should the nurse expect the patient to manifest? A. Altered LOC with dizziness and weak carotid pulses B. A cardiac murmur characteristic of aortic valve insufficiency C. Severe "ripping" back or abdominal pain with decreased urine output D. Severe hypertension and orthopnea and dyspnea of pulmonary edema

C. Severe "ripping" back or abdominal pain with decreased urine output Rationale: A Type B aortic dissection involves the distal descending aorta and is usually characterized by a sudden, severe, tearing pain in the back. As it progresses down the aorta, the kidneys, abdominal organs, and lower extremities may begin to show evidence of ischemia. Type A aortic dissections of the ascending aorta and aortic arch may affect the heart and circulation to the head, with the development of cerebral ischemia, murmurs, ventricular failure, and pulmonary edema

Before surgical stabilization, what method of immobilization for the patient with a cervical SCI should the nurse expect to be used? A. Kinetic beds B. Hard cervical collar C. Skeletal traction with skull tongs D. Sternal-occipital-mandibular immobilizer brace

C. Skeletal traction with skull tongs Rationale: The development of better surgical stabilization has made surgery the more frequent treatment of cervical injuries. However, when surgery cannot be done, skeletal traction with the use of Crutchfeld, Gardner-Wells, or other types of skull tongs is required to immobilize the cervical vertebrae, even if a fracture has not occurred. Special turning or kinetic beds may be used to turn and mobilize patients who are in cervical traction. Hard cervical collars or a sternal-occipital-mandibular-immobilizer brace may be used for stabilization during emergency transport of the patient, after cervical stabilization surgery, or minor injuries

When teaching a patient with a seizure disorder about the medication regimen, what is it most important for the nurse to emphasize? A. The patient should increase the dosage of the medication if stress is increased B. Most over the counter and prescription drugs are safe to take with antiseizure drugs C. Stopping the medication abruptly may increase the intensity and frequency of seizures D. If gingival hypertrophy occurs, the HCP should be notified and the drug may be changed

C. Stopping the medication abruptly may increase the intensity and frequency of seizures Rationale: If antiseizure drugs are discontinued abruptly, seizures can be precipitated. Missed doses should be made up if the omission is remembered within 24 hours, and patients should not adjust medications without professional guidance because this also can increase seizure frequency and may cause status epilepticus. Antiseizure drugs have numerous interactions with other drugs, and the use of other medications should be evaluated by the HCP. If side effects occur, the HCP should be notified and drug regimens evaluated

A 20 year old woman is a college softball player who participates in strenuous practices and a heavy class schedule. She is describing an absence of menses. What could be contributing to he amenorrhea? A. Decreased sexual activity B. Excess prostaglandin production C. Strenuous exercise or elevated stress D. Endometrial cancer or uterine fibroids

C. Strenuous exercise or elevated stress Rationale: Young female athletes may experience amenorrhea related to excessive exercise, low body weight, stress, or severe dieting. If she had increased sexual activity, she would be assessed for pregnancy, but decreased sexual activity will not affect her menses. Excess prostaglandin production leads to dysmenorrhea. Intermenstrual bleeding is associated with endometrial cancer or uterine fibroids

A 55 year old with a history of prostate cancer in his family asks the nurse what he can do to decrease his risk of prostate cancer. What should the nurse teach him about prostate cancer risks? A. Nothing can decrease the risk because prostate cancer is primarily a disease of aging B. treatment of any enlargement of the prostate gland will help to prevent prostate cancer C. Substituting fresh fruits and vegetables for high fat foods in the diet may lower the risk of prostate cancer D. Using a natural herb, such as saw palmetto, has been found to be an effective protection against prostate cancer

C. Substituting fresh fruits and vegetables for high fat foods in the diet may lower the risk of prostate cancer Rationale: Most prostate cancers (about 75%) are considered sporadic. About the only modifiable risk factor for prostate cancer is its association with a diet high in red and processed meat and high fat dairy products along with a low intake of vegetables and fruit. Age, ethnicity, and family history are risk factors for prostate cancer but are not modifiable. Environment may also play a role. Simple enlargement or hyperplasia of the prostate is not a risk factor for prostate cancer. There is no evidence that saw palmetto is more effective than a placebo

A patient requests a prescription for birth control pills to control severe abdominal cramping and headaches during her menstrual periods. Which action should the nurse take first? A. Determine whether the patient is sexually active B. Teach about the side effects of oral contraceptives C. Take the patient's personal and family health history D. Suggest nonsteroidal antiinflammatory drugs (NSAIDs)

C. Take the patient's personal and family health history Rationale: Oral contraceptives may be appropriate to control this patient's symptoms, but the patient's health history may indicate contraindications to oral contraceptive use. Because the patient is requesting contraceptives for management of dysmenorrhea, whether she is sexually active is irrelevant. Because the patient is asking for birth control pills, responding that she try NSAIDs is nontherapeutic. The patient does not need teaching about oral contraceptive side effects at this time

How is cranial nerve (CN) III, originating in the midbrain, assessed by the nurse for an early indication of pressure on the brainstem? A. Assess for nystagmus B. Test the corneal reflex C. Test pupillary reaction to light D. Test for oculocephalic (doll's eyes) reflex

C. Test pupillary reaction to light Rationale: One of the functions of cranial nerve (CN) III, the oculomotor nerve, is pupillary constriction and testing for pupillary constriction is important to identify patients at risk for brainstem herniation caused by increased ICP. Nystagmus is commonly associated with specific lesions or chemical toxicities and is not a definitive sign of ICP. The corneal reflex is used to assess the functions of CN V and VII and the oculocephalic reflex tests all cranial nerves involved with eye movement

Which action by the unlicensed assistive personnel (UAP) who are assisting with the care of male patients with reproductive problems indicates that the nurse should provide more teaching? A. The UAP apply a cold pack to the scrotum for a patient with mumps orchitis B. The UAP help a patient who has had a prostatectomy to put on antiembolism hose. C. The UAP leave the foreskin pulled back after cleaning the glans of a patient who has a retention catheter D. The UAP encourage a high fluid oral intake for a patient who had transurethral resection of the prostate yesterday

C. The UAP leave the foreskin pulled back after cleaning the glans of a patient who has a retention catheter Rationale: Paraphimosis can be caused by failing to replace the foreskin back over the glans after cleaning. The other actions by UAP are appropriate

The charge nurse is observing a new staff nurse who is assessing a patient with a traumatic spinal cord injury for sensation. Which action indicates a need for further teaching of the new nurse about neurologic assessment? A. The new nurse tests for light touch before testing for pain B. The new nurse has the patient close the eyes during testing C. The new nurse asks the patient if the instrument feels sharp D. The new nurse uses an irregular pattern to test for intact touch

C. The new nurse asks the patient if the instrument feels sharp Rationale: When performing a sensory assessment, the nurse should not provide verbal clues. The other actions by the new nurse are appropriate

Which information about a 60-year-old patient with MS indicates that the nurse should consult with the health care provider before giving the prescribed dose of dalfampridine (Ampyra)? A. The patient walks a mile each day for exercise B. The patient complains of pain with neck flexion C. The patient has an increased serum creatinine level D. The patient has the relapsing-remitting form of MS

C. The patient has an increased serum creatinine level Rationale: Dalfampridine should not be given to patients with impaired renal function. The other information will not impact whether the dalfampridine should be administered

The nurse is caring for a patient who has a head injury and fractured right arm after being assaulted. Which assessment information requires rapid action by the nurse? A. The apical pulse is slightly irregular B. The patient complains of a headache C. The patient is more difficult arouse D. The blood pressure (BP) increase to 140/62mmHg

C. The patient is more difficult arouse Rationale: The change in level of consciousness (LOC) is an indicator of increased intracranial pressure (ICP) and suggests that action by the nurse is needed to prevent complications. The change in BP should be monitored but is not an indicator or a need for immediate nursing action. Headache and a slightly irregular apical pulse are not unusual in a patient after a head injury

The charge nurse observes an inexperienced staff nurse caring for a patient who has had a craniotomy for resection of a brain tumor. Which action by the inexperienced requires the charge nurse to intervene? A. The staff nurse assesses neurologic status every hour B. The staff nurse elevates the head of the bed to 30 degrees C. The staff nurse suctions the patient routinely every 2 hours D. The staff nurse administers an analgesic before turning the patient

C. The staff nurse suctions the patient routinely every 2 hours Rationale: Suctioning increases intracranial pressure and should only be done when the patient's respiratory condition indicates it is needed. The other actions by the staff nurse are appropriate

What is the rationale for the regular use of nonsteroidal antiinflammatory drugs (NSAIDs) during the first several days of the menstrual period for women who have primary dysmenorrhea? A. They suppress ovulation and the production of prostaglandins that occur with ovulation B. They cause uterine relaxation and small vessel constriction, preventing cramping and abdominal congestion C. They inhibit the production of prostaglandins believed to be responsible for menstrual pain and associated with symptoms D. The block the release of luteinizing hormone, preventing the increase in progesterone associated with maturation of the corpus luteum

C. They inhibit the production of prostaglandins believed to be responsible for menstrual pain and associated with symptoms Rationale: The release of excess prostaglandins from the endometrium at thee time of menstruation of increased sensitivity to the prostaglandins is responsible for symptoms of primary dysmenorrhea and drugs that inhibit prostaglandin production and release, such as nonsteroidal antinflammatory drugs (NSAIDs), are effective in many patients with primary dysmenorrhea. Oral contraceptives may be used for primary dysmenorrhea by reducing endometrial hyperplasia

While the nurse is transporting a patient on a stretcher to the radiology department, the patient begins having a tonic-clonic seizure. Which action should the nurse take? A. Insert an oral airway during the seizure to maintain a patent airway. B. Restrain the patient's arms and legs to prevent injury during the seizure. C. Time and observe and record the details of the seizure and postictal state. D. Avoid touching the patient to prevent further nervous system stimulation.

C. Time and observe and record the details of the seizure and postictal state. Rationale: Because the diagnosis and treatment of seizures frequently are based on the description of the seizure, recording the length and details of the seizure is important. Insertion of an oral airway and restraining the patient during the seizure are contraindicated. The nurse may need to move the patient to decrease the risk of injury during the seizure

The patient is suspected of having endometriosis and/or uterine leiomyoma. What best describes what is found with these conditions? A. Endometriosis and uterine leiomyoma are two gynecologic conditions that increase in incidence with the onset of menopause B. Danazol and Lupron (GnRH analog) are used to treat endometriosis and leiomyomas to create pseudopregnancy. C. Treatment of endometriosis and leiomyomas depends on the severity of symptoms and the woman's desire to maintain her fertililty D. The presence of ectopic uterine tissue that bleeds and causes pelvic and abdominal adhesions, cysts and pain is known as uterine leiomyoma

C. Treatment of endometriosis and leiomyomas depends on the severity of symptoms and the woman's desire to maintain her fertility Rationale: The treatment for endometriosis and leiomyomas is surgical when the patient does not tolerate the symptoms, with they type of surgery for endometriosis dependent on the desire for pregnancy. Endometriosis and leiomyomas subside with the onset of menopause. Therefore the medications to treat them create a pseudomenopause. The ectopic uterine tissue is endometriosis, while leiomyomas are fibrous smooth muscle tumors

Which of the following CNs responds to the corneal reflex test? A. Optic (CN II) B. Vagus (CN X) C. Trigeminal (CN V) D. Spinal accessory (CN XI)

C. Trigeminal (CN V) Rationale: The trigeminal (CN V) and facial (CN VII) nerves both respond to the corneal reflex test. The optic (CN II) nerve is tested by confrontation or field of vision. The vagus (CN X) nerve provides the gag reflex with the glossopharyngeal (CN IX). The spinal accessory (CN XI) nerve is tested with the resistive shoulder shrug

The current incidence of sexually transmitted infections (STIs) is related in part to what? A. Increased social acceptance of homosexuality B. Increased virulence of organisms that cause STIs C. Use of oral agents rather than condoms as contraceptives D. Increased microorganism resistance to common antibiotics

C. Use of oral agents rather than condoms as contraceptives Rationale: Although many factors relate to the current sexually transmitted infections (STI) rates, one major factor is the widespread use of oral contraceptives instead of condoms (both male and female). Condoms are the only contraceptive device that protects against STIs

Which action will help the nurse determine whether a new patient's confusion is caused by dementia and delirium? A. Ask about a family history of dementia B. Administer the Mini Mental Status Exam C. Use the Confusion Assessment Method tool D. Obtain a list of the patient's usual medications

C. Use the Confusion Assessment Method tool Rationale: The Confusion Assessment Method tool has been extensively tested in assessing delirium. The other actions will be helpful in determining cognitive function or risk factors for dementia or delirium, but they will not be useful in differentiating between dementia and delirium

The patient is describing a feeling of something coming down her vagina and having a backache. What is most likely the cause of this discomfort? A. Cystocele B. Dysmenorrhea C. Uterine prolapse D. Abdominal distention

C. Uterine prolapse Rationale: A uterine prolapse occurs when the uterus is displaced through the vagina, causing the feeling of something coming down her vagina, a backache, dyspareunia, or a heavy feeling in the pelvis

What describes hypospdias? A. Scrotal lymphedema B. Undescended testicle C. Ventral urinary meatus D. Inflammation of the prepuce

C. Ventral urinary meatus Rationale: C. Hydrospadias is the urethral meatus located on the ventral surface of the penis. Scrotal lymphedema is called a hydrocele. An undescended testicle is cryptorchidism. Inflammation of the prepuce or foreskin is called phimosis

A 72 year old patient is brought to the clinic by the patient's spouse, who reports that the patient is unable to solve common problems around the house. To obtain information about the patient's current mental status, which question should the nurse ask the patient? A. Are you sad right now B. How is your self image C. What did you eat for lunch D. Where were you born

C. What did you eat for lunch Rationale: This question tests the patient's short term memory, which is decreased in the mild stage of Alzheimer's disease or dementia. Asking the patient about her birthplace tests for remote memory, which is intact in the early stages. Questions about the patient's emotions and self image are helpful in assessing emotional status, but they are not as helpful in assessing mental state

A nursing diagnosis that is likely to be appropriate for a 67 year old patient who has just been diagnosed with stage III ovarian cancer is A. sexual dysfunction related to loss of vaginal sensation B. risk for infection related to impaired immune function C. anxiety related to cancer diagnosis and need for treatment decisions D. situational low self esteem related to guilt about delaying medical care

C. anxiety related to cancer diagnosis and need for treatment decisions Rationale: The patient with stage III ovarian cancer is likely to be anxious about the poor prognosis and about the need to make decisions about the multiple treatments that may be used. Decreased vaginal sensation does not occur with ovarian cancer. The patient may develop immune dysfunction when she receives chemotherapy, but she is not currently at risk. It is unlikely that the patient has delayed seeking medical care because the symptoms of ovarian cancer are vague and occur late in the course of the cancer

To prevent autonomic hyperreflexia, which nursing action will the home health nurse include in the plan of care for a patient who has paraplegia at the T4 level? A. support selection of a high protein diet B. discuss options for sexuality and infertility C. assist in planning a prescribed bowel program D. use quad coughing to strengthen cough efforts

C. assist in planning a prescribed bowel program Rationale: Fecal impaction is a common stimulus for autonomic hyperreflexia. Dietary protein, coughing, and discussing sexuality and fertility should be included in the plan of care but will not reduce the risk for autonomic hyperreflexia

A young adult patient tells the healthcare provider about experiencing cold, numb fingers when running during the winter, and Raynaud's phenomenon is suspected. The nurse will anticipate teaching the patient about tests for A. hyperglycemia B. hyperlipidemia C. autoimmune disorders D. coronary artery disease

C. autoimmune disorders Rationale: Secondary Raynaud's phenomenon may occur in conjunction with autoimmune diseases such as rheumatoid arthritis. Patients should be screened for autoimmune disorders. Raynaud's phenomenon is not associated with hyperlipidemia, hyperglycemia, or coronary artery disease

The nurse is providing teaching by telephone to a patient who is scheduled for a pelvic examination and Pap test next week. The nurse instructs the patient that she should A. not have sexual intercourse the day before the Pap test B. shower, but not take a tub bath, before the examination C. avoid douching for at least 24 hours before the examination D. schedule to have the Pap test just after her menstrual period

C. avoid douching for at least 24 hours before the examination Rationale: Because the results of the Pap test may be affected by douching, the patient should not douche before the examination. The examination may be scheduled without regard to the menstrual period. The patient may shower or bathe before the examination. Sexual intercourse does not affect the results of the examination or Pap test

The patient's SCI is at T4. What is the highest level goal of rehabilitation that is realistic for this patient to have? A. indoor mobility in a wheelchair B. ambulate with crutches and leg braces C. be independent in self care and wheel chair use D. completely independent ambulation with short leg braces and canes

C. be independent in self care and wheel chair use Rationale: With the injury at T4, the highest level realistic goal for this patient is to be able to be independent in self care and wheelchair use because arm function will not be affected. Indoor mobility in a manual wheelchair will be achievable, but it is not the highest level goal. Ambulating with crutches and leg braces can be achieved only by patients with injuries in T6-T12 area. Independent ambulation with short leg braces and canes could occur for a patient with an L3-L4 injury

The nurse is caring for a patient with a descending aortic dissection. Which assessment finding is most important to report to the health care provider? A. weak pedal pulses B. absent bowel sounds C. blood pressure of 138/88 mmHg D. 25 mL of urine output over the past hour

C. blood pressure of 138/88 mmHg Rationale: The blood pressure is typically kept at less than 120 mmHg systolic to minimize extension of the dissection. The nurse will need to notify the health care provider so that B-Blockers or other antihypertensive drugs can be prescribed. The other findings are typical with aortic dissection and should also be reported but do not require immediate action

Increased ICP in the left cerebral cortex, caused by intracranial bleeding causes displacement of brain tissue to the right hemisphere beneath the falx cerebri. The nurse knows that this is referred to as A. uncal herniation B. tentorial herniation C. cingulate herniation D. temporal lobe herniation

C. cingulate herniation Rationale: The dural structures that separate the two hemispheres and the cerebral hemispheres from the cerebellum influence the patterns of cerebral herniation. A cingulate herniation occurs where there is lateral displacement of brain tissue beneath the falx cerebri. Uncal herniation occurs when there is lateral and downward herniation. Tentorial herniation occurs when the brain herniates down through the opening created by the brainstem. The temporal lobe can be involved in central herniation.

Vascular dementia is associated with A. transient ischemic attacks B, bacterial or viral infection of neuronal tissue C. cognitive changes secondary to cerebral ischemia D. abrupt changes in cognitive function that are irreversible

C. cognitive changes secondary to cerebral ischemia Rationale: Vascular dementia is the loss of cognitive function that results from ischemic, ischemic-hypoxic, or heemorrhagic brain lesions caused by cardiovascular disease. In this type of dementia, narrowing and blocking of arteries that supply the brain cause a decrease in blood supply

The patient has a depressed skull fracture and scalp lacerations with communication to the intracranial cavity. Which type of injury should the nurse record? A. linear skull fracture B. depressed skull fracture C. compound skull fracture D. comminuted skull fracture

C. compound skull fracture Rationale: The compound skull fracture is a depressed skull fracture and scalp lacerations with communicating pathway(s) to the intracranial cavity. A linear skull fracture is a straight break in the bone without alteration in the fragments. A depressed skull fracture is an inward indentation of the skull that may cause pressure on the brain. A comminuted skull fracture has multiple linear fractures with bone fragmented into many pieces

When a brain-injured patient responds to nail bed pressure with internal rotation, adduction, and flexion of the arms, the nurse reports the response as A. flexion withdrawal. B. localization of pain. C. decorticate posturing. D. decerebrate posturing.

C. decorticate posturing. Rationale: Internal, rotation, adduction, and flexion of the arms in an unconscious patient is documented as decorticate posturing. Extension of the arms and legs is decerebrate posturing. Because the flexion is generalized, it does not indicate localization of pain or flexion withdrawal.

Which finding in a patient with a spinal cord tumor requires an immediate report to the health care provider? A. depression about the diagnosis B. anxiety about scheduled surgery C. decreased ability to move the legs D. back pain that worsens with coughing

C. decreased ability to move the legs Rationale: Decreasing sensation and leg movement indicates spinal cord compression, an emergency that will require rapid action (such as surgery) to prevent paralysis. The other findings will also require nursing action but are not emergencies

The nurse recognizes the presence of Cushing's triad in the patient with A. Increased pulse, irregular respiration, increased BP B. decreased pulse, increased respiration, decreased systolic BP C. decreased pulse, irregular respiration, widened pulse pressure D. Increased pulse, decreased respiration, widened pulse pressure

C. decreased pulse, irregular respiration, widened pulse pressure Rationale: Cushing's triad consists of three vital signs measures that reflect increased ICP and its effect on the medulla, hypothalamus, pons and thalamus. Because these structures are very deep, Cushing's triad is usually a late sign of increased ICP. The signs include an increasing SBP with a widening pulse pressure, bradycardia with a full and bounding pulse, and irregular respirations

An early sign of increased ICP that the nurse should assess for is A. Cushing's triad B. unexpected vomiting C. decreasing level of consciousness D. dilated pupil with sluggish response to light

C. decreasing level of consciousness Rationale: One of the most sensitive signs of increased ICP is a decreasing level of consciousness (LOC). A decrease in LOC will occur before changes in vital signs, ocular signs, or projectile vomiting occur

A 38 year old patient who has had a spinal cord injury returned home following a stay in a rehabilitation facility. The home care nurse notes the spouse is performing many of the activities that the patient had been managing unassisted during rehabilitation. The appropriate nursing action at this phase of rehabilitation is to A. remind the patient about the importance of independence in daily activities B. tell the spouse to stop helping because the patient is able to perform activities independently C. develop a plan to increase the patient's independence in consultation with the patient and the spouse D. recognize that it is important for the spouse to be involved in the patient's care and encourage the participation

C. develop a plan to increase the patient's independence in consultation with the patient and the spouse Rationale: The best action by the nurse will be to involve all parties in developing an optimal plan of care. Because family members who will be assisting with the patient's ongoing care need to believe their input is important, telling the spouse that the patient can perform activities independently is not the best choice. Reminding the patient about the importance of independence may not change the behaviors of the spouse. Supporting the activities of the spouse will lead to ongoing dependency by the patient

The nurse is alerted to a possible acute subdural hematoma in the patient who A. has a linear skull fracture crossing a major artery B. has focal symptoms of brain damage with no recollection of a head injury C. develops decreased level of consciousness and a headache within 48 hours of a head injury D. has an immediate loss of consciousness with a brief lucid interval followed by decreasing level of consciousness

C. develops decreased level of consciousness and a headache within 48 hours of a head injury Rationale: An acute subdural hematoma manifests within 24 to 48 hours of the injury. The signs and symptoms are similar to those associated with brain tissue compression in elevated ICP and include decreasing level of consciousness and headache

To determine the severity of the symptoms for a patient with benign prostatic hyperplasia (BPH), the nurse will ask the patient about A. blood in the urine. B. lower back and hip pain. C. force of urinary stream D. erectile dysfunction

C. force of urinary stream Rationale: The American Urological Association Symptom Index for a patient with BPH asks questions about the force and frequency of urination, nocturia, and so on. Blood in the urine, ED, and back or hip pain are not typical signs of BPH

In noting the results of an analysis of CSF, what should the nurse identify as an abnormal finding? A. pH 7.35 B. clear, colorless appearance C. glucose level of 30mg/dL (1.7 mmol/L) D. WBC count of 5 cells/uL (5x10^6 cells/L)

C. glucose level of 30mg/dL (1.7 mmol/L) Rationale: Normal glucose levels in CSF are 40 to 70 mg/dL. All types of organisms consume glucose, and a decreased glucose level reflects bacterial activity. Increased levels are associated with diabetes. The other results are all normal

To decrease the patient's discomfort about care involving his reproductive organs, the nurse should A. relate his sexual concerns to his sexual partner. B. arrange to have male nurses care for the patient. C. maintain a nonjudgmental attitude toward his sexual practices. D. use technical terminology when discussing reproductive function.

C. maintain a nonjudgmental attitude toward his sexual practices. Rationale: Conducting routine health assessments on men places the nurse in a unique position. It provides an opportunity to ask the patient questions about general health and about sexual health and function. Given the opportunity, men are less hesitant to answer these questions when they know that someone cares and can provide them with answers. The nurse must remain nonjudemental about sexual practices

A patient with Guillain-Barre syndrome asks whether he is going to die as the paralysis spreads toward his chest. In responding to the patient, what should the nurse know to be able to best answer this question? A. patients who require ventilatory support almost always die B. death occurs when nerve damage affects the brain and meninges C. most patient with Guillain-Barre syndrome make a complete recovery D. if death can be prevented, residual paralysis and sensory impairment are usually permanent

C. most patient with Guillain-Barre syndrome make a complete recovery Rationale: As nerve involvement ascends, it is very frightening for the patient, about 80% of patients with Guillain Barre syndrome recover completely with care, although 65% may have a residual weakness. Patients have a poor prognosis but also recover if ventilatory support is provided during respiratory failure. Guillain Barre syndrome affects only peripheral nerves and does not affect the brain

A patient with a T4 spinal cord injury asks the nurse if he will be able to be sexually active. Which initial response by the nurse is best? A. reflex erections frequently occur, but orgasm may not be possible B. sildenafil (Viagra) is used by many patients with spinal cord injury C. multiple options are available to maintain sexuality after spinal cord injury D. penile injection, prostheses, or vacuum suction devices are possible options

C. multiple options are available to maintain sexuality after spinal cord injury Rationale: Although sexuality will be changed by the patient's spinal cord injury, there are options for expression of sexuality and for fertility. The other information also is correct, but the choices will depend on the degrees of injury and the patient's individual feelings about sexuality

A patient with infective endocarditis develops sudden left leg pain with pallor, parasthesia, and a loss of peripheral pulses. The nurse's initial action should be to A. elevate the leg to promote venous return B. start anticoagulant therapy with IV heparin C. notify the HCP of the change in peripheral perfusion D. place the bed in reverse Trendelenburg to promote perfusion

C. notify the HCP of the change in peripheral perfusion Rationale: The patient has potentially developed acute arterial ischemia (sudden interruption in the arterial blood supply to the extremity), caused by an embolism from a cardiac thrombus that occurred as a complication of infective endocarditis. Clinical manifestations of acute arterial ischemia include any or all of the six P's: pain, pallor, paralysis, pulselessness, parasthesia, and poikilothermia. Without immediate intervention, ischemia may progress quickly to tissue necrosis and gangrene within a few hours. If the nurse detects these signs, the HCP should be notified immediately

A 40 year old patient is diagnosed with early Huntington's disease (HD). When teaching the patient, spouse, and adult children about this disorder, the nurse will provide information about the A. use of levodopa/carbidopa (Sinemet) to help reduce HD symptoms B. prophylactic antibiotics to decrease the risk of aspiration pneumonia C. option of genetic testing for the patient's children to determine their own HD risks D. lifestyle changes of improved nutrition and exercise that delay disease progression

C. option of genetic testing for the patient's children to determine their own HD risks Rationale: Genetic testing is available to determine whether an asymptomatic individual has the HD gene. The patient and family should be informed of the benefits and problems associated with genetic testing. Sinemet will increase symptoms of HD because HD involves an increase in dopamine. Antibiotic therapy will not reduce the risk for aspiration. There are no effective treatments of lifestyle changes that delay the progression of symptoms in HD

Dementia with Lewy bodies (DLB) is characterized by A. remissions and exacerbations over many years. B. memory impairment, , muscle jerks, and blindness C. parkinsonian symptoms, including muscle rigidity D. increased intracranial pressure secondary to decreased CSF drainage

C. parkinsonian symptoms, including muscle rigidity Rationale: Dementia with Lewy bodies (DLB) is characterized by features of dementia and Parkinson's disease. These patients typically have manifestations of parkinsonism, hallucinations, short term memory loss, unpredictable cognitive shifts, and sleep disturbances

The nurse on the clinical unit is assigned to four patients. Which patient should she assess first? A. patient with a skull fracture whose nose is bleeding B. elderly patient with a stroke who is confused and whose daughter is present C. patient with meningitis who is suddenly agitated and reporting a HA of 10 on a 0 to 10 scale D. patient who had a craniotomy for a brain tumor who is now 3 days postoperative and has had continued emesis

C. patient with meningitis who is suddenly agitated and reporting a HA of 10 on a 0 to 10 scale Rationale: The patient with meningitis should be seen first; patients with meningitis must be observed closely for manifestations of elevated ICP. which is thought to result from swelling around the dura and increased cerebrospinal fluid (CSF) volume. Sudden change in the level of consciousness or change in behavior along with a sudden severe headache may indicate an acute elevation of ICP. The patient who has undergone cranial surgery should be seen second; although nausea and vomiting are common after cranial surgery. it can result in elevations of ICP. Nausea and vomiting should be treated with antiemetics. The patient with a skull fracture needs to be evaluated for CSF leakage occurring with the nose bleed and should be seen third. Confusion after a stroke may be expected; the patient should have a family member present.

A patient who had a C7 spinal cord injury 1 week ago has a weak cough effort and crackles. The initial intervention by the nurse should be to A. suction the patient's nasopharynx B. notify the patient's health care provider C. push upward on the epigastric area as the patient coughs D. encourage incentive spirometry every 2 hours during the day

C. push upward on the epigastric area as the patient coughs Rationale: Because the cough effort is poor, the initial action should be to use assisted coughing techniques to improve the patient's ability to mobilize secretions. The use of the spirometer may improve respiratory status, but the patient's ability to take deep breaths is limited by the loss of intercostal muscle function. Suctioning may be needed if the patient is unable to expel secretions by coughing but should bot be the nurse's first action. The health care provider should be notified if airway clearance interventions are not effective or additional collaborative interventions are needed

When teaching a patient with premenstrual dysphoric disorder (PMDD) about management of the disorder, the nurse includes thee need to A. limit dietary intake of caffeine and refined sugar B. use estrogen supplements during the luteal phase C. supplement the diet with B6, calcium, and magnesium D. limit exercise and physical activity when symptoms are present

C. supplement the diet with B6, calcium, and magnesium Rationale: Vitamin B6, calcium, and magnesium are recommended, as well as foods high in tryptophan, which may promote serotonin production, to improve mood changes. Limitation of refined sugar and caffeine in the day may decrease the PMS symptoms of abdominal bloating, increased appetite, and irritability. Estrogen is not used during the luteal phase, but progesterone may be tried. Exercise is encouraged because it increases the release of endorphins, which elevate the mood, and has a tranquilizing effect on muscle tension

A patient with a C7 spinal cord injury undergoing rehabilitation tells the nurse he must have the flu because he has a bad headache and nausea. The nurse's first priority is to A. Call the HCP B. check the patient's temperature C. take the patient's blood pressure D. elevate the head of the bed to 90 degrees

C. take the patient's blood pressure Rationale: Autonomic hyperreflexia is a massive, uncompensated cardiovascular reaction mediated by the sympathetic nervous system. Manifestations include hypertension (up to 300mmHg systolic), throbbing headache, marked diaphoresis above the level of the injury, bradycardia (30 to 40 beats/min), piloerection, flushing of the skin above the level of the injury, blurred vision or spots in the visual fields, nasal congestion, anxiety, and nausea. It is important to measure BP when a patient with a spinal cord injury complains of a headache. Other nursing interventions in this serious emergency are elevation of the head of the bed 45 degrees or sitting the patient upright, notification of the physician, and assessment to determine the cause. Table 60-8 lists the causes and symptoms of autonomic hyperreflexia. The nurse must monitor BP frequently during the episode. An A-adrenergic blocker or an arteriolar vasodilator may be administered.

To assess the functions of the trigeminal and facial nerves (CNs V and VII), the nurse should A. check for unilateral eyelid droop B. shine a light into the patient's pupil C. touch a cotton wisp strand to the cornea D. have the patient read a magazine or book

C. touch a cotton wisp strand to the cornea Rationale: The trigeminal and facial nerves are responsible for the corneal reflex. The optic nerve is tested by having the patient read a Snellen chart or a newspaper. Assessment of pupil response to light and ptosis are used to evaluate function of the oculomoter nerve

The nurse taking a focused health history for a patient with possible testicular cancer will ask the patient about a history of A. testicular torsion. B. testicular trauma. C. undescended testicles. D. sexually transmitted infection (STI).

C. undescended testicles. Rationale: Cryptorchidism is a risk factor for testicular cancer if it is not corrected before puberty. STI, testicular torsion, and testicular trauma are risk factors for other testicular conditions but not for testicular cancer

The nurse determines that teaching about management of migraine headaches has been effective when the patient says which of the following? A. "I can take the (Topamax) as soon as a headache starts." B."A glass of wine might help me relax and prevent a headache." C."I will lie down someplace dark and quiet when the headaches begin." D. "I should avoid taking aspirin and sumatriptan (Imitrex) at the same time."

C."I will lie down someplace dark and quiet when the headaches begin." Rationale: It is recommended that the patient with a migraine rest in a dark, quiet area. Topiramate (Topamax) is used to prevent migraines and must be taken for several months to determine effectiveness. Aspirin or other nonsteroidal antiinflammatory medications can be taken with the triptans. Alcohol may precipitate migraine headaches

In caring for a patient with endometriosis, the nurse teaches the patient that interventions used to treat or cure this condition may include (select all that apply) A. radiation B. antibiotic therapy C. oral contraceptives D. surgical removal of tissue. E. total abdominal hysterectomy and salpingo-oophorectomy

C.D.E. Oral contraceptives; surgical removal of tissue; total abdominal hysterectomy and salpingo-oophorectomy Rationale: Interventions to treat or manage endometriosis include oral contraceptives, laparotomy to remove implanted tissue and adhesions, and/or total hysterectomy and bilateral salpingo-oophorectomy

The patient is diagnosed with complex focal seizures. Which characteristics are related to complex focal seizures (select all that apply)? A. Formerly known as grand mal seizure B. Often accompanied by incontinence or tongue or cheek biting C. Psychomotor seizures with repetitive behaviors and lip smacking D. Altered memory, sexual sensations, and distortions of visual auditory sensations E. Loss of consciousness and stiffening of the body with subsequent jerking of extremities F. Often involves behavioral, emotional, and cognitive functions with altered consiousness

C.D.F. Psyschomotor seizures with repetitive behaviors and lip smacking; Altered memory, sexual sensations, and distortions of visual or auditory sensations; Often involves behavioral, emotional, and cognitive functions with altered consciousness Rationale: Complex focal seizures are psychomotor seizures with automatisms such as lip smacking. They cause altered consciousness or loss of conciousness producing a dreamlike state and may involve behavioral, emotional, or cognitive experiences without memory of what was done during the seizure. In generalized tonic clonic seizures (previously known as grand mal seizures) there is loss of consciousness and stiffening of the body with subsequent jerking of extremities. Incontinence or tongue or cheek biting may also occur

What are characteristics of a HSV infection? (select all that apply) A. Treatment with acyclovir can cure genital herpes B. Herpes simplex virus type 2 (HSV-2) is capable of causing only genital lesions C Recurrent symptomatic genital herpes may be precipitated by sexual activity and stress D. To prevent transmission of genital herpes, condoms should be used when lesions are present E. The primary symptom of genital herpes is painful vesicular lesions that rupture and ulcerate

C.E. Recurrent symptomatic genital herpes may be precipitated by sexual activity and stress; The primary symptom of genital herpes is painful vesicular lesions that rupture and ulcerate Rationale: Sexual activity and stress my precipitate the recurrence of genital herpes symptoms of painful vesicular lesions that rupture and ulcerate. Acyclovir only decreases recurrence of genital herpes. Herpes simplex virus type 2 (HSV-2) may cause oral or genital lesions. Prevention of the spread of genital herpes is best done with avoidance of sexual activity when lesions are present

Nursing management of a patient with a brain tumor includes (select all that apply) A. discussing with the patient methods to control inappropriate behavior B. using diversion techniques to keep the patient stimulated and motivated C. assisting and supporting the family in understanding any changes in behavior D. limiting self care activities until the patient has regained maximum physical functioning E. planning for seizure precautions and teaching the patient the caregiver about antiseizure drugs

C.E. assisting and supporting the family in understanding any changes in behavior; planning for seizure precautions and teaching the patient the caregiver about antiseizure drugs Rationale: Nursing interventions should be based on a realistic appraisal of the patient's condition and prognosis after cranial surgery. The nurse should provide support and education to the caregiver and family about the patient's behavioral changes. The nurse should be prepared to manage seizures and teach the caregiver and family about antiseizure medications and how to manage a seizure. An overall goal is to foster the patient's independence for as long as possible and to the highest degree possible. The nurse should decrease stimuli in thee patient's environment to prevent increases in intracranial pressure

What methods are used to assess to facial (CN VII) nerve (select all that apply)? A. Gag reflex B. Visual fields C. Corneal (blink) reflex test D. Light touch to the face E. Smile, frown, and close eyes F. Salt and sugar discrimination

C.E.F. Corneal (blink) reflex test; Smile, frown, and close eyes; Salt and sugar discrimination Rationale: The facial nerve (CN VII) is assessed with the corneal (blink) reflex test; smile, frown, and close eues; salt and sugar discrimination. Gag reflex is used to evaluate the glossopharyngeal (CN IX) and vagus (CN X) nerves. Visual field testing is used to assess the optic (CN II) nerve. Light touch to the face and also the corneal (blink) reflex test are used to evaluate the trigeminal (CN V) nerve

A patient has just been instructed in the treatment of Chlamydia trachomatis vaginal infection. Which patient statement indicates that the nurse's teaching has been effective? A. "I can purchase an over the counter medication to treat this infection." B. "The symptoms are due to the overgrowth of normal vaginal bacteria." C. "The medication will need to be inserted once daily with an applicator." D. "Both my partner and I will need to take the medication for a full week."

D. "Both my partner and I will need to take the medication for a full week." Rationale: Chlamydia is a sexually transmitted bacterial infection that requires treatment of both partners with antibiotics for 7 days. The other statements are true for the treatment of Candida albicans infection

An appropriate question to ask the patient with painful menstruation to differentiate primary from secondary dysmenorrhea is A. "Does your pain become worse with activity or overexertion?" B. "Have you had a recent personal crisis or change in your lifestyle?" C. "Is your pain relieved by nonsteroidal antiinflammatory medications?" D. "When in your menstrual history did the pain with your period begin?"

D. "When in your menstrual history did the pain with your period begin?" Rationale: Primary dysmenorrhea starts 12 to 24 hours before the onset of menses. The pain is most severe on the first day of menses and rarely lasts more than two days. Secondary dysmenorrhea usually occurs after the woman has experienced some problem free periods for some time. The pain may be unilateral, and it is usually more constant and continues longer than in primary dysmenorrhea. Depending on the cause, symptoms such as dyspareunia (Pain during intercourse), pain during defecation, or irregular bleeding may occur at times other than menstruation

The nurse in the women's health clinic has four patients who are waiting to be seen. Which patient should the nurse see first? A. 22-year-old with persistent red-brown vaginal drainage 3 days after having balloon thermotherapy B. 42-year-old with secondary amenorrhea who says that her last menstrual cycle was 3 months ago C. 35-year-old with heavy spotting after having a progestin-containing IUD (Mirena) inserted a month ago D. 19-year-old with menorrhagia who has been using superabsorbent tampons and has fever with weakness

D. 19-year-old with menorrhagia who has been using superabsorbent tampons and has fever with weakness Rationale: The patient's history and clinical manifestations suggest possible toxic shock syndrome, which will require rapid intervention. The symptoms for the other patients are consistent with their diagnoses and do not indicate life threatening complications

A woman calls the clinic because she is having an unusually heavy menstrual flow. She tells the nurse that she has saturated 3 tampons in the past 2 hours. The nurse estimates that the amount of blood loss over the past 2 hours is __________mL. A. 20 to 30 B. 30 to 40 C. 40 to 60 D. 60 to 90

D. 60 to 90 Rationale: The average tampon absorbs 20 to 30mL

After the emergency department nurse has received a status report on the following patients who have been admitted with head injuries, which patient should the nurse assess first? A. A patient whose cranial x-ray shows a linear skull fracture B. A patient who has an initial Glasgow Coma Scale score of 13 C. A patient who lost consciousness for a few seconds after a fall D. A patient whose right pupil is 10 mm and unresponsive to light

D. A patient whose right pupil is 10 mm and unresponsive to light Rationale: The dilated and unresponsive pupil may indicate an intracerebral hemorrhage and increased intracranial pressure. The other patients are not at immediate risk for complications such as herniation

How do spinal nerves of the peripheral nervous system (PNS) differ from the cranial nerves (CNs)? A. Only spinal nerves occur in pairs B. CNs affect only sensory and motor functions of the neck and head C. Cell bodies of all CNs are located in the brain, whereas cell bodies of spinal nerves are located in the spinal cord D. All spinal nerves contain both afferent sensory and efferent motor fibers, whereas CNs contain one or the other or both

D. All spinal nerves contain both afferent sensory and efferent motor fibers, whereas CNs contain one or the other or both Rationale: All spinal nerves have both afferent sensory and efferent motor fibers. Some cranial nerves (CNs) are only efferent (e.g. III, IV, VI, VII, XI, XII) some are only afferent (e.g. I, II, VIII) and some have both motor and sensory functions (e.g. V, IX, X). Both cranial and spinal nerves occur in pairs. Most CNs affect the head, neck, and shoulder area, but CN X affects the thorax and abdomen as well. Cranial cell bodies are located in specific areas of the brainstem, except CN I and II are located in the brain

A patient asks the nurse what the difference is between benign prostatic hyperplasia (BPH) and prostate cancer. The best response by the nurse includes what information about BPH? A. BPH is a benign tumor that does not spread beyond the prostate gland B. BPH is a precursor to prostate cancer but does not yet show any malignant changes C. BPH is an enlargement of the gland caused by an increase in the size of existing cells D. BPH is a benign enlargement of the gland caused by an increase in the number of normal cells

D. BPH is a benign enlargement of the gland caused by an increase in the number of normal cells Rationale: Hyperplasia is an increase in the number of normal cells and in benign prostatic hyperplasia (BPH), it is thought that the enlargement caused by the increase in new cells results from hormonal changes associated with aging. The hyperplasia is not considered a tumor, nor has BPH been proven to predispose to cancer of the prostate. Hypertrophy refers to an increase in the size of existing cells

How are the metabolic and nutritional needs of the patient with increased ICP best met? A. Enteral feedings that are low in sodium B. Simple glucose available in D5W IV solutions C. Fluid restriction that promotes a moderate dehydration D. Balanced, essential nutrition in a form that the patient can tolerate

D. Balanced, essential nutrition in a form that the patient can tolerate Rationale: A patient with increased ICP is in a hypermetabolic and hypercatabolic state and needs adequate glucose to maintain fuel for the brain and other nutrients to meet metabolic needs. Malnutrition promotes cerebral edema and if a patient cannot take oral nutrition, other means of providing nutrition should be used, such as tube feedings or parenteral nutrition. Glucose alone is not adequate to meet nutritional requirements, and 5% dextrose solutions may increase cerebral edema by lowering serum osmolarity. Patients should remain in a nonvolemic state with close monitoring of clinical factors such as urine output, fluid intake, serum and urine osmolality, serum electrolytes, and insensible loses

Which information about continuous bladder irrigation will the nurse teach to a patient who is being admitted for a transurethral resection of the prostate (TURP)? A. Bladder irrigation decreases the risk of postoperative bleeding B. Hydration and urine output are maintained by bladder irrigation C. Antibiotics are infused continuously through the bladder irrigation D. Bladder irrigation prevents obstruction of the catheter after surgery

D. Bladder irrigation prevents obstruction of the catheter after surgery Rationale: The purpose of bladder irrigation is to remove clots from the bladder and prevent obstruction of the catheter by clots. The irrigation does not decrease bleeding or improve hydration. Antibiotics are given by the IV route, not through bladder irrigation

The nurse identifies a patient with type 1 diabetes and a history of herpes simplex infection as being at risk for Bell's palsy. Which information should the nurse include in teaching the patient? A. You may be able to prevent Bell's palsy by doing facial exercises regularly B. Prophylactic treatment of herpes with antiviral agents prevents Bell's palsy C. Medications to treat Bell's palsy work only if started before paralysis onset D. Call the doctor if you experience pain or develop herpes lesions near the ear

D. Call the doctor if you experience pain or develop herpes lesions near the ear Rationale: Pain or herpes lesions near the ear may indicate the onset of Bell's palsy, and rapid corticosteroid treatment may reduce the duration of Bell's palsy symptoms. Antiviral therapy for herpes simplex does not reduce the risk for Bell's palsy. Corticosteroid therapy will be most effective in reducing symptoms if started before paralysis is complete but will still be somewhat effective when started later. Facial exercises do not prevent Bell's palsy

A female patient with a purulent vaginal discharge is seen at an outpatient clinic. The nurse suspects a diagnosis of gonorrhea. How would this STI be treated? A. Oral acyclovir (Zovirax) B. Penicillin G Benzathine given IM C. Need a confirmatory test result before treatment D. Ceftriaxone IM with oral azithromycin (Zithromax)

D. Ceftriaxone IM with oral azithromycin (Zithromax) Rationale: Gonorrhea is treated with dual therapy of a single dose of IM ceftriaxone and oral azithromycin (Zithromax). Acyclovir would be used for HSV. Penicillin was used to treat gonorrhea, but gonorrhea is now resistant to penicillin. Penicillin G is used to treat syphilis. Nucleic acid amplification testing (NAAT) is used to confirm the diagnosis in women. Because of a short incubation period and high rates of infectivity, treatment for gonorrhea is often given without waiting for positive test results

A thoracic aortic aneurysm is found when a patient has a routine chest x-ray. The nurse anticipates that additional diagnostic testing to determine the size and structure of the aneurysm will include which test? A. Angiography B. Ultrasonography C. Echocardiography D. Computed tomography (CT) scan

D. Computed tomography (CT) scan Rationale: A computed tomography (CT) scan is the most accurate test to determine the length and diameter of the aneurysm and whether a thrombus is present. The other tests may also be used, but the CT scan yields the most descriptive results

To reduce the risk for falls in the patient with Parkinson's disease, what is the best thing the nurse should teach the patient to do? A. Use an elevated toilet seat B. Use a wheelchair for mobility C. Use a walker or cane for support D. Consciously lift the toes when stepping

D. Consciously lift the toes when stepping Rationale: The shuffling gait of PD causes the patient to be off balance and at risk for falling. Teaching the patient to use a wide stance with the feet apart, to lift the toes when walking, and to look ahead helps to promote a more balanced gait. Use of an elevated toilet seat will enable a patient to initiate movement but not prevent falls. Using a wheelchair will not maintain independence or optimize psychosocial wellbeing. Canes and walkers are difficult for patients with PD to maneuver and may make thee patient more prone to injury

Which observation should indicate to the nurse the presence of the complication of graft thrombosis after AAA repair? A. Cardiac dysrhythmias or chest pain B. Absent bowel sounds, abdominal distention, or diarrhea C. Increased temperature and increased white blood cell count D. Decreased pulses and cool, painful extremities below the level of repair

D. Decreased pulses and cool, painful extremities below the level of repair Rationale: Decreased or absent pulses in conjunction with cool, painful extremities below the level of repair indicate graft thrombosis. Cardiac dysrhythmias or chest pain indicates myocardial ischemia. Absent bowel sounds, abdominal distention, diarrhea, or bloody stools indicate bowel infarction. Increased temperature and white blood cells, surgical site inflammation, or drainage indicates graft infection.

The nurse will anticipate teaching a patient with a possible seizure disorder about which test? A. Cerebral angiography B. Evoked potential studies C. Electromyography (EMG) D. Electroencephalography (EEG)

D. Electroencephalography (EEG) Rationale: Seizure disorders are usually assessed using EEG testing. Evoked potential is used to diagnose problems with the visual or auditory systems. Cerebral angiography is used to diagnose vascular problems. EMG is used to evaluate electrical innervation to skeletal muscle

How do generalized seizures differ from focal seizures? A. Focal seizures are confined to one side of the brain and remain focal in nature B. Generalized seizures result in loss of consciousness, whereas focal seizures do not C. Generalized seizures result in temporary residual deficits during the postictal phase D. Generalized seizures have bilateral synchronous epileptic discharges affecting the the whole brain at onset of the seizure

D. Generalized seizures have bilateral synchronous epileptic discharges affecting the the whole brain at onset of the seizure Rationale: Generalized seizures have bilateral synchronous epileptic discharge affecting the entire brain at onset of the seizure. Focal seizures begin in one side of the brain but may spread to involve the entire brain. Loss of consciousness is characteristic of generalized seizures but complex focal seizures also include an altered consciousness. Focal seizures that start with a local focus and spread to the entire brain, causing a secondary generalized seizure, are associated with a transient residual neurologic deficit postictally known as Todd's paralysis

Which STI actively occurring at the time of delivery would indicate the need for a cesarean section delivery of the woman's baby? A. Syphilis B. Chlamydia C. Gonorrhea D. Genital herpes

D. Genital herpes Rationale: Women with an active HSV genital lesion at the time of delivery have the highest risk of transmitting genital herpes to the neonate, so delivery will be done with a cesarean section (C-section). Syphilis is spread to the fetus in utero and has a high risk of stillbirth, but C-sections are not required. Treatment with parenteral penicillin will cure both the mother and the fetus. Prevention of the spread of gonorrhea to the neonate's eyes is done with erythromycin ophthalmic ointment or silver nitrate aqueous solution. Chlamydia spread to the fetus can be prevented by treating the pregnant woman, so a C-section is not required

Which nursing intervention for a patient who had an open repair of an abdominal aortic aneurysm 2 days previously is appropriate for the nurse to delegate to unlicensed assistive personnel (UAP)? A. Monitor the quality and presence of the pedal pulses B. Teach the patient the signs of possible wound infection C. Check the lower extremities for strength and movement D. Help the patient to use a pillow to splint while coughing

D. Help the patient to use a pillow to splint while coughing Rationale: Assisting a patient who has already been taught how to cough is part of a routine postoperative care and within the education and scope of practice for a UAP. Patient teaching and assessment of essential postoperative functions such as circulation and movement should be done by RNs

Following discharge teaching with a male patient with an AAA repair, the nurse determines that further instruction is needed when the patient says, A. I should avoid heavy lifting for 6 weeks B. I may have some sexual dysfunction as a result of the surgery C. I should maintain a low-fat and low-cholesterol diet to help keep the new graft open D. I should take the pulses in my extremities and let the doctor know if they get too fast or too slow.

D. I should take the pulses in my extremities and let the doctor know if they get too fast or too slow. Rationale: Patients are taught to palpate peripheral pulses to identify changes in their quality or strength, but the rate is not a significant factor in peripheral perfusion. The color and temperature of the extremities are also important for patients to observe. The remaining statements are all true.

A 29 year old woman is at the clinic with heavy menstrual bleeding. What finding would the nurse expect in this patient's assessment? A. Pain with each menstrual period B. Excessive bleeding at irregular intervals C. Bleeding between regular menstrual cycles D. Increased duration or amount of menstrual bleeding

D. Increased duration or amount of menstrual bleeding Rationale: Heavy menstrual bleeding in increased duration or amount of bleeding with menses. Pain with menses is called dysmenorrhea. Chronic abdominal uterine bleeding is classified as uterine bleeding that is abnormal in volume, timing, or regularity and has been present for most of the past 6 months. Intermenstrual bleeding is bleeding between regular menstrual cycles

To help prevent embolization of the thrombus in a patient with acute VTE and severe edema and limb pain, what should the nurse teach the patient to do first? A. Dangle the feet over the edge of the bed every 2-3 hours B. Ambulate around the bed three to four times a day C. Keep the affected leg elevated above the heart D. Maintain bed rest until edema is relieved and anticoagulation is established

D. Maintain bed rest until edema is relieved and anticoagulation is established Rationale: With acute VTE, prevention of emboli formation, decreased edema, and pain can be achieved initially by bed rest and limiting movement of the involved extremity. Ambulation will be the next priority. Dangling the legs promotes venous stasis and further clot formation. Elevating the affected limb will promote venous return, but it does not prevent embolization

Which drug treatment helps to decrease ICP by expanding plasma and the osmotic effect to move fluid? A. Dexamethasone B. Oxygen administration C. Pentobarbital (Nembutal) D. Mannitol (Osmitrol) (25%)

D. Mannitol (Osmitrol) (25%) Rationale: Mannitol (Osmitrol)(25%) is an osmotic diuretic that expands plasma and causes fluid to move from tissues into the blood vessels. Hypertonic saline reduces brain swelling by moving water out of brain tissue. The corticosteroid dexamethasone is used to treat vasogenic edema to stabilize cell membranes and improve neuronal function by improving CBF and restoring autoregulation. Oxygen administration is done to maintain brain function. Pentobarbital (Nembutal) and other barbiturates are used to reduce cerebral metabolism

After a transurethral resection of the prostate (TURP), a 64-year-old patient with continuous bladder irrigation complains of painful bladder spasms. The nurse observes clots in the urine. Which action should the nurse take first? A. Increase the flow rate of the bladder irrigation. B. Administer the prescribed IV morphine sulfate. C. Give the patient the prescribed belladonna and opium suppository. D. Manually instill and then withdraw 50 mL of saline into the catheter.

D. Manually instill and then withdraw 50 mL of saline into the catheter. Rationale: The assessment suggests that obstruction by a clot is causing the bladder spasms, and the nurse's first action should be to irrigate the catheter manually and to try to remove the clots. IV morphine will not decrease the spasm, although pain may be reduced. Increasing the flow rate of the irrigation will further distend the bladder and may increase spasms. The belladonna and opium suppository will decrease bladder spasms but will not remove the obstructing blood clot

What N-methyl-d-aspartate (NMDA) receptor antagonist is frequently used for a patient with AD who is experiencing decreased memory and cognition? A. Zolpidem (Ambien) B. Olanzapine (Zyprexa) C. Rivastigmine (Exelon) D. Memantine (Namenda)

D. Memantine (Namenda) Rationale: Memantine (Namenda) is the N-methyl-d-aspartate (NMDA) receptor antagonist frequently used for AD patients with decreased memory and cognition. Zolpidem (Ambien) is a sedative to help with sleep problems. Olanzapine (Zyprexa) is an antipsychotic medication used for behavior management. Rivastigmine (Exelon) is a cholinesterase inhibitor used for decreased memory and cognition

The husband of a patient is complaining that his wife's memory has been decreasing lately. When asked for examples of her memory loss, the husband says that she is forgetting the neighbors' names and forgot their granddaughter's birthday. What kind of loss does the nurse recognize this to be? A. Delirium B. Memory loss in AD C. Normal forgetfulness D. Memory loss in mild cognitive impairment

D. Memory loss in mild cognitive impairment Rationale: In mild cognitive impairment, people frequently forget people's names and begin to forget important events. Delerium changes usually occur abruptly. In Alzheimer's disease the patient may not remember knowing a person and loses the sense of time and which day it is. Normal forgetfulness includes momentarily forgetting names and occasionally forgetting to run an errand

When caring for a patient in the severe stage of AD, the nurse could use what diversion or distraction activities? A. Watching TV B. Books to read C. Playing games D. Mobiles or dangling ribbons

D. Mobiles or dangling ribbons Rationale: In the severe stage of AD, the patient is at a developmental level of 15 months of less; therefore appropriate distractions would be infant toys. Watching TV and playing games are more appropriate in the mild to moderate stages. Books to read should be at developmentally appropriate levels to be used as a diversion

After change of shift report on the Alzheimer's disease/dementia unit, which patient will the nurse assess first? A. Patient who has not had a bowel movement for 5 days B. Patient who has a stage II pressure ulcer on the coccyx C. Patient who is refusing to take the prescribed medications D. Patient who developed a new cough after eating breakfast

D. Patient who developed a new cough after eating breakfast Rationale: A new cough after a meal in a patient with dementia suggests possible aspiration, and the patient should be assessed immediately. The other patients also require assessment and intervention but not as urgently as a patient with possible aspiration or pneuomonia

The patient's diagnosis is a large rectocele requiring surgery. What nursing interventions will be the priority postoperatively? A. An ice pack to relieve the swelling B. An enema each day to relieve constipation C. Administration of a stool softener each night D. Perineal care after each urination or defecation

D. Perineal care after each urination or defecation Rationale: The primary goal of care is to prevent wound infection and pressure on the vaginal incision, which requires perineal cleansing at least twice daily and after each urination and defecation. An ice pack and stool softener will be used, but they are not the priority. The enema would have been done preoperatively

What should the nurse do to prepare a patient for a lumbar puncture? A. Sedate the patient with medication before the test B. Withhold beverages containing caffeine for 8 hours C. Assess the patient for a stroke before the procedure for baseline data D. Position the patient in a lateral recumbent position with the hips, knees, and neck flexed

D. Position the patient in a lateral recumbent position with the hips, knees, and neck flexed Rationale: To facilitate insertion of the spinal needle between the third and fourth lumbar vertebrae, the patient should round the spine by flexing the knees, hips and neck while in a lateral recumbent position, although a seated position may also be used. Sedation is used for more invasive tests, such as myelograms and angiography. Stimulants are withheld for 8 hours before an electroencephalogram (EEG). Assessing for stroke symptoms is not needed before a lumbar puncture but is done before cerebral angiography

What is an explanation that the nurse should give to the wife of the patient who ask what her husband's diagnosis of paraphimosis means? A. Painful, prolonged erection B. Inflammation of the epididymis C. Painful downward curvature of an erect penis D. Retracted tight foreskin preventing return over the glans

D. Retracted tight foreskin preventing return over the glans Rationale: Paraphimosis is tightness of the foreskin and the inability to pull it forward from a retracted position to return it over the glans. It is usually associated with leaving the foreskin pulled back during a bath, use of a urinary catheter, or intercourse. Painful, prolonged erection is priapism. Epididymitis is inflammation of the epididymis. An painful downward curvature of an erect penis is chordee

The patient is a perimenopausal woman who has an unexpected and unwanted pregnancy. She wants an abortion. What should the nurse teach her about the effects of an abortion? A. D&C will be needed B. She will feel much better afterward C. The products of conception will pass immediately D. She will need someone to support her through her loss

D. She will need someone to support her through her loss Rationale: There is physical and emotional pain and grieving after an abortion that puts the patient in need of support. D&C is needed only if the products of conception do not pass completely or bleeding becomes excessive. The time it takes for the products of conception to pass depends on the type of abortion being done and is immediate with surgical abortion and slower with medical abortion

What method is used to assess for extinction? A. Cotton wisp B. Sharp and dull end of a pin C. Tuning fork to bony prominences D. Simultaneously touching both sides of the body

D. Simultaneously touching both sides of the body Rationale: Extinction is assessed by simultaneously touching both sides of the body; it is abnormal if the patient extinguishes one stimulus and perceives the stimulus only on one side. The cotton wisp assesses light touch. Pain sensation is assessed by touching the sharp and dull end of a pin to each of the patient's limbs with the patient responding "sharp" or "dull" each time. A tuning fork to bony prominences assesses vibration sense

Which drug therapy is included for acute migraine and cluster headaches that appears to alter the pathophysiologic process for these headaches? A. Tricyclic antidepressants such as amitriptyline B. Nonsteroidal antiinflammatory drugs (NSAIDs) C. B-Adrenergic blockers such as propranolol (Inderal) D. Specific serotonin receptor agonists such as sumatriptan (Imitrex)

D. Specific serotonin receptor agonists such as sumatriptan (Imitrex) Rationale: Triptans (sumatriptan [Imitrex]) affect selected serotonin receptors that decrease neurogenic inflammation of the cerebral blood vessels and produce vasoconstriction. Both migraine headaches and cluster headaches appear to be related to vasodilation of cranial vessels and drugs that cause vasoconstriction are useful in treatment of migraine and cluster headaches. Tricyclic antidepressants and B-adrenergic blockers are used prophylactically for migraine headaches but are not effective for cluster headaches. NSAIDs may be used for treatment of migraine headaches, but do not change the pathophysiologic process of migraine or cluster headaches

A patient has been admitted with meningococcal meningitis. Which observation by the nurse requires action? A. The patient receives a regular diet tray B. The bedrails on both sides of the bed are elevated C. Staff have turned off the lights in the patient's room D. Staff have entered the patient's room without a mask

D. Staff have entered the patient's room without a mask Rationale: Meningococcal meningitis is spread by respiratory secretions, so it is important to maintain respiratory isolation as well as standard precautions. Because the patient may be confused and weak, bedrails should be elevated at both the foot and head of the bed. Low light levels in the room decrease pain caused by photophobia. Nutrition is an important aspect of care in a patient with meningitis

The couple has not been able to become pregnant. The wife has not been diagnosed with any fertility problems. Which treatment will the nurse expect to teach the couple about if the problem is the most common testicular problem causing male infertility? A. Antibiotics B. Semen analysis C. Avoidance of scrotal heat D. Surgery to correct the problem

D. Surgery to correct the problem Rationale: Varicocele is the most common testicular cause of infertility. Surgical ligation of the spermatic vein is done to correct the problem. Antibiotics are used if there is an infection, but this is not as common as a varicocele. Semen analysis is the first study done when investigating male infertility, but it is not a treatment. Avoidance of scrotal heat is a lifestyle change that may be used with idiopathic infertility

A patient being admitted with bacterial meningitis has a temperature of 102.5 and a severe headache. Which order should the nurse implement first? A. Administer ceftizoxime (Cefizox) 1g IV B. Give acetaminophen (Tylenol) 650mg PO C. Use a cooling blanket to lower temperature D. Swab the nasopharyngeal mucosa for cultures

D. Swab the nasopharyngeal mucosa for cultures Rationale: Antibiotic therapy should be instituted rapidly in bacterial meningitis, but cultures must be done before antibiotics are started. As soon as the cultures are done, the antibiotic should be started. Hypothermia therapy and acetaminophen administration are appropriate but can be stated after the other actions are implemented

A 19 year old patient has been diagnosed with primary dysmenorrhea. How will the nurse suggest that the patient manage discomfort? A. Avoid aerobic exercise during her menstrual period B. Use cold packs on the abdomen and back for pain relief C. Talk with her health care provider about beginning antidepressant therapy D. Take nonsteroidal antiinflammatory drugs (NSAIDs) when her period starts

D. Take nonsteroidal antiinflammatory drugs (NSAIDs) when her period starts Rationale: NSAIDs should be started as soon as the menstrual period begins and taken at regular intervals during the usual time frame when pain occurs. Aerobic exercise may help reduce symptoms. Heat therapy, such as warm packs, is recommended for relief of pain. Antidepressant therapy is not a typical treatment for dysmenorrhea

The nurse is caring for a patient who was admitted the previous day with a basilar skull fracture after a motor vehicle crash. Which assessment finding indicates a possible complication that should be reported to the health care provider? A. Complaint of severe headaches B. Large contusion behind left ear C. Bilateral periorbital ecchymosis D. Temperature of 101.4

D. Temperature of 101.4 Rationale: Patients who have a basilar skull fracture are at risk for meningitis, so the elevated temperature should be reported to the health care provider. The other findings are typical of a patient with a basilar skull fracture

In the neurologic nursing assessment, the patient is unable to hear a ticking watch. What neurologic problem could be the cause of this finding? A. The patient is distracted B. The patient is hard of hearing C. The vagus (CN X) nerve is malfunctioning D. The cochlear branch of the acoustic (CN VIII) nerve is damaged

D. The cochlear branch of the acoustic (CN VIII) nerve is damaged Rationale: The cochlear branch of the acoustic nerve (CN VIII) should enable the patient to hear. The patient may be distracted or hard of hearing, but the damage to this nerve is the most likely neurologic problem causing the inability to hear the ticking watch. The nurse should ensure that there are no distractions or extraneous noise when performing this test. The vagus (CN X) nerve is unrelated to hearing

A 66 year old patient is experiencing ED. He and his wife have used tadalafil (Cialis), but because he experienced priapism they have decided to change their treatment option to an intraurethral device. How should the nurse explain how this device works? A. The device relaxes smooth muscle in the penis B. Blood is drawn into corporeal bodies and held with a ring C. The device is implanted into corporeal bodies to firm the penis D. The device directly applies drugs that increase blood flow to the penis

D. The device directly applies drugs that increase blood flow to the penis Rationale: Intraurethral devices include the use of vasoactive drugs administered as a topical gel or medication pellet (alprostadil) inserted into the urethra (intraurethral) using a medicated urethral system for erection (MUSE) device, or an injection into the penis (intracavernosal self injection). The vasoactive drugs enhance blood flow into the penile arteries for erection. Erectogenic drugs (e.g. tadalafil [Cialis]) cause smooth muscle relaxation and increase blood flow to promote an erection. Blood drawn into corporeal bodies and held with a ring is achieved with a vacuum constriction device (VCD). Devices implanted into corporeal bodies to firm the penis are penile implants. Androgen or testosterone replacement therapy may also be used for erectile dysfunction

Why should the nurse encourage serologic testing for human immunodeficiency virus (HIV) in the patient with syphilis? A. Syphilis is more difficult to treat in patients with HIV infection B. She should question her fiance about prior sexual contacts C. Additional testing to detect specific antitreponemal antibodies is necessary D. The incidence of syphilis is increased in those with high rates of indiscriminate sexual activity and drug abuse

D. The incidence of syphilis is increased in those with high rates of indiscriminate sexual activity and drug abuse Rationale: The risk factors of drug abuse and sexual activity with multiple partners or homosexuality are found in patients with both syphilis and human immunodeficiency virus (HIV) infection and persons at highest risk for acquiring syphilis are also at high risk for acquiring HIV. Syphilitic lesions on the genitals enhance HIV transmission. Also, HIV infected patients with syphilis appear to be at greatest risk for central nervous system (CNS) involvement and may require more intensive treatment with penicillin to prevent this complication of HIV

Skull x-rays and a CT scan provide evidence of a depressed parietal fracture with a subdural hematoma in a patient admitted to the ED following an automobile accident. In planning care for the patient, what should the nurse anticipate? A. The patient will receive life support measures until the condition stabilizes. B. Immediate burr holes will be made to rapidly decompress the intracranial cavity. C. The patient will be treated conservatively with close monitoring for changes in neurologic status. D. The patient will be taken to surgery for a craniotomy for evacuation of blood and decompression of the cranium.

D. The patient will be taken to surgery for a craniotomy for evacuation of blood and decompression of the cranium. Rationale: When there is a depressed fracture or a fracture with loose fragments, a craniotomy is indicated to elevate the depressed bone and remove free fragments. A craniotomy is also indicated in cases of acute subdural and epidural hematomas to remove the blood and control the bleeding. Burr holes may be used in an extreme emergency for rapid decompression or to aid in removing a bone flap but with a depressed fracture, surgery would be the treatment of choice

When assessing a 53 year old patient with bacterial meningitis, the nurse obtains the following data. Which finding requires the most immediate intervention? A. The patient exhibits nuchal rigidity B. The patient has a positive Kernig's sign C. The patient's temperature is 101 D. The patient's blood pressure is 88/42

D. The patient's blood pressure is 88/42 Rationale: Shock is a serious complication of meningitis, and the patient's low blood pressure indicates the need for interventions such as fluids or vasopressors. Nuchal rigidity and a positive Kernig's sign are expected with bacterial meningitis. The nurse should intervene to lower the temperature, but this is not life threatening as the hypotension

An 18 year old patient with irregular menstrual periods, hirsutism, and obesity has been diagnosed with polycystic ovary syndrome (PCOS). What is an accurate rationale for the expected treatment? A. Hirsutism may be treated with leuprolide to decrease an altered body image B. The medication used will cure the hormonal abnormality of excess testosterone C. The loss of weight will improve all of the symptoms, so this will be the first treatment tried D. The progression of PCOS leads to cardiovascular disease and abnormal insulin resistance if untreated

D. The progression of PCOS leads to cardiovascular disease and abnormal insulin resistance if untreated Rationale: Left untreated, polycystic ovary syndrome (PCOS) may lead to cardiovascular disease and abnormal insulin resistance with type 2 diabetes mellitus. Hirsutism may be treated with spironolactone. Leuprolide is used to treat, hyperandrogenism, but PCOS cannot be cured. Severity of symptoms is associated with obesity, but the hormone abnormalities will be treated along with the obesity to prevent complications. If other treatment is not successful, a hysterectomy with bilateral salpingectomy and oophorectomy may be performed

A 68 year old man is admitted to the ED with multiple blunt trauma following a one vehicle car accident. He is restless; disoriented to person, place, and time; and agitated, He resists attempts at examination and calls out the name Janice. Why should the nurse suspect delirium rather than dementia in this patient? A. The fact that he should not have been allowed to drive if he had dementia B. His hyperactive behavior, which differentiates his condition from the hypoactive behavior of dementia C. The report of the emergency personnel that he was noncommunicative when they arrived at the accident scene D. The report of his family that, although he has heart disease and is very hard of hearing, this behavior is unlike him

D. The report of his family that, although he has heart disease and is very hard of hearing, this behavior is unlike him Rationale: Delirium is an acute problem that usually has a rapid onset in response to a precipitating event, especially when the patient has underlying health problems, such as heart disease and sensory limitations. In the absence of prior cognitive impairment, a sudden onset of confusion, disorientation, and agitation is usually delirium. Delirium may manifest with both hypoactive and hyperactive symptoms

Premenstrual syndrome (PMS) is most likely to be diagnosed in a woman with which occurrence? A. Symptoms that can be controlled with the use of progesterone B. The woman has symptoms only when contraceptives are used C. Symptoms can be correlated with altered serum levels of estrogen and progesterone D. The woman has consistent syndrome complex with symptoms ending after menses begins

D. The woman has consistent syndrome complex with symptoms ending after menses begins Rationale: Premenstrual syndrome (PMS) is diagnosed when other possible causes for symptoms have been eliminated. A diagnosis is based on (1) consistency of the syndrome complex, (2) occurrence of symptoms in the luteal phase and resolution after menses begins, (3) documented ovulatory cycles, and (4) symptoms that disrupt the woman's life. Oral contraceptives may be used to control the symptoms of PMS by suppressing ovulation, and although progesterone may also relieve the symptoms of PMS, its effectiveness is not associated with the diagnosis of PMS. There are no laboratory findings that account for the premenstrual symptoms

The nurse is reinforcing teaching with a patient newly diagnosed with amyotrophic lateral sclerosis (ALS). Which statement would be appropriate to include in the teaching? A. ALS results from an excess chemical in the brain, and the symptoms can be controlled with medication B. Even though the symptoms you are experiencing are severe, most people recover with treatment C. You need to consider advance directives now, because you will lose cognitive function as the disease progresses D. This is a progressing disease that eventually results in permanent paralysis, though you will not lose any cognitive function

D. This is a progressing disease that eventually results in permanent paralysis, though you will not lose any cognitive function Rationale: The disease results in destruction of the motor neurons in the brainstem and spinal cord, causing gradual paralysis. Cognitive function is maintained. Because no cure exists for amyotrophic lateral sclerosis (ALS), interprofessional care is palliative and based on symptom relief. Death often occurs within 2 to 5 years after diagnosis

A patient is being treated with carbidopa/levodopa (Sinemet) for Parkinson's disease. Which information indicates a need for change in the medication or dosage? A. Shuffling gait B. Tremor at rest C. Cogwheel rigidity of limbs D. Uncontrolled head movement

D. Uncontrolled head movement Rationale: Dyskinesia is an adverse effect of the Sinemet, indicating a need for a change in medication or decrease in dose. The other findings are typical with Parkinson's disease

When discussing risk factors modification for a patient who has a 5cm abdominal aortic aneurysm, the nurse will focus teaching on which patient risk factor? A. Male gender B. Turner syndrome C. Abdominal trauma history D. Uncontrolled hypertension

D. Uncontrolled hypertension Rationale: All of the factors contribute to the patient's risk, but only hypertension can potentially be modified to decrease the patient's risk for further expansion of the aneurysm

Which situation most accurately describes dementia? A. Overproduction of B-amyloid protein causes all dementias B. Dementia resulting from neurodegenerative causes can be prevented C. Dementia caused by hepatic or renal encephalopathy cannot be reversed D. Vascular dementia can be diagnosed by brain lesions identified with neuroimaging

D. Vascular dementia can be diagnosed by brain lesions identified with neuroimaging Rationale: The diagnosis of vascular dementia can be aided by neuroimaging studies showing vascular brain lesions along with exclusion of other causes of dementia. Overproduction of B-amyloid protein contributes to Alzheimer's disease (AD). Vascular dementia can be prevented or slowed by treating underlying diseases (e.g. diabetes mellitus, cardiovascular disease). Dementia caused by hepatic or renal encephalopathy potentially can be reversed

A patient with intracranial pressure monitoring has a pressure of 12 mmHg. The nurse understands that this pressure reflects A. a severe decrease in cerebral perfusion pressure B. an alteration in the production of cerebral spinal fluid C. the loss of autoregulatory control of intracranial pressure D. a normal balance between brain tissue, blood, and cerebrospinal fluid

D. a normal balance between brain tissue, blood, and cerebrospinal fluid Rationale: Normal intracranial pressure (ICP) is 5 to 15 mmHg. A sustained pressure above the upper limit is considered abnormal

In planning community education for prevention of spinal cord injuries, the nurse targets A. older men B. teenage girls C. elementary school-age children D. adolescent and young adult men

D. adolescent and young adult men Rationale: Young adult men ages 16 to 30 years, who are impulsive or risk takers in daily living, have the greatest risk for spinal cord injury (SCI). Other risk factors include alcohol and drug abuse as well as participation in sports and occupational exposure to trauma or violence

A patient is admitted to the hospital with possible bacterial meningitis. During the initial assessment, the nurse questions the patient about a recent history of what? A. mosquito or tick bites B. chickenpox or measles C. cold sores or fever blisters D. an upper respiratory infection

D. an upper respiratory infection Rationale: Meningitis is often the result of an upper respiratory infection or a penetrating wound of the skull, where organisms gain entry into the CNS. Epidemic encephalitis is transmitted by ticks and mosquitos, and nonepidemic encephalitis may occur as a complication of the measles, chickenpox, or mumps. Encephalitis caused by the herpes simplex virus carries a high fatality rate

The nurse finds an 87 year old woman with Alzheimer's disease is continually rubbing, flexing, and kicking her legs throughout the day. The night shift reports this same behavior escalates at night, preventing her from obtaining her required sleep. The next step the nurse should take is to A. ask the physician for a daytime sedative for the patient B. request soft restraints to prevent her from falling out of her bed C. ask the physician for a nighttime sleep medication for the patient D. assess the patient more closely, suspecting a disorder such as restless leg syndromes

D. assess the patient more closely, suspecting a disorder such as restless leg syndromes Rationale: The severity of sensory symptoms of restless legs syndrome (RLS) ranges from infrequent, minor discomfort (paresthesias, including numbness, tingling, and "pins and needles" sensation) to severe pain. The discomfort occurs when the patient is sedentary and is most common in the evening or at night. The pain at night can disrupt sleep and is often relieved by physical activity, such as walking, stretching, rocking, or kicking. In the most severe cases, patients sleep only a few hours at night, leading to daytime fatigue and disruption of the daily routine. The motor abnormalities associated with RLS include of voluntary restlessness and stereotyped, periodic, involuntary movements. The involuntary movements usually occur during sleep. Symptoms are aggravated by fatigue

The nurse is concerned about a postoperative patient's risk for injury during an episode of delirium. The most appropriate action by the nurse is to A. secure the patient in bed using a soft chest restraint. B. ask the health care provider to order an antipsychotic drug. C. instruct family members to remain with the patient and prevent injury. D. assign unlicensed assistive personnel (UAP) to stay with the patient and offer reorientation.

D. assign unlicensed assistive personnel (UAP) to stay with the patient and offer reorientation. Rationale: The priority goal is to protect the patient from harm. Having a UAP stay with the patient will ensure the patient's safety. Visits by family member are helpful in reorienting the patient, but families should not be responsible for protecting patients from injury. Antipsychotic medications may be ordered, but only if other measures are not effective because these medications have many side effects. Restraints are not recommended because they can increase the patient's agitation and disorientation

Drugs or diseases that impair the function of the extrapyramidal system may cause loss of A. sensations of pain and temperature. B. regulation of the autonomic nervous system. C. integration of somatic and special sensory inputs. D. automatic movements associated with skeletal muscle activity.

D. automatic movements associated with skeletal muscle activity. Rationale: A group of descending motor tracts carries impulses from the extrapyramidal system, which includes all motor systems (except the pyramidal system) concerned with voluntary movement. It includes descending pathways originating in the brainstem, basal ganglia, and cerebellum. The motor output exits the spinal cord by way of the ventral roots of the spinal nerves

The nurse is caring for a patient scheduled for an endometrial biopsy who is having difficulty becoming pregnant. The nurse explains to the woman that A. the outpatient procedure is usually done preovulation B. bleeding and discharge is common 2 to 4 days after the procedure C. a small sample of tissue is obtained to diagnose and treat cervical dysplasia D. common changes in endometrial cells in relation to progesterone levels will be assessed

D. common changes in endometrial cells in relation to progesterone levels will be assessed Rationale: Endometrial biopsy may be done in cases of infertility to assess if the lining of the uterus (endometrium) is going through the normal menstrual cycle changes including progesterone secretion post ovulation. The procedure may also be done to assess abnormal menstrual or postmenopausal uterine bleeding. Uterine cramping may occur during the procedure. A light amount of bloody vaginal bleeding may occur for about 24 hours after the procedure

When obtaining a health history from a 72 year old man with peripheral artery disease (PAD) of the lower extremities, the nurse asks about a history of related conditions, including A. venous thrombosis B. venous stasis ulcers C. pulmonary embolism D. coronary artery disease (CAD)

D. coronary artery disease (CAD) Rationale: Regardless of the location, atherosclerosis is responsible for peripheral artery disease (PAD) and is related to other cardiovascular disease and its risk factors, such as coronary artery disease (CAD) and carotid artery disease. Venous thrombosis, venous stasis ulcers, and pulmonary embolism are diseases of the veins and are not related to atherosclerosis

Which cranial surgery would require the patient to learn how to protect the surgical are from trauma? A. Burr holes B. craniotomy C. Cranioplasty D. craniectomy

D. craniectomy Rationale: A craniectomy is excision of cranial bone without replacement, so the patient will need to protect the brain from trauma in this surgical area. Burr holes are opened into the cranium with a drill to remove blood and fluid. A craniotomy is opening the cranium with removal of a bone flap to open the dura. The replaced bone flap is wired or sutured after surgery. A cranioplasty replaces part of the cranium with an artificial plate

When caring for a patient who has a radium implant for treatment of cervical cancer, the nurse will A. assist the patient to ambulate every 2 to 3 hours B. use gloves and gown when changing the patient's bed C. flush the toilet several times right after the patient voids D. encourage the patient to discuss any concerns by telephone

D. encourage the patient to discuss any concerns by telephone Rationale: The nurse should spend minimal time in the patient's room to avoid exposure to radiation. The patient and nurse can have longer conversations by telephone between the patient room and nursing station. To prevent displacement of the implant, absolute bed rest is required. Wearing of gloves and gown when changing linens and flushing the toilet several times are not necessary because the isotope is confined to the implant

Nursing responsibilities related to the patient with endometrial cancer who has a total abdominal hysterectomy and salpingectomy and oophorectomy include A. maintaining absolute bed rest B. keeping the patient in high Fowler's position C. need for supplemental estrogen after removal of ovaries D. encouraging movement and walking as much as tolerated

D. encouraging movement and walking as much as tolerated Rationale: After an abdominal hysterectomy and salpingectomy and oophorectomy the nurse should encourage the patient to be mobile. Bed rest is not indicated and may increase complications. Estrogen is not used following surgery

Postoperatively, a patient who has had a laser prostatectomy has continuous bladder irrigation with a three-way urinary catheter with a 30-mL balloon. When he complains of bladder spasms with the catheter in place, you should A. deflate the catheter balloon to 10 mL to decrease bulk in the bladder. B. deflate the catheter balloon and then reinflate it to ensure that it is patent. C. encourage the patient to try to have a bowel movement to relieve colon pressure. D. explain that this feeling is normal and that he should not try to urinate around the catheter.

D. explain that this feeling is normal and that he should not try to urinate around the catheter. Rationale: Bladder spasms occur as a result of irritation of the bladder mucosa from the insertion of the resectoscope, presence of a catheter, or clots that cause obstruction of the catheter. Thee nurse should instruct the patient not to urinate around the catheter because this increases the likelihood of spasm

The nurse explains to a patient newly diagnosed with MS that the diagnosis is made primarily by A. spinal xray findings B. T-cell analysis of the blood C. analysis of cerebrospinal fluid D. history and clinical manifestations

D. history and clinical manifestations Rationale: There is no specific diagnostic test for MS. A diagnosis is made primarily by history and clinical manifestations. Certain diagnostic tests may be used to help establish a diagnosis of MS. Positive findings on MRI include evidence of at least two inflammatory demyelinating lesions in at least two different locations within the central nervous system (CNS). Cerebrospinal fluid (CSF) may have increased immunoglobulin G and the presence of oligoclonal banding. Evoked potential responses are often delayed in persons with MS

Several hours after a patient had an open surgical repair of an abdominal aortic aneurysm, the UAP reports to the nurse that urinary output for the past 2 hours has been 45mL. The nurse notifies the health care provider and anticipates an order for a(n) A. hemoglobin count B. additional antibiotic C. serum creatinine level D. increased IV infusion rate

D. increased IV infusion rate Rationale: The decreased urine output suggests decreased renal perfusion and monitoring of renal function is needed. There is no indication that infection is a concern, so antibiotic therapy and a WBC count are not needed. The IV rate may be increased because hypovolemia may be contributing to the patient's decreased urinary output

A patient is being evaluated for postthrombotic syndrome. Which assessment will the nurse perform? A. Ask about leg pain with exercise B. Determine the ankle brachial index C. Assess capillary refill in the patient's toes D. inspect for presence of lipodermatosclerosis

D. inspect for presence of lipodermatosclerosis Rationale: Clinical signs of postthrombotic syndrome include lipodermatosclerosis. In this situation, the skin on the lower leg becomes scarred, and the leg becomes tapered like an "inverted bottle." The other assessments would be done for patients with peripheral artery disease

The nurse is caring for a patient with peripheral neuropathy who is scheduled for EMG studies tomorrow morning. The nurse should A. ensure the patient has an empty bladder B. instruct the patient about the risk of electric shock C. ensure the patient has not metallic jewelry or metal fragments D. instruct the patient that pain may be experienced during the study

D. instruct the patient that pain may be experienced during the study Rationale: Electromyography (EMG) is used to assess electrical activity associated with nerves and skeletal muscles. Activity is recorded by insertion of needle electrodes to detect muscle and peripheral nerve disease. The nurse should inform the patient that pain and discomfort are associated with insertion of needles. There is no risk of electric shock with this procedure

An older patient with chronic atrial fibrillation develops sudden severe pain, pulselessness, pallor, and coolness in the right leg. The nurse should notify the health care provider and immediately A. apply a compression stocking to the leg B. elevate the leg above the level of the heart C. assist the patient in gently exercising the leg D. keep the patient in bed in the supine position

D. keep the patient in bed in the supine position Rationale: The patient's history and clinical manifestations are consistent with acute arterial occlusion, and resting the leg will decrease the O2 demand of the tissues and minimize ischemic damage until circulation can be restored. Elevating the leg or applying an elastic wrap will further compromise blood flow to the leg. Exercise will increase oxygen demand for the tissues of the leg.

Which indirect thrombin inhibitor is only administered subcutaneously and does not need routine coagulation tests? A. warfarin (Coumadin) B. Unfractionated heparin C. Hirudin derivatives (bivalirudin [Angiomax]) D. low molecular weight heparin (enoxaparin [Lovenox])

D. low molecular weight heparin (enoxaparin [Lovenox]) Rationale: Low molecular weight heparin (enoxaparin [Lovenox]) is only given subcutaneously and does not require routine coagulation testing. Unfractionated heparin is the only other indirect thrombin inhibitor option, but it can be given subcutaneously or IV and therapeutic effects must be monitored with coagulation testing.

A patient was recently diagnosed with polycystic ovary syndrome. It is most important for the nurse to teach the patient A. reasons for a total hysterectomy B. how to decrease facial hair growth C. ways to reduce the occurrence of acne D. methods to maintain appropriate weight

D. methods to maintain appropriate weight Rationale: Obesity exacerbates the problems associated with polycystic ovary syndrome, such as insulin resistance and type 2 diabetes. The nurse should also address the problems of acne and hirsutism, but these symptoms are lower priority because they do not have long term health consequences. Although some patients do require total hysterectomy, it is usually performed only after other therapies have been unsuccessful

During the physical assessment of a female patient with HPV infection, what should the nurse expect to find? A. Purulent vaginal drainage B. Painful perineal vesicles and ulcerations C. A painless, indurated lesion on the vulva D. multiple coalescing gray warts in the perineal area

D. multiple coalescing gray warts in the perineal area Rationale: HPV is responsible for causing genital warts, which manifest as discrete single or multiple white to gray warts that may coalesce to form large cauliflower like masses on thee vulva, vagina, cervix, and perianal area. Purulent vaginal discharge is associated with gonorrhea or chlamydia. Painful perineal vesicles and ulcerations are characteristic of genital herpes and a chancre of syphilis is a painless, indurated lesion on the vulva, vagina, lips, or mouth

Following a T2 spinal cord injury, the patient develops paralytic ileus. While this condition is present, what should the nurse anticipate that the patient will need? A. IV fluids B. tube feedings C. parenteral nutrition D. nasogastric suctioning

D. nasogastric suctioning Rationale: During the first 2 to 3 days after a SCI, paralytic ileus may occur and nasogastric suction must be used to remove secretions and gas from the GI tract until peristalsis resumes. IV fluids are used to maintain fluid balance but do not specifically relate to paralytic ileus. Tube feedings would be used only for patient who have difficulty swallowong and not until peristalsis returns. Parenteral nutrition would be used only if the paralytic ileus was unusually prolonged

In assessing a patient for testicular cancer, the nurse understands that the manifestations of this disease often include A. acute back spasms and testicular pain B. rapid onset of scrotal swelling and fever C. fertility problems and bilateral scrotal tenderness D. painless mass and heaviness sensation in the scrotal area

D. painless mass and heaviness sensation in the scrotal area Rationale: Clinical manifestations of testicular cancer include a painless lump in the scrotum, scrotal swelling, and a feeling of heaviness. The scrotal mass usually is not tender and is very firm. Some patients complain of a dull ache or heavy sensation in the lower abdomen, perineal area, or scrotum

During routine assessment of a patient with Guillain-Barre syndrome, the nurse finds the patient is short of breath. The patient's respiratory distress is caused by A. elevated protein in the CSF B. immobility resulting from ascending paralysis C. degeneration of motor neurons in the brainstem and spinal cord D. paralysis ascending to the nerves that stimulate the thoracic area

D. paralysis ascending to the nerves that stimulate the thoracic area Rationale: Guillain-Barre syndrome is characterized by ascending, symmetric paralysis that usually affects cranial nerves and the peripheral nervous system. The most serious complication of this syndrome is respiratory failure, which occurs as the paralysis progresses to the nerves that innervate the thoracic area.

The clinical diagnosis of dementia is based on A. CT or MRI B. brain biopsy C. electroencephalogram D. patient history and cognitive assessment

D. patient history and cognitive assessment Rationale: The diagnosis of dementia depends on determining the cause. A thorough physical examination is performed to rule out other potential medical conditions. Cognitive testing (e.g. Mini-Mental State examination) is focused on evaluating memory, ability to calculate, language, visual spatial skills, and degree of alertness. Diagnosis of dementia related to vascular causes is based on the presence of cognitive loss, the presence of vascular brain lesions demonstrated by neuroimaging techniques, and the exclusion of other causes of dementia. Structural neuroimaging with CT scan or MRI is used in the evaluation of patients with dementia but does not provide a definitive diagnosis. A psychologic evaluation is also indicated to determine the presence of depression

After having a craniectomy and left anterior fossae incision, a 64-year-old patient has a nursing diagnosis of impaired physical mobility related to decreased level of consciousness and weakness. An appropriate nursing intervention is to A. cluster nursing activities to allow longer rest periods. B. turn and reposition the patient side to side every 2 hours. C. position the bed flat and log roll to reposition the patient. D. perform range-of-motion (ROM) exercises every 4 hours.

D. perform range-of-motion (ROM) exercises every 4 hours. Rationale: ROM exercises will help prevent the complications of immobility. Patients with anterior craniotomies are positioned with the head elevated. The patient with the craniectomy should not be turned to the operative side. When the patient is weak, clustering nursing activities may lead to more fatigue and weakness

The nurse will plan to teach a 51 year old man who is scheduled for an annual physical examination about a(n) A. increased risk for testicular cancer B. possible changes in erectile function C. normal decreases in testosterone level D. prostate specific antigen (PSA) testing

D. prostate specific antigen (PSA) testing Rationale: PSA testing may be recommended annually for men, starting at age 50. There is no indication that the other patient teaching topics are appropriate for this patient

To prevent the infection and transmission of STIs, the nurse's teaching plan would include an explanation of A. the appropriate use of oral contraceptives B. sexual positions that can be used to avoid infection C. the necessity of annual Pap tests for patients with HPV D. sexual practices that are considered high risk behaviors

D. sexual practices that are considered high risk behaviors Rationale: Many approaches to curtailing the spread of STIs have been advocated and have met with various degrees of success. Be prepared to discuss "safe" sex practices with all patients, not just those who are perceived to be at risk. These practices include abstinence, monogamy, avoidance of certain high risk sexual practices, and use of condoms and other barriers to limit contact with potentially infectious body fluids or lesions. Sexual abstinence is a certain method of avoiding all STIs, but few people consider this a feasible alternative to sexual expression. Limiting sexual intimacies outside of a well established monogamous relationship can reduce the risk of contracting an STI

The recommended treatment for an initial VTE in an otherwise healthy person with no significant co-morbidities would include A. IV argatroban (Acova) was an inpatient B. IV unfractionated heparin as an inpatient C. subcutaneous unfractioned heparin as an outpatient D. subcutaneous low molecular weight heparin as an outpatient

D. subcutaneous low molecular weight heparin as an outpatient Rationale: Patients with confirmed VTE should receive initial treatment with low molecular weight heparin (LMWH), unfractioned heparin (UH), fondaparinux, or rivaroxaban, followed by warfarin for 3 months to maintain the international normalized ratio (INR) between 2.0 and 3.0 for 24 hours. Patients with multiple comorbid conditions, complex medical issues, or a very large VTE usually are hospitalized for treatment and typically receive intravenous UH. LMWH only for 3 months is another option for patients with acute VTE. Depending on the clinical presentation, patients often can be managed safely and effectively as outpatients

Explain to the patient with gonorrhea that treatment will include both ceftriaxone and azithromycin because A. azithromycin helps prevent recurrent infections B. some patients do not respond to oral drugs alone C. coverage with more than one antibiotic will prevent reinfection D. the increasing rates of drug resistance requires the use of at least two drugs

D. the increasing rates of drug resistance requires the use of at least two drugs Rationale: The treatment of gonorrhea is a single intramuscular (IM) dose of ceftriaxone (Rocephin) in combination with azithromycin (Zithromax). Due to increasing rates of drug resistant gonorrhea, dual therapy is needed to address potential drug resistance

The nurse will anticipate that a 61 year old patient who has an enlarged prostate detected by digital rectal examination (DRE) and an elevated prostate specific antigen (PSA) level will need teaching about A. cystourethroscopy B. uroflowmetry studies C. magnetic resonance imaging (MRI) D. transrectal ultrasonography (TRUS)

D. transrectal ultrasonography (TRUS) Rationale: In a patient with an abnormal DRE and elevated PSA, transrectal ultrasound is used to visualize thee prostate for biopsy. Uroflowmetry studies help determine the extent of urine blockage and treatment, but there is no indication that this is a problem for this patient. Cytstoscopy may be used before prostatectomy but will not be done until after the TRUS and biopsy. MRI is used to determine whether prostatic cancer has metastasized but would not be ordered at this stage of the diagnostic process

The nurse suspects the presence of an arterial epidural hematoma in the patient who experiences A. failure to regain consciousness following a head injury B. a rapid deterioration of neurologic function within 24 to 48 hours following a head injury C. nonspecific, nonlocalizing progression of alteration in LOC occurring over weeks or months D. unconsciousness at the time of a head injury with a brief period of consciousness followed by a decrease in LOC

D. unconsciousness at the time of a head injury with a brief period of consciousness followed by a decrease in LOC Rationale: An arterial epidural hematoma is the most acute neurologic emergency, and typical symptoms include unconsciousness at the scene with a brief lucid interval followed by a decrease in LOC. An acute subdural hematoma manifests the signs within 48 hours of an injury. A chronic subdural hematoma develops over weeks or months

Male orgasm is characterized by A. resolution B. increased testicular size C. vasodilation and dystonia D. vasocongestion and myotonia

D. vasocongestion and myotonia Rationale: During the male sexual response, orgasm is characterized by the rapid release of the vasocongestion and myotonia have developed. During the male sexual response, testicle size increases during the plateau phase. After ejaculation, the male enters the resolution phase, with the penis undergoing involution

The nurse explains to a patient that being prepared for a colposcopy with a cervical biopsy that the procedure A. involves dilation of the cervix and biopsy of the tissue lining the uterus. B. will take place in a same day surgery center so that local anesthesia can be used. C. requires that the patient have nothing to eat or drink for 6 hours before the procedure D.is similar to a speculum examination of the cervix and should result in little discomfort

D.is similar to a speculum examination of the cervix and should result in little discomfort Rationale: Colposcopy involves visualization of the cervix with a binocular microscope and is similar to a speculum examination. Anesthesia is not required and fasting is not necessary. A cervical biopsy may cause a minimal amount of pain


Kaugnay na mga set ng pag-aaral

Chapter 7 - Review of Federal Law - Part 3

View Set

AP Psych quizzo/prac tests chapter 9 2/13/24

View Set

1035 Lecture 3 and 4 Practice Questions

View Set

Procedures III: Exam 2 - Digestive System-Esophagus/Stomach 2-5-18

View Set

Corporate Finance 5160 Exam #2 Review

View Set